Med Sug Success final set

¡Supera tus tareas y exámenes ahora con Quizwiz!

2

. 22. The client with a temperature of 94˚F is being treated in the ED. Which intervention should the nurse implement to directly elevate the client's temperature? 1. Remove the client's clothing. 2. Place a warm air blanket over the client. 3. Have the client change into a hospital gown. 4. Raise the temperature in the room.

Which interventions should the nurse discuss with the client diagnosed with coronary artery disease? Select all that apply. 1. Instruct the client to stop smoking. 2. Encourage the client to exercise three (3) days a week. 3. Teach about coronary vasodilators. 4. Prepare the client for a carotid endarterectomy. 5. Eat foods high in monosaturated fats.

1,2,3 1. Smoking is the one risk factor that must be stopped totally; there is no compromise. 2.Exercising helps develop collateral circulation and decrease anxiety; it also helps clients to lose weight. 3. Clients with coronary artery disease are usually prescribed nitroglycerin, which is the treatment of choice for angina.

The client with coronary artery disease asks the nurse, "Why do I get chest pain?" Which statement would be the most appropriate response by the nurse? 1. "Chest pain is caused by decreased oxygen to the heart muscle." 2. "There is ischemia to the myocardium as a result of hypoxemia." 3. "The heart muscle is unable to pump effectively to perfuse the body." 4. "Chest pain occurs when the lungs cannot adequately oxygenate the blood."

1. "Chest pain is caused by decreased oxygen to the heart muscle." 1. This is a correct statement presented in layman's terms. When the coronary arteries cannot supply adequate oxygen to the heart muscle, there is chest pain. 2. This is the explanation in medical terms that should not be used when explaining medical conditions to a client. 3. This explains congestive heart failure but does not explain why chest pain occurs. 4. Respiratory compromise occurs when the lungs cannot oxygenate the blood, such as occurs with altered level of consciousness, cyanosis, and increased respiratory rate. TEST-TAKING HINT: The nurse must select the option that best explains the facts in terms a client who does not have medical training can understand.

The client with a mechanical valve replacement asks the nurse, "Why do I have to take antibiotics before getting my teeth cleaned?" Which response by the nurse is most appropriate? 1. "You are at risk of developing an infection in your heart." 2. "Your teeth will not bleed as much if you have antibiotics." 3. "This procedure may cause your valve to malfunction." 4. "Antibiotics will prevent vegetative growth on your valves."

1. "You are at risk of developing an infection in your heart." 1. The client is at risk for developing endocarditis and should take prophylactic antibiotics before any invasive procedure. 2. Antibiotics have nothing to do with how much the teeth bleed during a cleaning. 3. Teeth cleaning will not cause the valve to malfunction. 4. Vegetation develops on valves secondary to bacteria that cause endocarditis, but the client may not understand "vegetative growth on your valves"; therefore, this is not the most appropriate answer. TEST-TAKING HINT: The test taker should select an option that answers the client's question in the easiest and most understandable terms, not in medical jargon. This would cause the test taker to eliminate option "4" as a possible correct answer. The test taker should know antibiotics do not affect bleeding and so can eliminate option "2."

1. The elderly client being seen in the clinic has complaints of urinary frequency, urgency, and "leaking." Which priority intervention should the nurse implement when interviewing the client? 1. Ensure communication is nonjudgmental and respectful. 2. Set the temperature for comfort in the examination room. 3. Speak loudly to ensure the client understands the nurse. 4. Ensure the examining room has adequate lighting.

1. 1. Clients who have urinary incontinence are often embarrassed, so it is the responsibility of the nurse to approach this subject with respect and consideration.

The nurse is preparing to administer warfarin (Coumadin), an oral anticoagulant, to a client with a mechanical valve replacement. The client's international normalized ratio (INR) is 2.7. Which action should the nurse implement? 1. Administer the medication as ordered. 2. Prepare to administer vitamin K (AquaMephyton). 3. Hold the medication and notify the HCP. 4. Assess the client for abnormal bleeding.

1. Administer the medication as ordered. 1. The therapeutic range for most clients' INR is 2 to 3, but for a client with a mechanical valve replacement it is 2 to 3.5. The medication should be given as ordered and not withheld. 2. Vitamin K is the antidote for an overdose of warfarin, but 2.7 is within therapeutic range. 3. This laboratory result is within the therapeutic range, INR 2 to 3, and the medication does not need to be withheld. 4. There is no need for the nurse to assess for bleeding because 2.7 is within therapeutic range. TEST-TAKING HINT: The test taker has to know the therapeutic range for the INR to be able to answer this question correctly. The test taker should keep a list of normal and therapeutic laboratory values that must be remembered.

The nurse is assessing the client diagnosed with congestive heart failure. Which laboratory data would indicate that the client is in severe congestive heart failure? 1. An elevated B-type natriuretic peptide (BNP). 2. An elevated creatine kinase (CK-MB). 3. A positive D-dimer. 4. A positive ventilation/perfusion (V/Q) scan.

1. An elevated B-type natriuretic peptide (BNP). 1. BNP is a specific diagnostic test. Levels higher than normal indicate congestive heart failure, with the higher the number, the more severe the CHF. 2. An elevated CK-MB would indicate a myocardial infarction, not severe CHF. CK-MB is an isoenzyme. 3. A positive D-dimer would indicate a pulmonary embolus. 4. A positive ventilation/perfusion (V/Q) scan (ratio) would indicate a pulmonary embolus. TEST-TAKING HINT: This question requires the test taker to discriminate between CHF, MI, and PE. If unsure of the answerof this type of question, the test taker should eliminate any answer options that the test taker knows are wrong. For example, the test taker may not know about pulmonary embolus but might know that CK-MB data are used to monitor MI and be able to eliminate option "2" as a possibility. Then, there is a 1:3 chance of getting the correct answer.

42. The client receiving dialysis for end-stage renal disease wants to quit dialysis and die. Which ethical principle supports the client's right to die? 1. Autonomy. 2. Self-determination. 3. Beneficence. 4. Justice.

1. Autonomy implies the client has the right to make choices and decisions about his or her own care even if it may result in death or is not in agreement with the health-care team

The client comes to the emergency department saying, "I am having a heart attack." Which question is most pertinent when assessing the client? 1. "Can you describe your chest pain?" 2. "What were you doing when the pain started?" 3. "Did you have a high-fat meal today?" 4. "Does the pain get worse when you lie down?"

1. Can you describe your chest pain? The chest pain for an MI usually is described as an elephant sitting on the chest or a belt squeezing the substernal midchest, often radiating to the jaw or left arm.

18. The client diagnosed with ESRD has a new arteriovenous fistula in the left forearm. Which intervention should the nurse implement? 1. Teach the client to carry heavy objects with the right arm. 2. Perform all laboratory blood tests on the left arm. 3. Instruct the client to lie on the left arm during the night. 4. Discuss the importance of not performing any hand exercises.

1. Carrying heavy objects in the left arm could cause the fistula to clot by putting undue stress on the site, so the client should carry objects in the right arm.

The hospice care nurse is planing the care of an elderly client diagnosed with endstage renal disease. Which interventions should be included in the plan of care? Select all that apply 1. Discuss financial concerns 2. Assess any comorbid conditions 3. Monitor increased visual or auditory abilities. 4. Note any spiritual distress. 5. Encourage euphoria at the time of death.

1. Discuss financial concerns. The elderly are frequently on fixed incomes and financial concerns are important of the nurses to address. A social services referral may be needed. 2. Assess any comorbid conditions. The elderly may have co-morbid conditions, which affect the type and amount of medications the client can tolerate and the quality of the clients life. 4. Note any spiritual distress. The client may feel some spiritual distress at the terminal diagnosis. Even if the client processes a strong faith, the unknown can be frightening

The client's telemetry reading shows a P wave before each QRS complex and the rate is 78. Which action should the nurse implement? 1. Document this as normal sinus rhythm. 2. Request a 12-lead electrocardiogram. 3. Prepare to administer the cardiotonic digoxin PO. 4. Assess the client's cardiac enzymes.

1. Document this as normal sinus rhythm. 1. The P wave represents atrial contraction, and the QRS complex represents ventricular contraction—a normal telemetry reading. A rate between 60 and 100 indicates normal sinus rhythm. Therefore, the nurse should document this as normal sinus rhythm and not take any action. 2. A 12-lead ECG should be requested for chest pain or abnormal dysrhythmias. 3. Digoxin is used to treat atrial fibrillation. 4. Cardiac enzymes are monitored to determine if the client has had a myocardial infarction. Nothing in the stem indicates the client has had an MI. TEST-TAKING HINT: The test taker must know normal sinus rhythm, and there are no Test-Taking Hints to help eliminate incorrect options. The test taker should not automatically select assessment as the correct answer, but if the test taker had no idea of the answer, remember assessment of laboratory data is not the same as assessing the client.

41. The client is reporting chills, fever, and left costovertebral pain. Which diagnostic test would the nurse expect the HCP to order first? 1. A midstream urine for culture. 2. A sonogram of the kidney. 3. An intravenous pyelogram for renal calculi. 4. A CT scan of the kidneys.

1. Fever, chills, and costovertebral pain are symptoms of a urinary tract infection (acute pyelonephritis), which requires a urine culture first to confirm the diagnosis

65. The nurse in an outpatient rehabilitation facility is working with convicted child abusers. Which characteristics should the nurse expect to observe in the abusers? Select all that apply. 1. The abuser calls the child a liar. 2. The abuser has a tendency toward violence. 3. The abuser exhibits a high self-esteem. 4. The abuser is unable to admit the need for help. 5. The abuser was spoiled as a child.

1. Frequently child abusers will deny the child's reports of abuse and say the child is a habitual liar. 2. Child abusers believe violence is an acceptable way to reduce tension. They tend to have a low tolerance for frustration and have poor impulse control. 4. Child abusers tend to blame the child for the abuse and not admit the problem is their own. 3. Child abusers have a tendency toward feelings of helplessness and hopelessness. 5. The child abuser may have been abused as a child, but there is no evidence of the child abuser being spoiled as a child. TEST-TAKING HINT: This is an alternative-type question. The test taker should examine each option carefully to determine if it could be a correct answer. Option "3" could be eliminated because of the adjective "high" and "5" could be eliminated because of the adjective "spoiled."

Which medical client problem should the nurse include in the plan of care for a client diagnosed with cardiomyopathy? 1. Heart failure. 2. Activity intolerance. 3. Powerlessness. 4. Anticipatory grieving.

1. Heart failure Medical client problems indicate the nurse and the physician must collaborate to care for the client; the client must have medications for heart failure.

The client has chronic atrial fibrillation. Which discharge teaching should the nurse discuss with the client? 1. Instruct the client to use a soft-bristle toothbrush. 2. Discuss the importance of getting a monthly partial thromboplastin time (PTT). 3. Teach the client about signs of pacemaker malfunction. 4. Explain to the client the procedure for synchronized cardioversion.

1. Instruct the client to use a soft-bristle toothbrush. 1. A client with chronic atrial fibrillation will be taking an anticoagulant to help prevent clot formation. Therefore, the client is at risk for bleeding and should be instructed to use a soft-bristle toothbrush. 2. The client will need a regularly scheduled INR to determine the therapeutic level for the anticoagulant warfarin (Coumadin); PTT levels are monitored for heparin. 3. A client with symptomatic sinus bradycardia, not a client with atrial fibrillation, may need a pacemaker. 4. Synchronized cardioversion may be prescribed for new-onset atrial fibrillation but not for chronic atrial fibrillation. TEST-TAKING HINT: The key to answering this question is the adjective "chronic." The test taker must know that HCPs prescribe anticoagulant therapy for clients with chronic atrial fibrillation.

Which signs/symptoms should the nurse assess in any client who has a long-term valvular heart disease? Select all that apply. 1. Paroxysmal nocturnal dyspnea. 2. Orthopnea. 3. Cough. 4. Pericardial friction rub. 5. Pulsus paradoxus.

1. Paroxysmal nocturnal dyspnea. 2. Orthopnea. 3. Cough. 1. Paroxysmal nocturnal dyspnea is a sudden attack of respiratory distress, usually occurring at night because of the reclining position, and occurs in valvular disorders. 2. This is an abnormal condition in which a client must sit or stand to breathe comfortably and occurs in valvular disorders. 3. Coughing occurs when the client with long-term valvular disease has difficulty breathing when walking or performing any type of activity. 4. Pericardial friction rub is a sound auscultated in clients with pericarditis, not valvular heart disease. 5. Pulsus paradoxus is a marked decrease in amplitude during inspiration. It is a sign of cardiac tamponade, not valvular heart disease. TEST-TAKING HINT: The test taker should notice that options "1," "2," and "3" are all signs/symptoms that have something to do with the lungs. It would be a good choice to select these three as correct answers. They are similar in description.

10. The client is admitted to the emergency department after a gunshot wound to the abdomen. Which nursing intervention would the nurse implement first to prevent ARF? 1. Administer normal saline IV. 2. Take vital signs. 3. Place client on telemetry. 4. Assess abdominal dressing

1. Preventing and treating shock with blood and fluid replacement will prevent acute renal failure from hypoperfusion of the kidneys. Significant blood loss would be expected in the client with a gunshot wound.

4. The client is diagnosed with ARF. Which signs/symptoms would indicate to the nurse that the client is in the recovery period? Select all that apply. 1. Increased alertness and no seizure activity. 2. Increase in hemoglobin and hematocrit. 3. Denial of nausea and vomiting. 4. Decreased urine-specific gravity. 5. Increased serum creatinine level.

1. Renal failure affects almost every system in the body. Neurologically the client may have drowsiness, headache, muscle twitching, and seizures. In the recovery period, the client would be alert and not have seizures. 2. In renal failure, levels of erythropoietin are decreased, leading to anemia. An increase in hemoglobin and hematocrit indicates the client is in the recovery period. 3. Nausea, vomiting, and diarrhea are common in the client with ARF; therefore, an absence of these indicates the client is in the recovery period.

The nurse is told in report the client has aortic stenosis. Which anatomical position should the nurse auscultate to assess the murmur? 1. Second intercostal space, right sternal notch. 2. Erb's point. 3. Second intercostal space, left sternal notch. 4. Fourth intercostal space, left sternal border.

1. Second intercostal space, right sternal notch. The second intercostal space, right sternal notch, is the area on the chest where the aorta can best be heard opening and closing.

52. When preparing a teaching plan for the client with chronic prostatitis, which intervention should the nurse include? 1. Sit in a warm sitz bath for 10 to 20 minutes several times daily. 2. Sit in the chair with the feet elevated for two (2) hours daily. 3. Drink at least 3000 mL of oral fluids, especially tea and coffee, daily. 4. Stop broad-spectrum antibiotics as soon as the symptoms subside.

1. The client should sit in a warm sitz bath for 10-20 minutes several times each day to provide comfort and assist with healing.

21. The client receiving dialysis is complaining of being dizzy and light-headed. Which action should the nurse implement first? 1. Place the client in the Trendelenburg position. 2. Turn off the dialysis machine immediately. 3. Bolus the client with 500 mL of normal saline. 4. Notify the health-care provider as soon as possible.

1. The nurse should place the client's chair with the head lower than the body, which will shunt blood to the brain; this is the Trendelenburg position.

Which data would cause the nurse to question administering digoxin to a client diagnosed with congestive heart failure? 1. The potassium level is 3.2 mEq/L. 2. The digoxin level is 1.2 mcg/mL. 3. The client's apical pulse is 64. 4. The client denies yellow haze.

1. The potassium level is 3.2 mEq/L This potassium level is below normal levels; hypokalemia can potentiate digoxin toxicity and lead to cardiac dysrhythmias.

54. Which data would indicate that discharge teaching has been effective for the client who is postoperative TURP? 1. "I will call the surgeon if I experience any difficulty urinating." 2. "I will take my Proscar daily, the same as before my surgery." 3. "I will continue restricting my oral fluid restriction." 4. "I will need to take my pain medication routinely even if I do not hurt."

1. This indicates that teaching is effective. 2. Clients do not need to take Proscar postoperatively. 3. There is no reason to restrict the client's fluid intake. 4. Pain medication should be taken as needed.

82. Which information regarding the care of a cutaneous ileal conduit should the nurse teach? 1. Teach the client to instill a few drops of vinegar into the pouch. 2. Tell the client that the stoma should be slightly dusky colored. 3. Inform the client that large clumps of mucus are expected. 4. Tell the client that it is normal for the urine to be pink or red in color.

1. Vinegar will act as a deodorizing agent in the pouch and help prevent a strong urine smell.

2

12. The ED nurse is caring for a client diagnosed with frostbite of the feet. Which intervention should the nurse implement? 1. Massage the feet vigorously. 2. Soak the feet in warm water. 3. Apply a heating pad to feet. 4. Apply petroleum jelly to feet.

4

13. A student reports to the school nurse with complaints of stinging and burning from a wasp sting. Which intervention should the nurse implement? 1. Grasp the stinger and pull it out. 2. Apply a warm, moist soak to the area. 3. Cleanse the site with alcohol. 4. Apply an ice pack to the site.

2

19. Which problem is most appropriate for the nurse to identify for the client experiencing renal trauma? 1. Infection of the renal tract. 2. Ineffective tissue perfusion. 3. Alteration in skin integrity. 4. Alteration in temperature.

2

19. Which signs/symptoms should the nurse assess in the client who has been exposed to the anthrax bacillus via the skin? 1. A scabby, clear fluid-filled vesicle. 2. Edema, pruritus, and a 2-mm ulcerated vesicle. 3. Irregular brownish-pink spots around the hairline. 4. Tiny purple spots flush with the surface of the skin.

The client is admitted to the emergency department, and the nurse suspects a cardiac problem. Which assessment interventions should the nurse implement? Select all that apply. 1. Obtain a midstream urine specimen. 2. Attach telemetry monitor to the client. 3. Start a saline lock in the right arm. 4. Draw a basal metabolic panel (BMP). 5. Request an order for a STAT 12-lead ECG.

2,3,5 2. Anytime a nurse suspects cardiac problems, the electrical conductivity of the heart should be assessed. 3. Emergency medications for heart problems are primarily administered intravenously, so starting a saline lock in the right arm is appropriate. 5.A 12-lead ECG evaluates the electrical conductivity of the heart from all planes.

The client is admitted to the medical unit to rule out carditis. Which question should the nurse ask the client during the admission interview to support this diagnosis? 1. "Have you had a sore throat in the last month?" 2. "Did you have rheumatic fever as a child?" 3. "Do you have a family history of carditis?" 4. "What over-the-counter (OTC) medications do you take?"

2. "Did you have rheumatic fever as a child?" 1. A sore throat in the last month would not support the diagnosis of carditis. 2. Rheumatic fever, a systemic inflammatory disease caused by an abnormal immune response to pharyngeal infection by group A beta-hemolytic streptococci, causes carditis in about 50% of people who develop it. 3. Carditis is not a genetic or congenital disease process. 4. This is an appropriate question to ask any client, but OTC medications do not cause carditis. TEST-TAKING HINT: This is a knowledgebased question, but the test taker could eliminate option "4," realizing this is a question to ask any client, and the stem asks which question will support the diagnosis of carditis.

The client tells the nurse, "Every time I come in the hospital you hand me one of these advance directives (AD). Why should I fill one of these out?" Which statement by the nurse is most appropriate? 1. "You must fill out this form because Medicare laws require it." 2. "An AD lets you participate in decisions about your health care." 3. "This paper will ensure no one can override your decisions." 4. "It is part of the hospital admission packet and I have to give it to you."

2. ADs allow the client to make personal health-care decisions about end-of-life issues, including cardiopulmonary resuscitation (CPR), ventilators, feeding tubes, and other issues concerning the client's death.

34. The client diagnosed with tuberculosis has been treated with antitubercular medications for six (6) weeks. Which data would indicate the medications have been effective? 1. A decrease in the white blood cells in the sputum. 2. The client's symptoms are improving. 3. No change in the chest x-ray. 4. The skin test is now negative.

2. As the bacilli are being destroyed, the client should begin to feel better and have fewer symptoms.

The nurse is caring for a client who goes into ventricular tachycardia. Which intervention should the nurse implement first? 1. Call a code immediately. 2. Assess the client for a pulse. 3. Begin chest compressions. 4. Continue to monitor the client.

2. Assess the client for a pulse. The nurse must first determine if the client has a pulse. Pulseless ventricular tachycardia is treated as ventricular fibrillation. Stable ventricular tachycardia is treated with medications.

62. The client diagnosed with renal calculi is admitted to the medical unit. Which intervention should the nurse implement first? 1. Monitor the client's urinary output. 2. Assess the client's pain and rule out complications. 3. Increase the client's oral fluid intake. 4. Use a safety gait belt when walking the client.

2. Assessment is the first part of the nursing process and is always priority. The intensity of the renal colic pain can be so intense it can cause a vasovagal response, with resulting hypotension and syncope.

8. The client diagnosed with ARF is placed on bed rest. The client asks the nurse, "Why do I have to stay in bed, I don't feel that bad." Which scientific rationale would support the nurse's response? 1. Bed rest helps increase the blood return to the renal circulation. 2. Bed rest reduces the metabolic rate during the acute stage. 3. Bed rest decreases the workload of the left side of the heart. 4. Bed rest aids in reduction of peripheral and sacral edema.

2. Bed rest reduces exertion and the metabolic rate, thereby reducing catabolism and subsequent release of potassium and accumulation of endogenous waste products (urea and creatinine).

8. The client with type 2 diabetes controlled with biguanide oral diabetic medication is scheduled for a computed tomography (CT) scan with contrast of the abdomen to evaluate pancreatic function. Which intervention should the nurse implement? 1. Provide a high-fat diet 24 hours prior to test. 2. Hold the biguanide medication for 48 hours prior to test. 3. Obtain an informed consent form for the test. 4. Administer pancreatic enzymes prior to the test.

2. Biguanide medication must be held for a test with contrast medium because it increases the risk of lactic acidosis, which leads to renal problems.

2. The client is experiencing urinary incontinence. Which intervention should the nurse implement? 1. Teach the client to drink prune juice weekly. 2. Encourage the client to eat a high-fiber diet. 3. Discuss the need to urinate every six (6) hours. 4. Explain the importance of wearing cotton underwear.

2. Clients experiencing incontinence should eat a high-fiber diet to avoid constipation, which increases pressure on the bladder, which may increase incontinence.

13. The nurse is teaching the female client diagnosed with tuberculosis of the urinary tract prior to discharge. Which information should the nurse include specific to this diagnosis? 1. Instruct the client to take the medication with food. 2. Explain condoms should be used during treatment. 3. Discuss the need for follow-up chest x-rays. 4. Encourage a well-balanced diet and fluid intake.

2. Clients who have been diagnosed with tuberculosis of the renal tract should use condoms to prevent transmission of the mycobacterium. If the infection is located in the penis or urethra, abstaining from sexual activity is recommended.

61. The client diagnosed with a pituitary tumor developed syndrome of inappropriate antidiuretic hormone (SIADH). Which interventions should the nurse implement? 1. Assess for dehydration and monitor blood glucose levels. 2. Assess for nausea and vomiting and weigh daily. 3. Monitor potassium levels and encourage fluid intake. 4. Administer vasopressin IV and conduct a fluid deprivation test.

2. Early signs and symptoms are nausea and vomiting. The client has the syndrome of inappropriate secretion of antidiuretic (against allowing the body to urinate) hormone. In other words, the client is producing a hormone that will not allow the client to urinate.

55. The client is one (1) day postoperative TURP. Which nursing task can be delegated to the unlicensed assistive personnel? 1. Increase the irrigation fluid to clear clots from the tubing. 2. Elevate the scrotum on a towel roll for support. 3. Change the dressing on the first postoperative day. 4. Teach the client how to care for the continuous irrigation catheter.

2. Elevating the scrotum on a towel for support is an intervention that can be delegated to the UAP

42. The client with chronic pyelonephritis is being admitted to a medical unit for intensive intravenous therapy. Which assessment data support the diagnosis of chronic pyelonephritis? 1. The client has fever, chills, flank pain, and dysuria. 2. The client complains of fatigue, headaches, and increased urination. 3. The client had a group b beta hemolytic strep infection last week. 4. The client has an acute viral pneumonia infection.

2. Fatigue, headache, and polyuria as well as loss of weight, anorexia, and excessive thirst are symptoms of chronic pyelonephritis.

The nurse is orienting to a hospice organization. Which statement does not indicate a right of the terminal client? The right to; 1. Be treated with respect and dignity 2. Have particulars of the death withheld 3. Receive optimal and effective pain management 4. Receive holistic and compassionate care.

2. Have particulars of the death withheld. The client has the right to discuss his or her feelings and direct his or her care. Withholding information would be lying to the client

36. The client diagnosed with acute pancreatitis is being discharged home. Which statement by the client indicates the teaching has been effective? 1. "I should decrease my intake of coffee, tea, and cola." 2. "I will eat a low-fat diet and avoid spicy foods." 3. "I will check my amylase and lipase levels daily." 4. "I will return to work tomorrow but take it easy."

2. High-fat and spicy foods stimulate gastric and pancreatic secretions and may precipitate an acute pancreatic attack.

84. Which signs/symptoms should make the nurse suspect the client is experiencing a thyroid storm? 1. Obstipation and hypoactive bowel sounds. 2. Hyperpyrexia and extreme tachycardia. 3. Hypotension and bradycardia. 4. Decreased respirations and hypoxia.+

2. Hyperpyrexia (high fever) and heart rate above 130 beats per minute are signs of thyroid storm, a severely exaggerated hyperthyroidism.

3. The client is diagnosed with rule out ARF. Which condition would predispose the client to developing pre-renal failure? 1. Diabetes mellitus. 2. Hypotension. 3. Aminoglycosides. 4. Benign prostatic hypertrophy.

2. Hypotension, which causes a decreased blood supply to the kidney, is one of the most common causes of pre-renal failure (before the kidney).

17. The nurse is preparing to administer influenza vaccines to a group of elderly clients in a long-term care facility. Which client should the nurse question receiving the vaccine? 1. The client diagnosed with congestive heart failure. 2. The client with a documented allergy to eggs. 3. The client who has had an anaphylactic reaction to penicillin. 4. The client who has an elevated blood pressure and pulse.

2. In clients who are allergic to egg protein, a significant hypersensitivity response may occur when they are receiving the influenza vaccine.

48. The nurse is discussing malpractice issues in an in-service class. Which situation is an example of malpractice? 1. The nurse fails to report a neighbor who is abusing his two children. 2. The nurse does not intervene in a client who has a BP of 80/50 and AP of 122. 3. The nurse is suspected of taking narcotics prescribed for a client. 4. The nurse falsifies vital signs in the client's medical records.

2. Malpractice is a failure to meet the standards of care which results in harm to or death of a client. Failing to heed warnings of shock is an example of malpractice.

69. Which intervention is most important for the nurse to implement for the client diagnosed with rule out renal calculi? 1. Assess the client's neurological status every 2 hours. 2. Strain all urine and send any sediment to the laboratory. 3. Monitor the client's creatinine and BUN levels. 4. Take a 24-hour dietary recall during the client interview.

2. Passing a renal stone may negate the need for the client to have lithotripsy or a surgical procedure. Therefore, all urine must be strained, and a stone, if found, should be sent to the laboratory to determine what caused the stone.

79. The male client diagnosed with metastatic cancer of the bladder is emaciated and refuses to eat. Which nursing action is an example of the ethical principle of paternalism? 1. The nurse allows the client to talk about not wanting to eat. 2. The nurse tells the client that if he does not eat, a feeding tube will be placed. 3. The nurse consults the dietitian about the client's nutritional needs. 4. The nurse asks the family to bring favorite foods for the client to eat.

2. Paternalism is deciding for the client what is best, such as a parent making decisions for a child. Feeding a client, as with a feeding tube, without the client wishing to eat is paternalism.

19. The male client diagnosed with ESRD secondary to diabetes has been receiving dialysis for 12 years. The client is notified that he will not be placed on the kidney transplant list. The client tells the nurse he will not be back for any more dialysis treatments. Which response would be most therapeutic? 1. "You cannot just quit your dialysis. This is not an option." 2. "Are you angry at not being on the list, so you want to quit dialysis?" 3. "I will call your nephrologist right now so you can talk to the HCP." 4. "Make your funeral arrangements because you are going to die."

2. Reflecting the client's feelings and restating them are therapeutic responses that the nurse should use when addressing the client's issues.

20. The nurse is discussing kidney transplants with clients at a dialysis center. Which population is less likely to participate in organ donation? 1. Caucasian. 2. African American. 3. Asian. 4. Hispanic.

2. The African American culture believes that the body must be kept intact after death, and organ donation is rare among African Americans. This is also why a client of African American descent will be on a transplant waiting list longer than people of other races. This is because of tissuetyping compatibility.

14. The nurse is discussing placing the client diagnosed with chronic obstructive pulmonary disease (COPD) in hospice care. Which prognosis must be determined to place the client in hospice care? 1. The client is doing well but could benefit from the added care by hospice. 2. The client has a life expectancy of six (6) months or less. 3. The client will live for about one (1) to two (2) more years. 4. The client has about eight (8) weeks to live and needs pain control.

2. The HCP must think that, without life-prolonging treatment, the client has a life expectancy of six (6) months or less. The client may continue receiving hospice care if the client lives longer.

57. The nurse and an unlicensed assistive personnel (UAP) are caring for clients on a postoperative transplant unit. Which task should the nurse delegate to the UAP? 1. Assess the hourly outputs of the client who is post-kidney transplantation. 2. Raise the head of the bed for a client who is post-liver transplantation. 3. Monitor the serum blood studies of a client who has rejected an organ. 4. Irrigate the nasogastric tube of the client who had a pancreas transplant.

2. The UAP can perform this function. There is no nursing judgment required

17. The nurse is discussing how to prioritize care with the UAP. Which client should the nurse instruct the UAP to see first? 1. The immobile client who needs sequential compression devices removed. 2. The elderly woman who needs assistance ambulating to the bathroom. 3. The surgical client who needs help changing the gown after bathing. 4. The male client who needs the intravenous catheter discontinued.

2. The elderly woman may have age-related changes (decreased bladder capacity, weakened urinary sphincter, and shortened urethra) causing urinary urgency or incontinence. The elderly client is at risk for falling while attempting to get to the bathroom, so this client should be seen first.

1

2. The nurse is caring for a client in the ED with abdominal trauma who has had peritoneal lavage. Which intervention should the nurse include in the plan of care? 1. Assess for the presence of blood, bile, or feces. 2. Palpate the client for bilateral femoral pulses. 3. Perform Leopold's maneuver every eight (8) hours. 4. Collect information on the client's dietary history.

Which situation would cause the nurse to question the validity of an AD when caring for the elderly client? 1. The client's child insists the client make his or her own decisions. 2. The nurse observes the wife making the husband sign the AD. 3. A nurse encouraged the client to think about end-of-life decisions. 4. A friend witnesses the client's signature on the AD form.

2. This is coercion and is illegal when signing an AD. The AD must be signed by the client's own free will; an AD signed under duress may not be valid.

44. Which element is not necessary to prove nursing malpractice? 1. Breach of duty. 2. Identify the ethical issues. 3. Injury to the client. 4. Proximate cause.

2. This is one (1) of the four (4) steps in ethical decision making. It is not one (1) of the four (4) elements necessary to prove nursing malpractice.

14. The nurse is teaching a class on ethical principles in nursing. Which statement supports the definition of beneficence? 1. The duty to prevent or avoid doing harm. 2. The duty to actively do good for clients. 3. The duty to be faithful to commitments. 4. The duty to tell the truth to the clients.

2. This is the ethical principle of beneficence.

20. The nurse and an unlicensed assistive personnel (UAP) are caring for an elderly client diagnosed with emphysema. Which nursing tasks could be delegated to the UAP to improve gas exchange? Select all that apply. 1. Keep the head of the bed elevated. 2. Encourage deep breathing exercises. 3. Record pulse oximeter reading. 4. Assess level of conscious. 5. Auscultate breath sounds.

20. 1. Keeping the head of the bed elevated maximizes lung excursion and improves gas exchange and can be delegated. 2. Encouraging breathing exercises can be delegated. 3. Recording pulse oximeter readings can be delegated. Evaluating is the responsibility of the nurse.

22. Which problem is appropriate for the nurse to identify for the client who is one (1) day postoperative thoracotomy? 1. Alteration in comfort. 2. Altered level of conscious. 3. Alteration in elimination pattern. 4. Knowledge deficit.

22. 1. Pain and discomfort are major problems for a client who had a thoracotomy because the chest wall has been opened and closed.

The elderly client has coronary artery disease. Which question should the nurse ask the client during the client teaching? 1. "Do you have a daily bowel movement?" 2. "Do you get yearly chest x-rays?" 3. "Are you sexually active?" 4. "Have you had any weight change?"

3. "Are you sexually active?" 1. Bowel movements are important, but they are not pertinent to coronary artery disease. 2. Chest x-rays are usually done for respiratory problems, not for coronary artery disease. 3. Sexual activity is a risk factor for angina resulting from coronary artery disease. The client's being elderly should not affect the nurse's assessment of the client's concerns about sexual activity. 4. Weight change is not significant in a client with coronary artery disease. TEST-TAKING HINT: Remember, if the client is described with an adjective such as "elderly," this may be the key to selecting the correct answer. The nurse must not be judgmental about the elderly, especially about issues concerning sexual activity.

The female client is diagnosed with rheumatic fever and prescribed penicillin, an antibiotic. Which statement indicates the client needs more teaching concerning the discharge teaching? 1. "I must take all the prescribed antibiotics." 2. "I may get a vaginal yeast infection with penicillin." 3. "I will have no problems as long as I take my medication." 4. "My throat culture was positive for a streptococcal infection."

3. "I will have no problems as long as I take my medication." 1. The full course of antibiotics must be taken to help ensure complete destruction of streptococcal infection. 2. Antibiotics kill bacteria but also destroy normal body flora in the vagina, bowel, and mouth, leading to a superinfection. 3. Even with antibiotic treatment for rheumatic fever, the client may experience bacterial endocarditis in later years and should know this may occur. 4. A throat culture is taken to diagnose group A beta-hemolytic streptococcus and is positive in 25% to 40% of clients with acute rheumatic fever. TEST-TAKING HINT: The question is asking the test taker to identify which statement indicates the client does not understand the teaching; this is an "except" question. The test taker can ask which statement indicates the teaching is effective and choose the one option that is not appropriate.

The male client diagnosed with chronic pain since a construction accident that broke several vertebrae tells the nurse that he has been referred to a pain clinic and asks, "What good will it do? I will never be free of this pain." Which statement is the nurse's best response? 1. "Are you afraid of the pain never going away?" 2. "The clinic will give you medication to cure the pain" 3. "Pain clinics work to help you achieve relief from pain" 4. "I am not sure. You should discuss this with your HCP."

3. "Pain clinics work to help you achieve relief from pain" Pain clinics use a variety of methods to help the client to achieve relief from pain. Some measures include guided imagery, TENs unit, nerve block surgery or injections or medications

3. Which information indicates to the nurse the client teaching about treatment of urinary incontinence has been effective? 1. The client prepares a scheduled voiding plan. 2. The client verbalizes the need to increase fluid intake. 3. The client explains how to perform pelvic floor exercises. 4. The client attempts to retain the vaginal cone in place the entire day.

3. 1. Scheduled voiding allows the client to void every two (2) to three (3) hours apart, and when the client has remained consistently dry, the interval is increased by about 15 minutes.

The client is diagnosed with pericarditis. When assessing the client, the nurse is unable to auscultate a friction rub. Which action should the nurse implement? 1. Notify the health-care provider. 2. Document that the pericarditis has resolved. 3. Ask the client to lean forward and listen again. 4. Prepare to insert a unilateral chest tube.

3. Ask the client to lean forward and listen again. Having the client lean forward and to the left uses gravity to force the heart nearer to the chest wall, which allows the friction rub to be heard.

The client has an implantable cardioverter defibrillator (ICD). Which discharge instructions should the nurse teach the client? 1. Do not lift or carry more than 23 kg. 2. Have someone drive the car for the rest of your life. 3. Carry the cell phone on the opposite side of the ICD. 4. Avoid using the microwave oven in the home.

3. Carry the cellphone on the opposite side of the ICD Cell phones may interfere with the functioning of the ICD if they are placed too close to it

74. Which is a modifiable risk factor for the development of cancer of the bladder? 1. Previous exposure to chemicals. 2. Pelvic radiation therapy. 3. Cigarette smoking. 4. Parasitic infections of the bladder

3. Cigarette smoke contains more than 400 chemicals, 17 of which are known to cause cancer. The risk is directly proportional to the amount of smoking.

25. The client is admitted to a nursing unit from a long-term care facility with a hematocrit of 56% and a serum sodium level of 152 mEq/L. Which condition would be a cause for these findings? 1. Overhydration. 2. Anemia. 3. Dehydration. 4. Renal failure.

3. Dehydration results in concentrated serum that causes lab values to increase because the blood has normal constituents but not enough volume to dilute the values to within normal range or possibly lower.

61. The laboratory data reveal a calcium phosphate renal stone for a client diagnosed with renal calculi. Which discharge teaching intervention should the nurse implement? 1. Encourage the client to eat a low-purine diet and limit foods such as organ meats. 2. Explain the importance of not drinking water two (2) hours before bedtime. 3. Discuss the importance of limiting vitamin D-enriched foods. 4. Prepare the client for extracorporeal shock wave lithotripsy (ESWL).

3. Dietary changes for preventing renal stones include reducing the intake of the primary substance forming the calculi. In this case, limiting vitamin D will inhibit the absorption of calcium from the gastrointestinal tract

43. Which federal agency is a resource for the nurse volunteering at the American Red Cross who is on a committee to prepare the community for any type of disaster? 1. The Joint Commission (JC). 2. Office of Emergency Management (OEM). 3. Department of Health and Human Services (DHHS). 4. Metro Medical Response Systems (MMRS).

3. Federal resources include organizations such as DHHS and the Department of Justice. Each of these federal departments oversees hundreds of agencies, including the American Red Cross, which respond to disasters. 1. This organization mandates all health-care facilities to have an emergency operations plan, but it is a national agency, not a federal agency. 2. Most cities and all states have an OEM, which coordinates the disaster relief efforts at the state and local levels. 4. MMRS teams are local teams located in cities deemed to be possible terrorist targets. TEST-TAKING HINT: The question asks for a federal agency. The word "metro" means "local"; therefore, option "4" could be eliminated. All health-care providers should be aware of the role of The Joint Commission in the hospital, so the test taker could eliminate option "1."

47. The elderly client is diagnosed with chronic glomerulonephritis. Which lab value indicates the condition has gotten worse? 1. The BUN is 15 mg/dL. 2. The creatinine level is 1.2 mg/dL. 3. The glomerular filtration rate is 40 mL/minute. 4. The 24-hour creatinine clearance is 100 mL/minute.

3. Glomerular filtration rate (GFR) is approximately 120 mL per minute. If the GFR is decreased to 40 mL per minute, the kidneys are functioning at about one-third filtration capacity

60. The client diagnosed with Cushing's disease has undergone a unilateral adrenalectomy. Which discharge instructions should the nurse discuss with the client? 1. Instruct the client to take the glucocorticoid and mineralocorticoid medications as prescribed. 2. Teach the client regarding sexual functioning and androgen replacement therapy. 3. Explain the signs and symptoms of infection and when to call the health-care provider. 4. Demonstrate turn, cough, and deep-breathing exercises the client should perform every (2) hours.

3. Notifying the HCP if signs/symptoms of infection develop is an instruction given to all surgical clients on discharge.

The client had a mastectomy and lymph node dissection 3 years ago and has experienced post mastectomy pain since. Which intervention should the nurse implement? 1. Have the client see a pyschologist because the pain is not real 2. Tell the client that the pain is the cancer coming back 3. Refer the client to a physical therapist to prevent a frozen shoulder 4. Discuss changing the client to a more potent narcotic medication.

3. Refer the client to a physical therapist to prevent a frozen shoulder.

10. The client received 10 units of Humulin R, a fast-acting insulin, at 0700. At 1030 the unlicensed assistive personnel (UAP) tells the nurse the client has a headache and is really acting "funny." Which intervention should the nurse implement first? 1. Instruct the UAP to obtain the blood glucose level. 2. Have the client drink eight (8) ounces of orange juice. 3. Go to the client's room and assess the client for hypoglycemia. 4. Prepare to administer one (1) ampule 50% dextrose intravenously.

3. Regular insulin peaks in 2 to 4 hours. Therefore, the nurse should think about the possibility the client is having a hypoglycemic reaction and should assess the client. The nurse should not delegate nursing tasks to a UAP if the client is unstable.

30. The male client diagnosed with chronic pancreatitis calls and reports to the clinic nurse he has been having a lot of "gas," along with frothy and very foul-smelling stools. Which intervention should the nurse implement? 1. Explain this is common for chronic pancreatitis. 2. Ask the client to bring in a stool specimen to the clinic. 3. Arrange an appointment with the HCP for today. 4. Discuss the need to decrease fat in the diet so this won't happen.

3. Steatorrhea (fatty, frothy, foul-smelling stool) is caused by a decrease in pancreatic enzyme secretion and indicates impaired digestion and possibly an increase in the severity of the pancreatitis. The client should see the HCP.

7. The 78-year-old Catholic client is in end-stage congestive heart failure and has a DNR order. The client has AP 50, RR 10, and BP 80/50, and Cheyne-Stokes respirations. Which action should the nurse implement? 1. Bring the crash cart to the bedside. 2. Apply oxygen via nasal cannula. 3. Notify a priest for last rites. 4. Turn the bed to face the sunset.

3. The Catholic religion requires last rites be performed immediately before or after death

67. The client diagnosed with renal calculi is scheduled for lithotripsy. Which postprocedure nursing task would be most appropriate to delegate to the unlicensed nursing assistant (NA)? 1. Monitor the amount, color, and consistency of urine output. 2. Teach the client about care of the indwelling Foley catheter. 3. Assist the client to the car when being discharged home. 4. Take the client's post-procedural vital signs.

3. The NA could assist the client to the car once the discharge has been completed.

50. The nurse is discussing the HCP's recommendation for removal of life support with the client's family. Which information concerning brain death should the nurse teach the family? 1. Positive waves on the electroencephalogram (EEG) mean the brain is dead and any further treatment is futile. 2. When putting cold water in the ear, if the client reacts by pulling away, this demonstrates brain death. 3. Tests will be done to determine if any brain activity exists before the machines are turned off. 4. Although the blood flow studies don't indicate activity, the client can still come out of the coma.

3. The Uniform Determination of Brain Death Act states brain death is determined by accepted medical standards which indicate irreversible loss of all brain function. Cerebral blood flow studies, EEG, and oculovestibular and oculocephalic tests may be done.

22. The nurse is caring for a client diagnosed with rule-out nephrotic syndrome. Which intervention should be included in the plan of care? 1. Monitor the urine for bright-red bleeding. 2. Evaluate the calorie count of the 500-mg protein diet. 3. Assess the client's sacrum for dependent edema. 4. Monitor for a high serum albumin level.

3. The classic sign/symptom of nephrotic syndrome is dependent edema located on the client's sacrum and ankles.

The nurse is caring for clients on the medical floor. Which client should the nurse access first after the shift report? 1. The client with arterial blood goes of pH 7.36, HC03 26 2. The client with a T 99 F, P 101,RR 28, and BP 120/80 3. The client complaining of pain at a "10" on a scale of 1-10 but can't localize the pain 4. The client that is post-appendectomy with pain at a "3" on a scale of 1-10

3. The client complaining of pain at a "10" on a scale of 1-10 but can't localize the pain. This the typical fo patients with chronic pain. They cannot localize the pain and frequently describe pain as always being there, as disturbing rest, and as demoralizing. The client should be seen, and appropriate pain-control measures should be taken.

The nurse is assessing the client diagnosed with congestive heart failure. Which signs/symptoms would indicate that the medical treatment has been effective? 1. The client's peripheral pitting edema has gone from 3+ to 4+. 2. The client is able to take the radial pulse accurately. 3. The client is able to perform ADLs without dyspnea. 4. The client has minimal jugular vein distention.

3. The client is able to perform ADLs without dyspnea. 1. Pitting edema changing from 3+ to 4+ indicates a worsening of the CHF. 2. The client's ability to take the radial pulse would evaluate teaching, not medical treatment. 3. Being able to perform activities of daily living (ADLs) without shortness of breath (dyspnea) would indicate the client's condition is improving. The client's heart is a more effective pump and can oxygenate the body better without increasing fluid in the lungs. 4. Any jugular vein distention indicates that the right side of the heart is failing, which would not indicate effective medical treatment. TEST-TAKING HINT: When asked to determine whether treatment is effective, the test taker must know the signs and symptoms of the disease being treated. An improvement in the signs and symptoms indicates effective treatment.

3. In which client situation would the AD be consulted and used in decision making? 1. The client diagnosed with Guillain-Barré who is on a ventilator. 2. The client with a C6 spinal cord injury in the rehabilitation unit. 3. The client in end-stage renal disease who is in a comatose state. 4. The client diagnosed with cancer who has Down syndrome.

3. The client must have lost decisionmaking capacity as a result of a condition which is not reversible or must be in a condition specified under state law, such as a terminal, persistent vegetative state; an irreversible coma; or as specified in the AD.

The charge nurse is making shift assignments for the medical floor. Which client should be assigned to the most experienced registered nurse? 1. The client diagnosed with congestive heart failure who is being discharged in the morning. 2. The client who is having frequent incontinent liquid bowel movements and vomiting. 3. The client with an apical pulse rate of 116, a respiratory rate of 26, and a blood pressure of 94/62. 4. The client who is complaining of chest pain with inspiration and a nonproductive cough.

3. The client with an apical pulse rate of 116, a respiratory rate of 26, and a blood 1. This client is stable because discharge is scheduled for the following day. Therefore, this client does not need to be assigned to the most experienced registered nurse. 2. This client is more in need of custodial nursing care than care from the most experienced registered nurse. Therefore, the charge nurse could assign a less experienced nurse to this client. 3. This client is exhibiting signs/symptoms of shock, which makes this client the most unstable. An experienced nurse should care for this client. 4. These complaints usually indicate muscular or pleuritic chest pain; cardiac chest pain does not fluctuate with inspiration. This client does not require the care of an experienced nurse as much as does the client with signs of shock. TEST-TAKING HINT: When deciding on an answer for this type of question, the test taker should reason as to which client is stable and which has a potentially higher level of need.

32. Which statement best explains the scientific rationale for Kussmaul's respirations in the client diagnosed with diabetic ketoacidosis (DKA)? 1. The kidneys produce excess urine and the lungs try to compensate. 2. The respirations increase the amount of carbon dioxide in the bloodstream. 3. The lungs speed up to release carbon dioxide and increase the pH. 4. The shallow and slow respirations will increase the HCO3 in the serum

3. The lungs attempt to increase the blood pH level by blowing off the carbon dioxide (carbonic acid).

20. The emergency department nurse is caring for a client diagnosed with HHNS who has a blood glucose of 680 mg/dL. Which question should the nurse ask the client to determine the cause of this acute complication? 1. "When is the last time you took your insulin?" 2. "When did you have your last meal?" 3. "Have you had some type of infection lately?" 4. "How long have you had diabetes?"

3. The most common precipitating factor is infection. The manifestations may be slow to appear, with onset ranging from 24 hours to 2 weeks.

46. The client admitted to rule out pancreatic islet tumors complains of feeling weak, shaky, and sweaty. Which priority intervention should be implemented by the nurse? 1. Start an IV with D5W. 2. Notify the health-care provider. 3. Perform a bedside glucose check. 4. Give the client some orange juice.

3. These are symptoms of an insulin reaction (hypoglycemia). A bedside glucose check should be done. Pancreatic islet tumors can produce hyperinsulinemia or hypoglycemia.

Which cardiac enzyme would the nurse expect to elevate first in a client diagnosed with a myocardial infarction? 1. Creatine kinase (CK-MB). 2. Lactate dehydrogenase (LDH). 3. Troponin. 4. White blood cells (WBCs).

3. Troponin. 1. CPK-MB elevates in 12 to 24 hours. 2. LDH elevates in 24 to 36 hours. 3. Troponin is the enzyme that elevates within 1 to 2 hours. 4. WBCs elevate as a result of necrotic tissue, but this is not a cardiac enzyme. TEST-TAKING HINT: The test taker should be aware of the words "cardiac enzyme," which would eliminate option "4" as a possible answer. The word in the stem is "first." This question requires the test taker to have knowledge of laboratory values.

78. Which medication order should the nurse question in the client diagnosed with untreated hypothyroidism? 1. Thyroid hormones. 2. Oxygen . 3. Sedatives. 4. Laxatives.

3. Untreated hypothyroidism is characterized by an increased susceptibility to the effects of most hypnotic and sedative agents; therefore, the nurse should question this medication.

34. The client with an acute exacerbation of chronic pancreatitis has a nasogastric tube. Which interventions should the nurse implement? Select all that apply. 1. Monitor the client's bowel sounds. 2. Monitor the client's food intake. 3. Assess the client's intravenous site. 4. Provide oral and nasal care. 5. Monitor the client's blood glucose.

34. 1. The return of bowel sounds indicates the return of peristalsis, and the nasogastric suction is usually discontinued within 24 to 48 hours thereafter.3. The nurse should assess for signs of infection or infiltration. 4. Fasting and the N/G tube increase the client's risk for mucous membrane irritation and breakdown. 5. Blood glucose levels are monitored because clients with chronic pancreatitis can develop diabetes mellitus.

The male client is diagnosed with coronary artery disease (CAD) and is prescribed sublingual nitroglycerin. Which statement indicates the client needs more teaching? 1. "I should keep the tablets in the dark-colored bottle they came in." 2. "If the tablets do not burn under my tongue, they are not effective." 3. "I should keep the bottle with me in my pocket at all times." 4. "If my chest pain is not gone with one tablet, I will go to the ER."

4. "If my chest pain is not gone with one tablet, I will go to the ER." 1. If the tablets are not kept in a dark bottle, they will lose their potency. 2. The tablets should burn or sting when put under the tongue. 3. The client should keep the tablets with him in case of chest pain. 4. The client should take one tablet every five (5) minutes and, if no relief occurs after the third tablet, have someone drive him to the emergency department or call 911. TEST-TAKING HINT: This question is an "except" question, requiring the test taker to identify which statement indicates the client doesn't understand the teaching. Sometimes the test taker could restate the question and think which statement indicates the client understands the teaching.

38. The nurse caring for a client diagnosed with cancer of the pancreas writes the problem of "altered nutrition: less than body requirements." Which collaborative intervention should the nurse include in the plan of care? 1. Continuous feedings via PEG tube. 2. Have the family bring in foods from home. 3. Assess for food preferences. 4. Refer to the dietitian.

4. A collaborative intervention is to refer to the nutrition expert, the dietitian. TEST-TAKING HINT: The key word in the stem is "collaborative," which means another health-care discipline must be involved. Only options "1" and "4" involve other members of the health-care team. The test taker could eliminate distracter "1" by rereading the stem and realizing the stem did not include the client having a feeding tube.

12. Which client would the nurse exclude from being a potential organ/tissue donor? 1. The 60-year-old female client with an inoperable primary brain tumor. 2. The 45-year-old female client with a subarachnoid hemorrhage. 3. The 22-year-old male client who has been in a motor-vehicle accident. 4. The 36-year-male client recently released from prison.

4. A male client who has been in prison is at risk for being HIV positive, which excludes him from being an organ/ tissue donor.

The client who is terminally ill called the significant others to the room and said goodbye, then dismissed them and now lies quietly and refuses to eat. The nurse understands that the client is in what stage of the grieving process? 1. Denial 2. Anger 3. Bargaining 4. Acceptance

4. Acceptance. The client has accepted the eminent death and is withdrawing from their significant others.

47. Which intervention should the nurse implement first when caring for a client with a respiratory disorder? 1. Administer a respiratory treatment. 2. Check the client's radial pulses daily. 3. Monitor the client's vital signs daily. 4. Assess the client's capillary refill time.

4. Assessing the client's capillary refill time has the highest priority for the nurse because it indicates the oxygenation of the client.

The nurse enters the room of the client diagnosed with congestive heart failure. The client is lying in bed gasping for breath, is cool and clammy, and has buccal cyanosis. Which intervention would the nurse implement first? 1. Sponge the client's forehead. 2. Obtain a pulse oximetry reading. 3. Take the client's vital signs. 4. Assist the client to a sitting position.

4. Assist the client to a sitting position. 1. Sponging the client's forehead would be appropriate, but it is not the first intervention. 2. Obtaining a pulse oximeter reading would be appropriate, but it is not the first intervention. 3. Taking the vital signs would be appropriate, but it is not the first intervention. 4. The nurse must first put the client in a sitting position to decrease the workload of the heart by decreasing venous return and maximizing lung expansion. Then, the nurse could take vital signs and check the pulse oximeter and then sponge the client's forehead. TEST-TAKING HINT: In a question that asks the nurse to set priorities, all the answer options can be appropriate actions by the nurse for a given situation. The test taker should apply some guidelines or principles, such as Maslow's hierarchy, to determine what will give the client the most immediate assistance.

61. The female client presents to the emergency department with facial lacerations and contusions. The spouse will not leave the room during the assessment interview. Which intervention should be the nurse's first action? 1. Call the security guard to escort the spouse away. 2. Discuss the injuries while the spouse is in the room. 3. Tell the spouse the police will want to talk to him. 4. Escort the client to the bathroom for a urine specimen.

4. By escorting the client to a bathroom for any reason, the nurse can get the client to a safe area out of the hearing of the spouse. This is the most innocuous way to get the client alone. 1. This action could cause the spouse to become violent. The security personnel should not attempt to remove the spouse unless the client wishes them to do so. 2. Injuries resulting from spousal abuse should be discussed without the abuser present. 3. This may or may not be true. The client will have to prosecute, and many times the abused client will not do so. The client may feel responsible for the abuse, or may fear for her children's lives or for her own, or there may be a financial hold the spouse has over the client. Battered woman syndrome has many facets. TEST-TAKING HINT: When dealing with a violent person, the nurse should use discretion to avoid the spouse erupting into violence directed against the nurse, client, or others in the emergency department.

51. Which data would support the client's diagnosis of acute bacterial prostatitis? 1. Terminal dribbling. 2. Urinary frequency. 3. Stress incontinence. 4. Sudden fever and chills

4. Clients with acute bacterial prostatitis will frequently experience a sudden onset of fever and chills. Clients with chronic prostatitis have milder symptoms.

16. Which statement indicates to the nurse the client diagnosed with sleep apnea needs further teaching? 1. "If I lose weight I may not need treatment for sleep apnea." 2. "The CPAP machine holds my airway open with pressure." 3. "The CPAP will help me stay awake during the day while I am at work." 4. "It is all right to have a couple of beers because I have this CPAP machine."

4. Drinking alcohol before sleep sedates the client, causing the muscles to relax, which, in turn, causes an obstruction of the client's airway. Drinking alcohol should be avoided even if the client uses a CPAP machine.

63. The client with possible renal calculi is scheduled for a renal ultrasound. Which intervention should the nurse implement for this procedure? 1. Ask if the client is allergic to shell fish or iodine. 2. Keep the client NPO eight (8) hours prior to the ultrasound. 3. Ensure the client has a signed informed consent form. 4. Explain the test is noninvasive and there is no discomfort.

4. No special preparation is needed for this noninvasive, nonpainful test. A conductive gel is applied to the back or flank and then a transducer is applied that produces sound waves that produce a picture.

22. The nurse caring for a client diagnosed with ESRD writes a client problem of "noncompliance of dietary restrictions." Which intervention should be included in the plan of care? 1. Teach the client the proper diet to eat while undergoing dialysis. 2. Refer the client and significant other to the dietician. 3. Explain the importance of eating the proper foods. 4. Determine the reason for the client not adhering to the diet.

4. Noncompliance is a choice the client has a right to make, but the nurse should determine the reason for the noncompliance and then take appropriate actions based on the client's rationale. For example, if the client has financial difficulties, the nurse may suggest how the client can afford the proper foods along with medications, or the nurse may be able to refer the client to a social worker.

66. The client is diagnosed with an acute episode of ureteral calculi. Which client problem is priority when caring for this client? 1. Fluid volume loss. 2. Knowledge deficit. 3. Impaired urinary elimination. 4. Pain.

4. Pain is priority. The pain can be so severe that a sympathetic response may occur, causing nausea; vomiting; pallor; and cool, clammy skin

5. The client is admitted with a diagnosis of rule-out severe acute respiratory syndrome (SARS). Which information is most important for the nurse to ask related to this diagnosis? 1. Current prescription and over-the-counter medication use. 2. Dates of and any complications associated with recent immunizations. 3. Any problems with recent or past use of blood or blood products. 4. Recent travel to mainland China, Hong Kong, or Taiwan.

4. Recent travel to mainland China, Taiwan, and Hong Kong is a risk factor for contracting SARS.

48. A gang war has resulted in 12 young males being brought to the emergency department. Which action by the nurse is priority when a gang member points a gun at a rival gang member in the trauma room? 1. Attempt to talk to the person who has the gun. 2. Explain to the person the police are coming. 3. Stand between the client and the man with the gun. 4. Get out of the line of fire and protect self.

4. Self-protection is priority; the nurse is not required to be injured in the line of duty. 1. This puts the nurse in a dangerous position and might cause the death of the nurse. 2. This will escalate the situation. 3. This is a dangerous position for the nurse to put himself or herself in. TEST-TAKING HINT: Self-protection is a priority. There is no advantage to protecting others if the caregivers are also injured. The only option protecting the nurse is to get out of the line of fire.

20. The client diagnosed with diabetes mellitus type 2 wants to be an organ donor and asks the nurse, "Which organs can I donate?" Which statement is the nurse's best response? 1. "It is wonderful you want to be an organ donor. Let's discuss this." 2. "You can donate any organ in your body, except the pancreas." 3. "You have to donate your body to science to be an organ donor." 4. "You cannot donate any organs, but you can donate some tissues."

4. The client can donate corneas, skin, and some joints, but organ donation from clients with type 2 diabetes mellitus usually is not allowed.

52. The nurse is checking the vital signs graphic sheet at 0800 on a client diagnosed with pneumonia secondary to chronic obstructive pulmonary disease who is receiving prednisone, a glucocorticoid. Which graphic should the nurse expect to see?

4. The client diagnosed with COPD and on long-term steroids has a suppressed immune system. Temperatures are in the normal range because steroids mask the symptoms of infection by suppressing the immune system. The only common symptoms would be rusty colored sputum or a change in sputum color and confusion.

53. Which nursing diagnosis would be priority for the client who has undergone a TURP? 1. Potential for sexual dysfunction. 2. Potential for an altered body image. 3. Potential for chronic infection. 4. Potential for hemorrhage.

4. This is a potential life-threatening problem.

37. The client from a long-term care facility is admitted with a fever, hot flushed skin, and clumps of white sediment in the indwelling catheter. Which intervention should the nurse implement first? 1. Start an IV with a 20-gauge catheter. 2. Initiate antibiotic therapy IVPB. 3. Collect a urine specimen for culture. 4. Change the indwelling catheter

4. Unless the nurse can determine that the catheter has been inserted within a few days, the nurse should replace the catheter and then get a specimen. This will provide the most accurate specimen for analysis.

5. The elderly client recovering from a prostatectomy has been experiencing stress incontinence. Which independent nursing intervention should the nurse discuss with the client? 1. Establish a set voiding frequency of every two (2) hours while awake. 2. Encourage a family member to assist the client to the bathroom to void. 3. Apply a transurethral electrical stimulator to relieve symptoms of urinary urgency. 4. Discuss the use of a "bladder drill," including a timed voiding schedule.

4. Use of the bladder training drill is helpful in stress incontinence. The client is instructed to void at scheduled intervals. After consistently being dry, the interval is increased by 15 minutes until the client reaches an acceptable interval.

43. The nurse is caring for a female client who is anxious, has a respiratory rate of 40, and is complaining of her fingers tingling and her lips feeling numb. Which intervention should the nurse implement first? 1. Have the client take slow, deep breaths. 2. Instruct her to put her head between her legs. 3. Determine why she is feeling so anxious. 4. Administer Xanax, an antianxiety agent.

43. 1. The client is hyperventilating and blowing off too much CO2 , which is why her fingers are tingling and her mouth is numb; she needs to retain CO2 by taking slow deep breaths.

3

6. A nurse is at the lake when a person nearly drowns. The nurse determines the client is breathing spontaneously. Which data should the nurse assess next? 1. Possibility of drug use. 2. Spinal cord injury. 3. Level of confusion. 4. Amount of alcohol.

68. The nurse is planning the care of a client diagnosed with syndrome of inappropriate antidiuretic hormone (SIADH). Which interventions should be implemented? Select all that apply. 1. Restrict fluids per health-care provider order. 2. Assess level of consciousness every two (2) hours. 3. Provide an atmosphere of stimulation. 4. Monitor urine and serum osmolality. 5. Weigh the client every three (3) days.

68. 1. Fluids are restricted to 500 to 600 mL per 24 hours. 2. Orientation to person, place, and time should be assessed every two (2) hours or more often. 4. Urine and serum osmolality are monitored to determine fluid volume status.

The nurse and a UAP are caring for an elderly client diagnosed with emphysema. Which nursing tasks could be delegated to the UAP to improve gas exchange? Select all that apply. A. Keep the HOB elevated B. Encourage deep breathing exercises C. Record pulse oximeter reading D. Assess LOC E. Auscultate breathe sounds

A, B, C

The nurse caring for a client with sepsis writes the client diagnosis of "alteration in comfort R/T chills and fever." Which intervention should be included in the plan of care? 1. Ambulate the client in the hallway every shift. 2. Monitor urinalysis, creatinine level, and BUN level. 3. Apply sequential compression devices to the lower extremities. 4. Administer an antipyretic medication every four (4) hours PRN.

Antipyretic medication will help decrease the client's fever, which directly addresses the etiology of the client's nursing diagnosis.

The client has recently experienced a myocardial infarction. Which action by the nurse helps prevent cardiogenic shock? 1. Monitor the client's telemetry. 2. Turn the client every two (2) hours. 3. Administer oxygen via nasal cannula. 4. Place the client in the Trendelenburg position.

Promoting adequate oxygenation of the heart muscle and decreasing the cardiac workload can prevent cardiogenic shock.

To what area should the nurse place the stethoscope to best auscultate the apical pulse? A B C D 1. A 2. B 3. C 4. D

The best place to auscultate the apical pulse is over the mitral valve area, which is the fifth intercostal space, midclavicular line.

The nurse is caring for a client diagnosed with septic shock. Which assessment data warrant immediate intervention by the nurse? 1. Vital signs T 100.4˚F, P 104, R 26, and BP 102/60. 2. A white blood cell count of 18,000/mm3. 3. A urinary output of 90 mL in the last four (4) hours. 4. The client complains of being thirsty.

The client must have a urinary output of at least 30 mL/hr, so 90 mL in the last four (4) hours indicates impaired renal perfusion, which is a sign of worsening shock and warrants immediate intervention.

3. Which intervention should the emergency department nurse implement first for the client admitted for an acute asthma attack? 1. Administer glucocorticoids intravenously. 2. Administer oxygen 5 L per nasal cannula. 3. Establish and maintain a 20-gauge saline lock. 4. Assess breath sounds every 15 minutes.

oxygen

30. The telemetry monitor technician notifies the nurse of the morning telemetry readings. Which client should the nurse assess first? 1. The client in normal sinus rhythm with a peaked T wave. 2. The client diagnosed with atrial fibrillation with a rate of 100. 3. The client diagnosed with a myocardial infarction who has occasional PVC. 4. The client with a first-degree AV block and a rate of 92.

1. A client with a peaked wave could be experiencing hyperkalemia. Changes in potassium levels can initiate cardiac dysrhythmias and instability

The client is on the ventilator and has been declared brain dead. The spouse refuses to allow the ventilator to be discontinued. Which collaborative action by the nurse is most appropriate? 1. Discuss referral of the case to the ethics board. 2. Pull the plug when the spouse is not in the room. 3. Ask the HCP to discuss the futile situation with the spouse. 4. Inform the spouse that what is happening is cruel.

1. Discuss referral of the case to the ethics board. The nurse should discuss using the ethics committee with the HCP to assist the family making the decision to terminate life support. Many families feel that there may be racial or financial reason the HCP wants to discontinue life support.

The client is experiencing multifocal premature ventricular contractions. Which antidysrhythmic medication would the nurse expect the health-care provider to order for this client? 1. Lidocaine. 2. Atropine. 3. Digoxin. 4. Adenosine

1. Lidocaine. 1. Lidocaine suppresses ventricular ectopy and is the drug of choice for ventricular dysrhythmias. 2. Atropine decreases vagal stimulation and is the drug of choice for asystole. 3. Digoxin slows heart rate and increases cardiac contractility and is the drug of choice for atrial fibrillation. 4. Adenosine is the drug of choice for supraventricular tachycardia. TEST-TAKING HINT: This is a knowledgebased question, and the test taker must know the answer. The nurse must know what medications treat specific dysrhythmias.

46. Which entity mandates the registered nurse's behavior when practicing professional nursing? 1. The state's Nurse Practice Act. 2. Client's Bill of Rights. 3. The United States legislature. 4. American Nurses Association.

1. Nurse Practice Acts provide the laws which control the practice of nursing in each state. All states have Nurse Practice Acts.

2

1. The 38-year-old client was brought to the emergency department with CPR in progress and expired 15 minutes after arrival. Which intervention should the nurse implement for postmortem care? 1. Do not allow significant others to see the body. 2. Do not remove any tubes from the body. 3. Prepare the body for the funeral home. 4. Send the client's clothing to the hospital laundry.

3

10. The male client asks the nurse, "Should I designate my wife as durable power of attorney for health care?" Which statement would be the nurse's best response? 1. "Yes, she should be because she is your next of kin." 2. "Most people don't allow their spouse to do this." 3. "Will your wife be able to support your wishes?" 4. "Your children are probably the best ones for the job."

1,2,4

10. The occupational health nurse is called to the scene of a traumatic amputation of a finger. Which intervention should the nurse implement prior to sending the client to the ED? Select all that apply. 1. Rinse the amputated finger with sterile normal saline. 2. Place the amputated finger in a sealed and watertight plastic bag. 3. Place the amputated finger into iced saline solution. 4. Wrap the amputated finger in saline-moistened gauze dressings. 5. Replace the amputated finger on the hand and wrap with gauze.

1

11. The client has been declared brain dead and is an organ donor. The nurse is preparing the wife of the client to enter the room to say good-bye. Which information is most important for the nurse to discuss with the wife? 1. Inform the wife the client will still be on the ventilator. 2. Instruct the wife to only stay a few minutes at the bedside. 3. Tell the wife it is all right to talk to the client. 4. Allow another family member to go in with the wife.

3

11. The nurse is teaching the client home care instructions for a reimplanted finger after a traumatic amputation. Which information should the nurse include the teaching? 1. Perform range-of-motion exercises weekly. 2. Smoking may be resumed if it does not cause nausea. 3. Protect the finger and be careful not to reinjure the finger. 4. An elevated temperature is the only reason to call the HCP.

2

13. The intensive care nurse is caring for a deceased client who is an organ donor, and the organ donation team is in route to the hospital. Which statement would be an appropriate goal of treatment for the client? 1. The urinary output is 20 mL/hr via a Foley catheter. 2. The systolic blood pressure is greater than 90 mm Hg. 3. The pulse oximeter reading remains between 88% and 90%. 4. The telemetry shows the client in sinus tachycardia.

2

13. The nurse in the emergency department has admitted five (5) clients in the last two (2) hours with complaints of fever and gastrointestinal distress. Which question is most appropriate for the nurse to ask each client to determine if there is a bioterrorism threat? 1. "Do you work or live near any large power lines?" 2. "Where were you immediately before you got sick?" 3. "Can you write down everything you ate today?" 4. "What other health problems do you have?"

1

14. The ED nurse is caring for a client who had a severe allergic reaction to a bee sting. Which discharge instructions should the nurse discuss with the client? 1. Instruct the client to wear a medical identification bracelet. 2. Apply corticosteroid cream to the site to prevent anaphylaxis. 3. Administer epinephrine 1:10,000 intravenously every three (3) minutes. 4. Teach the client to avoid attracting insects by wearing bright colors.

4

18. The nurse is teaching a class on biological warfare. Which information should the nurse include in the presentation? 1. Contaminated water is the only source of transmission of biological agents. 2. Vaccines are available and being prepared to counteract biological agents. 3. Biological weapons are less of a threat than chemical agents. 4. Biological weapons are easily obtained and result in significant mortality.

Which statement by the client diagnosed with coronary artery disease indicates that the client understands the discharge teaching concerning diet? 1. "I will not eat more than six (6) eggs a week." 2. "I should bake or grill any meats I eat." 3. "I will drink eight (8) ounces of whole milk a day." 4. "I should not eat any type of pork products."

2. "I should bake or grill any meats I eat." 1. According to the American Heart Association, the client should not eat more than three (3) eggs a week, especially the egg yolk. 2. The American Heart Association recommends a low-fat, low-cholesterol diet for a client with coronary artery disease. The client should avoid any fried foods, especially meats, and bake, broil, or grill any meat. 3. The client should drink low-fat milk, not whole milk. 4. Pork products (bacon, sausage, ham) are high in sodium, which is prohibited in a low-salt diet, not a low-cholesterol, lowfat diet. TEST-TAKING HINT: The test taker must be knowledgeable of prescribed diets for specific disease processes. This is mainly memorizing facts. There is no Test- Taking Hint to help eliminate any of the options.

29. The client is diagnosed with bronchiolitis obliterans. Which data indicate the glucocorticoid therapy is effective? 1. The client has an elevation in the blood glucose. 2. The client has a decrease in sputum production. 3. The client has an increase in the temperature. 4. The client appears restless and is irritable.

2. A decrease in sputum production indicates that the client is improving and the medication is effective; long-term use of corticosteroids is indicated for a client with bronchiolitis obliterans.

26. Which client problem has priority for the client diagnosed with acute pancreatitis? 1. Risk for fluid volume deficit. 2. Alteration in comfort. 3. Imbalanced nutrition: less than body requirements. 4. Knowledge deficit.

2. Autodigestion of the pancreas results in severe epigastric pain, accompaniedby nausea, vomiting, abdominal tenderness, and muscle guarding.

11. The nurse at a freestanding health care clinic is caring for a 56-year-old male client who is homeless and is a type 2 diabetic controlled with insulin. Which action is an example of client advocacy? 1. Ask the client if he has somewhere he can go and live. 2. Arrange for someone to give him insulin at a local homeless shelter. 3. Notify Adult Protective Services about the client's situation. 4. Ask the HCP to take the client off insulin because he is homeless.

2. Client advocacy focuses support on the client's autonomy. Even if the nurse disagrees with his living on the street, it is the client's right. Arranging for

20. The nurse is orienting to a hospice organization. Which statement does not indicate a right of the terminal client? The right to: 1. Be treated with respect and dignity. 2. Have particulars of the death withheld. 3. Receive optimal and effective pain management. 4. Receive holistic and compassionate care.

2. The client has the right to discuss his or her feelings and direct his or her care. Withholding information would be lying to the client.

3

2. The primary nurse caring for the client who died is crying with the family at the bedside. Which action should the charge nurse implement? 1. Request the primary nurse to come out in the hall. 2. Refer the nurse to the employee assistance program. 3. Allow the nurse and family this time to grieve. 4. Ask the chaplain to relieve the nurse at the bedside.

27. The nurse is preparing to administer a.m. medications to clients. Which medication should the nurse question before administering? 1. Pancreatic enzymes to the client who has finished breakfast. 2. The pain medication, morphine, to the client who has a respiratory rate of 20. 3. The loop diuretic to the client who has a serum potassium level of 3.9 mEq/L. 4. The beta blocker to the client who has an apical pulse of 68 bpm.

27. 1. Pancreatic enzymes must be administered with meals to enhance the digestion of starches and fats in the gastrointestinal tract.

13. The client diagnosed with type 2 diabetes is admitted to the intensive care unit with hyperosmolar hyperglycemic nonketonic syndrome (HHNS) coma. Which assessment data should the nurse expect the client to exhibit? 1. Kussmaul's respirations. 2. Diarrhea and epigastric pain. 3. Dry mucous membranes. 4. Ketone breath odor.

3. Dry mucous membranes are a result of the hyperglycemia and occur with both HHNS and DKA.

12. The client diagnosed with ARF is experiencing hyperkalemia. Which medication should the nurse prepare to administer to help decrease the potassium level? 1. Erythropoietin. 2. Calcium gluconate. 3. Regular insulin. 4. Osmotic diuretic.

3. Regular insulin, along with glucose, will drive potassium into the cells, thereby lowering serum potassium levels temporarily.

3

3. The elderly client is brought to the ED complaining of cramps, headache, and weakness after working outside in the sun. The telemetry shows sinus tachycardia. Which intervention should the nurse implement? 1. Determine if the client is experiencing any thirst. 2. Administer D5W intravenously at 250 mL/hr. 3. Maintain a cool environment to promote rest. 4. Withhold the client's oral intake.

83. The nurse is preparing to administer the following medications. Which medication should the nurse question administering? 1. The thyroid hormone to the client who does not have a T3 , T4 level. 2. The regular insulin to the client with a blood glucose level of 210 mg/dL. 3. The loop diuretic to the client with a potassium level of 3.3 mEq/L. 4. The cardiac glycoside to the client who has a digoxin level of 1.4 mg/dL.

3. This potassium level is below normal, which is 3.5 to 5.5 mEq/L. Therefore, the nurse should question administering this medication because loop diuretics cause potassium loss in the urine. 4. The digoxin

35. The nurse is administering a pancreatic enzyme to the client diagnosed with chronic pancreatitis. Which statement best explains the rationale for administering this medication? 1. It is an exogenous source of protease, amylase, and lipase. 2. This enzyme increases the number of bowel movements. 3. This medication breaks down in the stomach to help with digestion. 4. Pancreatic enzymes help break down fat in the small intestine.

35. 1. Pancreatic enzymes enhance the digestion of starches (carbohydrates) in the gastrointestinal tract by supplying an exogenous (outside) source of the pancreatic enzymes protease, amylase, and lipase.

39. The health-care provider has ordered a continuous intravenous infusion of aminophylline. The client weighs 165 pounds. The infusion order is 0.3 mg/kg/hr. The bag is mixed with 500 mg of aminophylline in 250 mL of D5W. At which rate should the nurse set the pump? _______

39. 11 mL/hr. First, convert pounds to kilograms: 165 pounds ÷ 2.2 = 75 kg Then, determine how many milligrams of aminophylline per hour should be administered: 0.3 mg × 75 kg = 22.5 mg/hr Then, determine how much aminophylline is delivered per milliliter: 500 mg ÷ 250 mL = 2 mg/1 mL If 2 mg/1 mL is delivered, then to deliver the prescribed 22.5 mg/hr, the rate must be set at: 22.5 ÷ 2 = 11.25 mL/hr Less than 0.5 should be rounded down, 0.5 and above is rounded up.

Which nursing diagnosis would be priority for the client diagnosed with myocarditis? 1. Anxiety related to possible long-term complications. 2. High risk for injury related to antibiotic therapy. 3. Increased cardiac output related to valve regurgitation. 4. Activity intolerance related to impaired cardiac muscle function.

4. Activity intolerance related to impaired cardiac muscle function. 1. Anxiety is a psychosocial nursing diagnosis, which is not a priority over a physiological nursing diagnosis. 2. Antibiotic therapy does not result in injury to the client. 3. Myocarditis does not result in valve damage (endocarditis does), and there would be decreased, not increased, cardiac output. 4. Activity intolerance is priority for the client with myocarditis, an inflammation of the heart muscle. Nursing care is aimed at decreasing myocardial work and maintaining cardiac output. TEST-TAKING HINT: If the test taker has no idea which is the correct answer, then "myo," which refers to muscle, and "card," which refers to the heart, should lead the test taker to the only option which has both muscle and heart in it, option "4."

The telemetry nurse notes a peaked T wave for the client diagnosed with congestive heart failure. Which laboratory data should the nurse assess? 1. CK-MB. 2. Troponin. 3. BNP. 4. Potassium.

4. Potassium Hyperkalemia will cause a peaked T wave; therefore, the nurse should check these laboratory data

83. The client is two (2) days post-ureterosigmoidostomy for cancer of the bladder. Which assessment data warrant notification of the HCP by the nurse? 1. The client complains of pain at a "3," 30 minutes after being medicated. 2. The client complains that it hurts to cough and deep breathe. 3. The client ambulates to the end of the hall and back before lunch. 4. The client is lying in a fetal position and has a rigid abdomen.

4. The client is drawn up in a position that takes pressure off the abdomen; a rigid abdomen is an indicator of peritonitis, a medical emergency

60. Which tissue or organ can be repeatedly donated to clients needing a transplant? 1. Skin. 2. Bones. 3. Kidneys. 4. Bone marrow

4. The human body reproduces bone marrow daily. There is a bone marrow registry for participants willing to undergo the procedure to donate to clients when a match is found.

71. The client had surgery to remove a kidney stone. Which laboratory assessment data would warrant immediate intervention by the nurse? 1. A serum potassium level of 3.8 mEq/L. 2. A urinalysis that shows microscopic hematuria. 3. A creatinine level of 0.8 mg/100 mL. 4. A white blood cell count of 14,000 mm/dL.

4. This white blood cell count is elevated; normal is 5,000-10,000 mm.

1

5. The hospice nurse is making the final visit to the wife whose husband died a little more than a year ago. The nurse realizes the husband's clothes are still in the closet and chest of drawers. Which action should the nurse implement first? 1. Discuss what the wife is going to do with the clothes. 2. Refer the wife to a grief recovery support group. 3. Do not take any action because this is normal grieving. 4. Remove the clothes from the house and dispose of them.

51. The nurse is planning the care of a client diagnosed with Addison's disease. Which intervention should be included? 1. Administer steroid medications. 2. Place the client on fluid restriction. 3. Provide frequent stimulation. 4. Consult physical therapy for gait training.

51. 1. Clients diagnosed with Addison's disease have adrenal gland hypofunction. The hormones normally produced by the gland must be replaced. Steroids and androgens are produced by the adrenal gland.

58. The client diagnosed with Cushing's disease has developed 1+ peripheral edema. The client has received intravenous fluids at 100 mL/hr via IV pump for the past 79 hours. The client received IVPB medication in 50 mL of fluid every 6 hours for 15 doses. How many mL of fluid did the client receive? ________

58. The client has received 8,650 mL of intravenous fluid. TEST-TAKING HINT: This is a basic addition problem. If the test taker has difficulty with this problem, then a math review course would be in order.

59. The nurse manager of a medical-surgical unit is asked to determine if the unit should adopt a new care delivery system. Which behavior is an example of an autocratic style of leadership? 1. Call a meeting and educate the staff on the new delivery system being used. 2. Organize a committee to investigate the various types of delivery systems. 3. Wait until another unit has implemented the new system and see if it works out. 4. Discuss with the nursing staff if a new delivery system should be adopted.

59. 1. An autocratic style is one in which the person in charge makes the decision without consulting anyone else. 2. This behavior is an example of a democratic

67. The health-care provider has ordered 40 g/24 hr of intranasal vasopressin for a client diagnosed with diabetes insipidus. Each metered spray delivers 10 g. The client takes the medication every 12 hours. How many sprays are delivered at each dosing time?

67. Two (2) sprays per dose. 40 g of medication every 24 hours is to be given in doses administered every 12 hours. First, determine number of doses needed: 24 ÷ 12 = 2 doses Then, determine the amount of medication to be given in each of those two (2) doses: 40 ÷ 2 = 20 g of medication per dose Finally, determine how many sprays are needed to deliver the 20 mg when each spray delivers 10 g: 20 ÷ 10 = 2 sprays

3

7. The 78-year-old Catholic client is in end-stage congestive heart failure and has a DNR order. The client has AP 50, RR 10, and BP 80/50, and Cheyne-Stokes respirations. Which action should the nurse implement? 1. Bring the crash cart to the bedside. 2. Apply oxygen via nasal cannula. 3. Notify a priest for last rites. 4. Turn the bed to face the sunset.

71. The nurse is admitting a client diagnosed with syndrome of inappropriate antidiuretic hormone (SIADH). Which clinical manifestations should be reported to the health-care provider? 1. Serum sodium of 112 mEq/L and a headache. 2. Serum potassium of 5.0 mEq/L and a heightened awareness. 3. Serum calcium of 10 mg/dL and tented tissue turgor. 4. Serum magnesium of 1.2 mg/dL and large urinary output.

71. 1. A serum sodium level of 112 mEq/L is dangerously low, and the client is at risk for seizures. A headache is a symptom of a low sodium level.

73. The client is diagnosed with hypothyroidism. Which signs/symptoms should the nurse expect the client to exhibit? 1. Complaints of extreme fatigue and hair loss. 2. Exophthalmos and complaints of nervousness. 3. Complaints of profuse sweating and flushed skin. 4. Tetany and complaints of stiffness of the hands.

73. 1. A decrease in thyroid hormone causes decreased metabolism, which leads to fatigue and hair loss. 2. These are signs of hyperthyroidism.

74. The nurse identifies the client problem "risk for imbalanced body temperature" for the client diagnosed with hypothyroidism. Which intervention should be included in the plan of care? 1. Discourage the use of an electric blanket. 2. Assess the client's temperature every two (2) hours. 3. Keep the room temperature cool. 4. Space activities to promote rest.

74. 1. External heat sources (heating pads, electric or warming blankets) should be discouraged because they increase the risk of peripheral vasodilation and vascular collapse.

8. Which client should the nurse not assign to a UAP working on a surgical floor? 1. The client with a suprapubic catheter inserted yesterday. 2. The client who has had an indwelling catheter for the past week. 3. The client who is on a bladder-training regimen. 4. The client who had a catheter removed this morning and is being discharged.

8. 1. This client requires the most skill and knowledge because this client has the greatest potential for an infection; therefore, the client should not be assigned to a UAP.

Pain disorder and depression have been diagnosed for a patient. The patient reports chronic low back pain and states, "None of these doctors has done anything to help." Which patient statement concerns you the most? A. I twisted my back last night, and now the pain is a lot worse B. I'm so sick of this pain. I think I am going to find a way to end it C. Occasionally, I buy painkillers from a guy in my neighborhood D. I am going to sue you and the doctor, you aren't doing anything for me

B. I'm so sick of this pain. I think I'm going to find a way to end it

The client is diagnosed with neurogenic shock. Which signs/symptoms should the nurse assess in this client? 1. Cool moist skin. 2. Bradycardia. 3. Wheezing. 4. Decreased bowel sounds.

The client will have bradycardia instead of tachycardia, which is seen in other forms of shock.

The client is being discharged from the hospital for intractable pain secondary to cancer and is prescribed morphine, a narcotic. Which statement indicates the client understands the discharge instruction? 1. "I will be sure to have my prescription filled before any holiday. 2. There should not be a problem having the prescription filled anytime" 3. "If I run out of medications, I can call the HCP to phone in a prescription" 4. "There are no side effects to morphine that I should be concerned about"

1. "I will be sure to have my prescription filled before any holiday. Narcotic medications require handwritten prescription forms that must be filled within a limited timeframe from the timothy prescriptions written. Many local pharmacies will not have the medication available or may not have the quantities needed. The client should anticipate the needs prior to any time when the HCP may not be available or the pharmacy may be closed.

81. The client with a continent urinary diversion is being discharged. Which discharge instructions should the nurse include in the teaching? 1. Have the client demonstrate catheterizing the stoma. 2. Instruct the client on how to pouch the stoma. 3. Explain the use of a bedside drainage bag at night. 4. Tell the client to call the HCP if the temperature is 99F or less.

1. A continent urinary diversion is a surgical procedure in which a reservoir is created that will hold urine until the client can selfcatheterize the stoma. The nurse should observe the client's technique before discharge.

52. The client has received a kidney transplant. Which assessment would warrant immediate intervention by the nurse? 1. Fever and decreased urine output. 2. Decreased creatinine and BUN levels. 3. Decreased serum potassium and calcium. 4. Bradycardia and hypotension.

1. Oliguria, fever, increasing edema, hypertension, and weight gain are signs of organ rejection.

The nurse is aware the Patient Self-Determination Act of 1991 requires the health-care facility to implement which action? 1. Make available an AD on admission to the facility. 2. Assist the client with legally completing a will. 3. Provide ethically and morally competent care to the client. 4. Discuss the importance of understanding consent forms.

1. The Patient Self-Determination Act of 1991 requires health-care facilities which receive Medicare or Medicaid funding to make ADs available to clients on admission into the facility

2

14. The nurse is teaching a class on ethical principles in nursing. Which statement supports the definition of beneficence? 1. The duty to prevent or avoid doing harm. 2. The duty to actively do good for clients. 3. The duty to be faithful to commitments. 4. The duty to tell the truth to the clients.

The client diagnosed with hypovolemic shock has a BP of 100/60. Fifteen minutes later the BP is 88/64. How much narrowing of the client's pulse pressure has occurred between the two readings? ______

16 mm Hg pulse pressure. The pulse pressure is the systolic BP minus the diastolic BP. 100 60 = 40 mm Hg pulse pressure in first BP reading 88 64 = 24 mm Hg pulse pressure in second reading 40 24 = 16 mm Hg pulse pressure narrowing. A narrowing or decreased pulse pressure is an earlier indicator of shock than a decrease in systolic blood pressure.

16. Which intervention should the nurse include when assessing the client for urinary retention? Select all that apply. 1. Inquire if the client has the sensation of fullness. 2. Percuss the suprapubic region for a dull sound. 3. Scan the bladder with the ultrasound scanner. 4. Palpate from the umbilicus to the suprapubic area. 5. Auscultate the two (2) lower abdominal quadrants.

16. 1. The nurse needs to assess the client's sensation of needing to void or feeling of fullness. 2. A dull sound heard when percussing the bladder indicates it is filled with urine. 3. A portable bladder scan is used to assess for the presence of urine, rather than using a straight catheter. 4. A distended bladder can be palpated.

4

16. The nurse is teaching a class on bioterrorism. Which statement is the scientific rationale for designating a specific area for decontamination? 1. Showers and privacy can be provided to the client in this area. 2. This area isolates the clients who have been exposed to the agent. 3. It provides a centralized area for stocking the needed supplies. 4. It prevents secondary contamination to the health-care providers.

1

16. The nurse is teaching a class on chronic pain to new graduates. Which information is most important for the nurse to discuss? 1. The nurse must believe the client's report of pain. 2. Clients in chronic pain may not show objective signs. 3. Alternate pain-control therapies are used for chronic pain. 4. Referral to a pain clinic may be necessary.

77. The client is admitted to the intensive care department diagnosed with myxedema coma. Which assessment data warrant immediate intervention by the nurse? 1. Serum blood glucose level of 74 mg/dL. 2. Pulse oximeter reading of 90%. 3. Telemetry reading showing sinus bradycardia. 4. The client is lethargic and sleeps all the time.

2. A pulse oximeter reading of less than 93% is significant. A 90% pulseoximeter reading indicates a PaO2 of approximately 60 on an arterial blood gas test; this is severe hypoxemia and requires immediate intervention.

22. The client has been in a persistent vegetative state for several years. The family, who have decided to withhold tube feedings because there is no hope of recovery, asks the nurse, "Will the death be painful?" Which intervention should the nurse implement? 1. Tell the family the death will be painful but the HCP can order medications. 2. Inform the family dehydration provides a type of natural euphoria. 3. Relate other cases where the clients have died in excruciating pain. 4. Ask the family why they are concerned because they want the client to die anyway.

2. Death from dehydration occurs when the client is unable to take in fluids. A natural euphoria occurs with dehydration. This is the body's way of allowing comfort at the time of death.

45. During a disaster, a local news reporter comes to the emergency department requesting information about the victims. Which action is most appropriate for the nurse to implement? 1. Have security escort the reporter off the premises. 2. Direct the reporter to the disaster command post. 3. Tell the reporter this is a violation of HIPAA. 4. Request the reporter to stay out of the way.

2. Emergency operations plans will have a designated disaster plan coordinator. All public information should be routed through this person. 1. The media have an obligation to report the news and can play a significant positive role in communication, but communication should come from only one source— the disaster command center. 3. Client confidentiality must be maintained, but the best action is for the nurse to help the reporter get to the appropriate area for information. 4. This allows the reporter to stay in the emergency room, which is inappropriate. TEST-TAKING HINT: The nurse should address the situation with the reporter and provide an access for the information. Options "1," "3," and "4" do not help the reporter get accurate information.

The nurse is discussing placing a patient with COPD in hospice care. Which prognosis must be determined to place the client in hospice care 1. The client is doing well but could benefit from the added care by hospice 2. The client has a life expectancy of 6 months or less 3. The client will live for about 1 to two years 4. The client has about 8 weeks to live and needs pain control

2. The client has a life expectancy of 6 months or less. The HCP must think that without life prolonging treatment, the client has a life expectancy of 6 months or less. The client may continue receiving hospice care if the client lives longer.

27. The client diagnosed with diabetes insipidus weighed 180 pounds when the daily weight was taken yesterday. This morning's weight is 175.6 pounds. One liter of fluid weighs approximately 2.2 pounds. How much fluid has the client lost?_______

2000 mL

46. The nurse is preparing a plan of care for the client diagnosed with acute glomerulonephritis. Which would be a long-term goal? 1. The client will have a blood pressure within normal limits. 2. The client will show no protein in the urine. 3. The client will maintain renal function. 4. The client will have clear lung sounds.

3. A long-term complication of glomerulonephritis is that it can become chronic if unresponsive to treatment and this can lead to end-stage renal disease. Maintaining renal function would be an appropriate long-term goal.

6. The nurse is assessing the feet of a client with long-term type 2 diabetes. Which assessment data warrant immediate intervention by the nurse? 1. The client has crumbling toenails. 2. The client has athlete's foot. 3. The client has a necrotic big toe. 4. The client has thickened toenails.

3. A necrotic big toe indicates "dead" tissue. The client does not feel pain, does not realize the injury, and does not seek treatment. Increased blood glucose levels decrease the oxygen supply needed to heal the wound and increase the risk for developing an infection.

33. The nurse is discussing the results of a tuberculosis skin test. Which explanation should the nurse provide the client? 1. A red area is a positive reading that means the client has tuberculosis. 2. The skin test is the only procedure needed to diagnose tuberculosis. 3. A positive reading means exposure to the tuberculosis bacilli. 4. Do not get another skin test for one (1) year if the skin test is positive.

3. A positive reading indicates the client has been exposed to the bacilli.

7. Which arterial blood gas (ABG) results support the diagnosis of acute respiratory distress syndrome (ARDS) after the client has received O2 at 10 LPM? 1. pH 7.38, PaO2 94, PaCO2 44, HCO3 24. 2. pH 7.46, PaO2 82, PaCO2 34, HCO3 22. 3. pH 7.48, PaO2 59, PaCO2 30, HCO3 26. 4. pH 7.33, PaO2 94, PaCO2 44, HCO3 20.

3. ABGs initially show hypoxemia with a PaO2 of less than 60 mm Hg and respiratory alkalosis resulting from tachypnea in a client with ARDS.

15. The male client in ESRD has received the initial dose of erythropoietin, a biologic response modifier, 1 week ago. Which complaint by the client would indicate the need to notify the health-care provider? 1. The client complains of flulike symptoms. 2. The client complains of being tired all the time. 3. The client reports an elevation in his blood pressure. 4. The client reports discomfort in his legs and back.

3. After the initial administration of erythropoietin, a client's antihypertensive medications may need to be adjusted. Therefore, this complaint requires notification of the HCP. Erythropoietin therapy is contraindicated in clients with hypertension that cannot be controlled.

36. The client has been vomiting and has had numerous episodes of diarrhea. Which laboratory test should the nurse monitor? 1. Serum calcium. 2. Serum phosphorus. 3. Serum potassium. 4. Serum sodium.

3. Clients lose potassium from the GI tract or through the use of diuretic medications. Potassium imbalances can lead to cardiac arrhythmias.

15. Which action by the unlicensed assistive personnel (UAP) would warrant immediate intervention by the nurse? 1. The UAP is holding the phone to the ear of a client who is a quadriplegic. 2. The UAP refuses to discuss the client's condition with the visitor in the room. 3. The UAP put a vest restraint on an elderly client found wandering in the hall. 4. The UAP is assisting the client with arthritis to open up personal mail.

3. Restraints are not allowed unless there is a health-care provider's order with documentation by the nurse of the client being a danger to himself or others. The UAP's putting the client in restraints warrants immediate intervention because it is battery

44. The client diagnosed with pneumonia has arterial blood gases of pH 7.33, PaO2 94, PaCO2 47, HCO3 25. Which intervention should the nurse implement? 1. Administer sodium bicarbonate. 2. Administer oxygen via nasal cannula. 3. Have the client cough and deep breathe. 4. Instruct the client to breathe into a paper bag.

3. The client is retaining CO2 , which causes respiratory acidosis, and the nurse should help the client remove the CO2 by instructing the client to cough and deep breathe.

27. The clinic nurse is reviewing information submitted by the UAP which states the presence of pediculosis pubis. Which area of the client's body should the nurse assess?

3. The pubic lice are found in the pubic area and are commonly transmitted during sexual intercourse.

5. The nurse is discussing the importance of exercising with a client diagnosed with type 2 diabetes whose diabetes is well controlled with diet and exercise. Which information should the nurse include in the teaching about diabetes? 1. Eat a simple carbohydrate snack before exercising. 2. Carry peanut butter crackers when exercising. 3. Encourage the client to walk 20 minutes three (3) times a week. 4. Perform warmup and cool-down exercises.

4. All clients who exercise should perform warmup and cool-down exercises to help prevent muscle strain and injury. TEST-TAKING HINT: Options "1" and "2" apply directly to clients diagnosed with diabetes and options "3" and "4" do not directly address clients diagnosed with diabetes. The reader could narrow the choices by either eliminating or including the two similar options.

82. The nurse is providing an in-service on thyroid disorders. One of the attendees asks the nurse, "Why don't the people in the United States get goiters as often?" Which statement by the nurse is the best response? 1. "It is because of the screening techniques used in the United States." 2. "It is a genetic predisposition rare in North Americans." 3. "The medications available in the United States decrease goiters." 4. "Iodized salt helps prevent the development of goiters in the United States."

4. Almost all of the iodine entering the body is retained in the thyroid gland. A deficiency in iodine will cause the thyroid gland to work hard and enlarge, which is called a goiter. Goiters are commonly seen in geographical regions having an iodine deficiency. Most table salt in the United States has iodine added. TEST-TAKING HINT: The nurse must know about disease processes. There is no Test-Taking Hint to help with knowledge.

73. The nurse is working on a renal surgery unit. After the afternoon report, which client should the nurse assess first? 1. The male client who just returned from a CT scan and states that he left his glasses in the x-ray department. 2. The client who is one (1) day post-op and has a moderate amount of serous drainage on the dressing. 3. The client who is scheduled for surgery in the morning and wants an explanation of the operative procedure before signing the permit. 4. The client who had ileal conduit surgery this morning and has not had any drainage in the drainage bag.

4. An ileal conduit is a procedure that diverts urine from the bladder and provides an alternate cutaneous pathway for urine to exit the body. Urinary output should always be at least 30 mL per hour. This client should be assessed to make sure that the stents placed in the ureters have not become dislodged or to ensure that edema of the ureters is not occurring.

Which laboratory data confirm the diagnosis of congestive heart failure? 1. Chest x-ray (CXR). 2. Liver function tests. 3. Blood urea nitrogen (BUN). 4. Beta-type natriuretic peptide (BNP).

4. BNP BNP is a hormone released by the heart muscle in response to changes in blood volume and is used to diagnose and grade heart failure.

43. The female client in an outpatient clinic is being sent home with a diagnosis of urinary tract infection. Which instruction should the nurse teach to prevent a recurrence of a UTI? 1. Clean the perineum from back to front after a bowel movement. 2. Take warm tub baths instead of hot showers daily. 3. Void immediately preceding sexual intercourse. 4. Avoid coffee, tea, colas, and alcoholic beverages.

4. Coffee, tea, cola, and alcoholic beverages are urinary tract irritants

78. The nurse and a licensed practical nurse (LPN) are caring for a group of clients. Which intervention should be assigned to the LPN? 1. Assessment of the client who has had a Kock pouch procedure. 2. Monitoring of the post-op client with a WBC of 22,000 mm/dL. 3. Administration of the prescribed antineoplastic medications. 4. Care for the client going for a MRI of the kidneys.

4. It is in the scope of practice for the LPN to care for this client.

4

12. Which client would the nurse exclude from being a potential organ/tissue donor? 1. The 60-year-old female client with an inoperable primary brain tumor. 2. The 45-year-old female client with a subarachnoid hemorrhage. 3. The 22-year-old male client who has been in a motor-vehicle accident. 4. The 36-year-male client recently released from prison.

1

23. Which action by the primary nurse would require the unit manager to intervene? 1. The nurse uses a correction fluid to correct a charting mistake. 2. The nurse is shredding the worksheet at the end of the shift. 3. The nurse circles an omitted medication time on the MAR. 4. The nurse documents narcotic wastage with another nurse.

24. The client who is terminally ill called the significant others to the room and said good-bye, then dismissed them and now lies quietly and refuses to eat. The nurse understands the client is in what stage of the grieving process? 1. Denial. 2. Anger. 3. Bargaining. 4. Acceptance.

4. The client has accepted the imminent death and is withdrawing from the significant others.

The spouse of a client dying from lung cancer states, "I don't understand this death rattle. She has not had anything to drink in days. Where is this fluid coming from? Which is the hospice care nurse's best reponse? 1. "The body produces about 2 teaspoons of fluid every minute on it's own" 2. "Are you sure that someone is not putting ice chips in her mouth." 3. "There is no reason for this, but it does happen from time to time." 4. "I can administer a patch to her skin to dry up the secretions if you wish"

1. "The body produces about 2 teaspoons of fluid every minute on it's own." The respiratory tract cells produce liquid as a defense mechanism against bacteria and other invaders. About 9 mL a minute are produced. The "death rattle" can be disturbing to family members, and the nurse should intervene but not with suctioning, which will increase secretions and the need to suction more.

The client diagnosed with a myocardial infarction asks the nurse, "Why do I have to rest and take it easy? My chest doesn't hurt anymore." Which statement would be the nurse's best response? 1. "Your heart is damaged and needs about four (4) to six (6) weeks to heal." 2. "There is necrotic myocardial tissue that puts you at risk for dysrhythmias." 3. "Your doctor has ordered bedrest. Therefore, you must stay in the bed." 4. "Just because your chest doesn't hurt anymore doesn't mean you are out of danger."

1. "Your heart is damaged and needs about four (4) to six (6) weeks to heal." 1. The heart tissue is dead, stress or activity may cause heart failure, and it does take about six (6) weeks for scar tissue to form. 2. The nurse should talk to the client in layperson's terms, not medical terms. Medical terminology is a foreign language to most clients. 3. This is not answering the client's question. The nurse should take any opportunity to teach the client. 4. This is a condescending response, and telling the client that he or she is not out of danger is not an appropriate response. TEST-TAKING HINT: When attempting to answer a client's question, the nurse should provide factual information in simple, understandable terms. The test taker should select the answer option that provides this type of information.

44. The pregnant client is admitted to a medical unit for the treatment of acute pyelonephritis. Which scientific rationale supports the client being hospitalized for this condition? 1. The client must be treated aggressively to prevent maternal/fetal complications. 2. The nurse can force the client to drink fluids and avoid nausea and vomiting. 3. The client will be dehydrated and there won't be sufficient blood flow to the baby. 4. Pregnant clients historically are afraid to take the antibiotics as ordered.

1. A pregnant client diagnosed with a UTI will be admitted for aggressive IV antibiotic therapy. After symptoms subside the client will be sent home to complete the course of treatment with oral medications. The mother and child need aggressive treatment to prevent systemic bacteremia.

58. The client asks, "What does an elevated PSA test mean?" On which scientific rationale would the nurse base the response? 1. An elevated PSA can result from several different causes. 2. An elevated PSA can be only from prostate cancer. 3. An elevated PSA can be diagnostic for testicular cancer. 4. An elevated PSA is the only test used to diagnose benign prostatic hypertrophy.

1. An elevated PSA can be from urinary retention, BPH, prostate cancer, or prostate infarct.

68. Which statement indicates that the client diagnosed with calcium phosphate renal calculi understands the discharge teaching for ways to prevent future calculi formation? 1. "I should increase my fluid intake, especially in warm weather." 2. "I should eat foods that contain cocoa and chocolate." 3. "I will walk about a mile every week and not exercise often." 4. "I should take one vitamin a day that has extra calcium."

1. An increased fluid intake that ensures 2-3 L of urine a day prevents the stone-forming salts from becoming concentrated enough to precipitate

The client is admitted to the telemetry unit diagnosed with acute exacerbation of congestive heart failure (CHF). Which signs/symptoms would the nurse expect to find when assessing this client? 1. Apical pulse rate of 110 and 4+ pitting edema of feet. 2. Thick white sputum and crackles that clear with cough. 3. The client sleeping with no pillow and eupnea. 4. Radial pulse rate of 90 and capillary refill time <3 seconds.

1. Apical pulse rate of 110 and 4+ pitting edema of feet. 1. The client with CHF would exhibit tachycardia (apical pulse rate of 110), dependent edema, fatigue, third heart sounds, lung congestion, and change in mental status. 2. The client with CHF usually has pink frothy sputum and crackles that do not clear with coughing. 3. The client with CHF would report sleeping on at least two pillows, if not sleeping in an upright position, and labored breathing, not eupnea, which means normal breathing. 4. In a client diagnosed with heart failure, the apical pulse, not the radial pulse, is the best place to assess the cardiac status. TEST-TAKING HINT: In option "3," the word "no" is an absolute term, and usually absolutes, such as "no," "never," "always," and "only," are incorrect because there is no room for any other possible answer. If the test taker is looking for abnormal data, then the test taker should exclude the options that have normal values in them, such as eupnea, pulse rate of 90, and capillary refill time (CRT) <3 seconds.

Which meal would indicate the client understands the discharge teaching concerning the recommended diet for coronary artery disease? 1. Baked fish, steamed broccoli, and garden salad. 2. Enchilada dinner with fried rice and refried beans. 3. Tuna salad sandwich on white bread and whole milk. 4. Fried chicken, mashed potatoes, and gravy.

1. Baked fish, steamed broccoli and garden salad. The recommended diet for CAD is low fat, low cholesterol, and high fiber. The diet described is a diet that is low in fat and cholesterol.

The client diagnosed with pericarditis is being discharged home. Which intervention should the nurse include in the discharge teaching? 1. Be sure to allow for uninterrupted rest and sleep. 2. Refer client to outpatient occupational therapy. 3. Maintain oxygen via nasal cannula at two (2) L/min. 4. Discuss upcoming valve replacement surgery.

1. Be sure to allow for uninterrupted rest and sleep. 1. Uninterrupted rest and sleep help decrease the workload of the heart and help ensure the restoration of physical and emotional health. 2. Occupational therapy addresses activities of daily living. The client should be referred to physical therapy to develop a realistic and progressive plan of activity. 3. The client with pericarditis is not usually prescribed oxygen, and 2 L/min is a low dose of oxygen that is prescribed for a client with chronic obstructive pulmonary disease (COPD). 4. Endocarditis, not pericarditis, may lead to surgery for valve replacement. TEST-TAKING HINT: A concept that the test taker must remember with any client being discharged from the hospital should be to alternate rest with activity to avoid problems associated with immobility. If the test taker does not know the answer to a question, using basic concepts is the best option.

2. The client is diagnosed with ARF. Which laboratory values are most significant for diagnosing ARF? 1. BUN and creatinine. 2. WBC and hemoglobin. 3. Potassium and sodium. 4. Bilirubin and ammonia level.

1. Blood urea nitrogen (BUN) levels reflect the balance between the production and excretion of urea from the kidneys. Creatinine is a byproduct of the metabolism of the muscles and is excreted by the kidneys. Creatinine is the ideal substance for determining renal clearance because it is relatively constant in the body and is the laboratory value most significant in diagnosing renal failure.

44. Which situation requires the emergency department manager to schedule and conduct a Critical Incident Stress Management (CISM)? 1. Caring for a two (2)-year-old child who died from severe physical abuse. 2. Performing CPR on a middle-aged male executive who died. 3. Responding to a 22-victim bus accident with no apparent fatalities. 4. Being required to work 16 hours without taking a break.

1. CISM is an approach to preventing and treating the emotional trauma affecting emergency responders as a consequence of their job. Performing CPR and treating a young child affects the emergency personnel psychologically, and the death increases the traumatic experience. 2. Caring for this type of client is an expected part of the job. If the nurse finds this traumatic enough to require a CISM, then the nurse should probably leave the emergency department. 3. This requires an intense time for triaging and caring for the victims, but without fatalities this should not be as traumatic for the staff. 4. This is a dangerous practice because medication errors and other mistakes may occur as a result of fatigue, but this is not a traumatic situation. TEST-TAKING HINT: The test taker should examine the words "critical," "incident," and "stress." Each option should be examined to determine which is the most traumatic. Needless deaths of innocent children are psychologically traumatic.

70. The client who was abused as a child is diagnosed with post-traumatic stress disorder (PTSD). Which intervention should the nurse implement when the client is resting? 1. Call the client's name to awaken him or her, but don't touch the client. 2. Touch the client gently to let him or her know you are in the room. 3. Enter the room as quietly as possible to not disturb the client. 4. Do not allow the client to be awakened at all when sleeping.

1. Clients diagnosed with PTSD are easily startled and can react violently if awakened from sleep by being touched. 2. Touching the client can cause the client to become afraid, to believe himself or herself to be under attack, and to react violently. The nurse should not touch a sleeping client diagnosed with PTSD. 3. If the client awakes with the nurse in the room, the client could become fearful and react to the fear. 4. There may be times when the nurse must awaken the client to determine if the client is physically stable. TEST-TAKING HINT: Option "4" can be eliminated because of the absolute statement "at all." Options "2" and "3" can be eliminated if the test taker thinks of how it feels to be startled when perceiving another person around them when the test taker was not aware of the other person's presence.

The nurse is transcribing the doctor's orders for a client with congestive heart failure. The order reads 2.5 mg of Lanoxin daily. Which action should the nurse implement? 1. Discuss the order with the health-care provider. 2. Take the client's apical pulse rate before administering. 3. Check the client's potassium level before giving the medication. 4. Determine if a digoxin level has been drawn.

1. Discuss the order with the health-care provider. This dosage is 10 times the normal dose for a client with CHF. This dose is potentially lethal.

Which client teaching should the nurse implement for the client diagnosed with coronary artery disease? Select all that apply. 1. Encourage a low-fat, low-cholesterol diet. 2. Instruct client to walk 30 minutes a day. 3. Decrease the salt intake to two (2) g a day. 4. Refer to counselor for stress reduction techniques. 5. Teach the client to increase fiber in the diet.

1. Encourage a low-fat, low-cholesterol diet. 2. Instruct client to walk 30 minutes a day. 4. Refer to counselor for stress reduction techniques. 5. Teach the client to increase fiber in the diet. 1. A low-fat, low-cholesterol diet will help decrease the buildup of atherosclerosis in the arteries. 2. Walking will help increase collateral circulation. 3. Salt should be restricted in the diet of a client with hypertension, not coronary artery disease. 4. Stress reduction is encouraged for clients with CAD because this helps prevent excess stress on the heart muscle. 5. Increasing fiber in the diet will help remove cholesterol via the gastrointestinal system. TEST-TAKING HINT: This is an alternatetype question where the test taker must select all interventions that are applicable to the situation. Coronary artery disease is a common disease, and the nurse must be knowledgeable about ways to modify risk factors.

The client with coronary artery disease is prescribed a Holter monitor. Which intervention should the nurse implement? 1. Instruct client to keep a diary of activity, especially when having chest pain. 2. Discuss the need to remove the Holter monitor during a.m. care and showering. 3. Explain that all medications should be withheld while wearing a Holter monitor. 4. Teach the client the importance of decreasing activity while wearing the monitor.

1. Instruct client to keep a diary of activity, especially when having chest pain. 1. The Holter monitor is a 24-hour electrocardiogram, and the client must keep an accurate record of activity so that the health-care provider can compare the ECG recordings with different levels of activity. 2. The Holter monitor should not be removed for any reason. 3. All medications should be taken as prescribed. 4. The client should perform all activity as usual while wearing the Holter monitor so the HCP can get an accurate account of heart function during a 24-hour period. TEST-TAKING HINT: In some instances, the test taker must be knowledgeable about diagnostic tests and there are no Test- Taking Hints. The test taker might eliminate option "3" by realizing that, unless the client is NPO for a test or surgery, medications are usually taken.

59. The 6-year-old client diagnosed with cystic fibrosis (CF) needs a lung transplant. Which individual would be the best donor for the client? 1. The 20-year-old brother who does not have cystic fibrosis. 2. The 45-year-old father who carries the cystic fibrosis gene. 3. The 18-year-old who died in an MVA who matches on four (4) points. 4. The 5-year-old drowning victim who is a three (3)-point match.

1. Living donors are able to donate some organs. The kidneys, a portion of the liver, and a lung may be donated, and the donor will still have functioning organs. An identical twin is the best possible match. However, in the situation in this question, the identical twin would also have CF because the genes would be identical. The next best chance for a compatible match comes from a sibling with both parents in common

4. The client who is of the Jewish faith died during the night. The nurse notified the family, who do not want to come to the hospital. Which intervention should the nurse implement to address the family's behavior? 1. Take no further action because this is an accepted cultural practice. 2. Notify the hospital supervisor and report the situation immediately. 3. Call the local synagogue and request the rabbi go to the family's home. 4. Assume the family does not care about the client and follow hospital protocol.

1. Many of the Jewish faith do not believe in viewing or touching the dead body. The body is sent to the funeral home for burial within 24 hours, and a closed casket is preferred.

The client has just had a pericardiocentesis. Which interventions should the nurse implement? Select all that apply. 1. Monitor vital signs every 15 minutes for the first hour. 2. Assess the client's heart and lung sounds. 3. Record the amount of fluid removed as output. 4. Evaluate the client's cardiac rhythm. 5. Keep the client in the supine position.

1. Monitor vital signs every 15 minutes for the first hour. 2. Assess the client's heart and lung sounds. 3. Record the amount of fluid removed as output. 4. Evaluate the client's cardiac rhythm 1. The nurse should monitor the vital signs for any client who has just undergone surgery. 2. A pericardiocentesis involves entering the pericardial sac. Assessing heart and lung sounds allows assessment for cardiac failure. 3. The pericardial fluid is documented as output. 4. Evaluating the client's cardiac rhythm allows the nurse to assess for cardiac failure, which is a complication of pericardiocentesis. 5. The client should be in the semi-Fowler's position, not in a flat position, which increases the workload of the heart. TEST-TAKING HINT: This is an alternatetype question that requires the test taker to select possibly more than one option as a correct answer.

The client is diagnosed with acute pericarditis. Which sign/symptom warrants immediate attention by the nurse? 1. Muffled heart sounds. 2. Nondistended jugular veins. 3. Bounding peripheral pulses. 4. Pericardial friction rub.

1. Muffled heart sounds. 1. Acute pericardial effusion interferes with normal cardiac filling and pumping, causing venous congestion and decreased cardiac output. Muffled heart sounds, indicative of acute pericarditis, must be reported to the health-care provider. 2. Distended, not nondistended, jugular veins would warrant immediate intervention. 3. Decreasing quality of peripheral pulses, not bounding peripheral pulses, would warrant immediate intervention. 4. A pericardial friction rub is a classic symptom of acute pericarditis, but it would not warrant immediate intervention. TEST-TAKING HINT: The test taker must understand what the question is asking. "Warrants" is a word that should make the test taker think the correct answer will be something "bad," so the test taker should look for abnormal data; this would cause the test taker to eliminate options "2" and "3."

29. The client is being discharged from the hospital for intractable pain secondary to cancer and is prescribed morphine, a narcotic. Which statement indicates the client understands the discharge instructions? 1. "I will be sure to have my prescriptions filled before any holiday." 2. "There should not be a problem having the prescriptions filled anytime." 3. "If I run out of medications, I can call the HCP to phone in a prescription." 4. "There are no side effects to morphine I should be concerned about."

1. Narcotic medications require handwritten prescription forms (Drug Enforcement Agency rules) which must be filled within a limited time frame from the time the prescription is written. Many local pharmacies will not have the medication available or may not have it in the quantities needed. The client should anticipate the needs prior to any time when the HCP may not be available or the pharmacy may be closed.

The intensive care department nurse is assessing the client who is 12 hours post-myocardial infarction. The nurse assesses an S3 heart sound. Which intervention should the nurse implement? 1. Notify the health-care provider immediately. 2. Elevate the head of the client's bed. 3. Document this as a normal and expected finding. 4. Administer morphine intravenously.

1. Notify the health-care provider immediately. 1. An S3 indicates left ventricular failure and should be reported to the healthcare provider. It is a potential lifethreatening complication of a myocardial infarction. 2. Elevating the head of the bed will not do anything to help a failing heart. 3. This is not a normal finding; it indicates heart failure. 4. Morphine is administered for chest pain, not for heart failure, which is suggested by the S3 sound. TEST-TAKING HINT: There are some situations in which the nurse must notify the health-care provider, and the test taker should not automatically eliminate this as a possible correct answer. The test taker must decide if any of the other three options will help correct a life-threatening complication. Normal assessment concepts should help identify the correct option. The normal heart sounds are S1 and S2 ("lubb-dupp"); S3 is abnormal.

39. The nurse is teaching a class on disaster preparedness. Which are components of an Emergency Operations Plan (EOP)? Select all that apply. 1. A plan for practice drills. 2. A deactivation response. 3. A plan for internal communication only. 4. A pre-incident response. 5. A security plan.

1. Practice drills allow for troubleshooting any issues before a real-life incident occurs. 2. A deactivation response is important so resources are not overused, and the facility can then get back to daily activities and routine care. 5. A coordinated security plan involving facility and community agencies is the key to controlling an otherwise chaotic situation. 3. Communication between the facility and external resources and an internal communication plan are critical. 4. A postincident response is important to include a critique and debriefing for all parties involved; a pre-incident response is the plan itself. Be sure to read adjectives closely. TEST-TAKING HINT: The test taker must notice adjectives such as "only" in option "3" and "pre-incident" in option "4." These words make these options incorrect. This question requires the test taker to select more than one option as the correct answer.

The client diagnosed with pericarditis is experiencing cardiac tamponade. Which collaborative intervention should the nurse anticipate for this client? 1. Prepare for a pericardiocentesis. 2. Request STAT cardiac enzymes. 3. Perform a 12-lead electrocardiogram. 4. Assess the client's heart and lung sounds.

1. Prepare for a pericardiocentesis. 1. A pericardiocentesis removes fluid from the pericardial sac and is the emergency treatment for cardiac tamponade. 2. Cardiac enzymes may be slightly elevated because of the inflammatory process, but evaluation of these would not be ordered to treat or evaluate cardiac tamponade. 3. A 12-lead ECG would not help treat the medical emergency of cardiac tamponade. 4. Assessment by the nurse is not collaborative; it is an independent nursing action. TEST-TAKING HINT: "Collaborative" means another member of the health-care team must order or participate in the intervention. Therefore option "4" could be eliminated as a possible correct answer.

63. The nurse working in a homeless shelter identifies an adolescent female sexually aggressive toward some of the males in the shelter. Which is the most common cause for this behavior? 1. The client is acting in a learned behavior pattern to get attention. 2. The client had to leave home because of promiscuous behavior. 3. The client has a psychiatric disorder called nymphomania. 4. The client is a prostitute and is trying to get customers.

1. Research suggests at least 67% of adolescents who are runaways or homeless have been abused in the home. This represents a learned behavior pattern getting the female adolescent attention. 2. One reason adolescents of both sexes run away from home is abuse in the home. Nothing in the stem indicates the client was turned out of the home for any behavior. 3. This has the nurse medically diagnosing the client. 4. This is a judgmental statement. TEST-TAKING HINT: The test taker should not read into the question or choose an option allowing the nurse to function outside the scope of practice. Option "2" is assuming facts not in the stem, and option "3" is asking the nurse to make a medical diagnosis.

The client had open-heart surgery to replace the mitral valve. Which intervention should the intensive care unit nurse implement? 1. Restrict the client's fluids as ordered. 2. Keep the client in the supine position. 3. Maintain oxygen saturation at 90%. 4. Monitor the total parenteral nutrition.

1. Restrict the client's fluids as ordered. 1. Fluid intake may be restricted to reduce the cardiac workload and pressures within the heart and pulmonary circuit. 2. The head of the bed should be elevated to help improve alveolar ventilation. 3. Oxygen saturation should be no less than 93%; 90% indicates an arterial oxygen saturation of around 60 (normal is 80 to 100) 4. Total parenteral nutrition would not be prescribed for a client with mitral valve replacement. It is ordered for clients with malnutrition, gastrointestinal disorders, or conditions in which increased calories are needed, such as burns. TEST-TAKING HINT: A client with a heart or lung problem should never have the head of the bed in a flat (supine) position; therefore, option "2" should be eliminated as a possible correct answer. The test taker must know normal values for monitoring techniques such as pulse oximeters and keep a list of normal values.

The client is in ventricular fibrillation. Which interventions should the nurse implement? Select all that apply. 1. Start cardiopulmonary resuscitation. 2. Prepare to administer the antidysrhythmic adenosine IVP. 3. Prepare to defibrillate the client. 4. Bring the crash cart to the bedside. 5. Prepare to administer the antidysrhythmic amiodarone IVP.

1. Start cardiopulmonary resuscitation. 3. Prepare to defibrillate the client. 4. Bring the crash cart to the bedside. 5. Prepare to administer the antidysrhythmic amiodarone IVP. 1. Ventricular fibrillation indicates the client does not have a heartbeat. Therefore, CPR should be instituted. 2. Adenosine, an antidysrhythmic, is the drug of choice for supraventricular tachycardia, not for ventricular fibrillation. 3. Defibrillation is the treatment of choice for ventricular fibrillation. 4. The crash cart has the defibrillator and is used when performing advanced cardiopulmonary resuscitation. 5. Amiodarone is an antidysrhythmic that is used in ventricular dysrhythmias. TEST-TAKING HINT: This is an alternatetype question that requires the test taker to possibly select more than one option. To receive credit, the test taker must select all correct options; partial credit is not given for this type of question.

The nurse assessing the client with pericardial effusion at 1600 notes the apical pulse is 72 and the BP is 138/94. At 1800, the client has neck vein distention, the apical pulse is 70, and the BP is 106/94. Which action would the nurse implement first? 1. Stay with the client and use a calm voice. 2. Notify the health-care provider immediately. 3. Place the client left lateral recumbent. 4. Administer morphine intravenous push slowly.

1. Stay with client and use a calm voice. This is a medical emergency; the nurse should stay with the client, keep him calm, and call the nurses' station to notify the health-care provider. Cardiac output declines with each contraction as the pericardial sac constricts the myocardium.

33. The client is NPO and is receiving total parenteral nutrition (TPN) via a subclavian line. Which precautions should the nurse implement? Select all that apply. 1. Place the solution on an IV pump at the prescribed rate. 2. Monitor blood glucose every six (6) hours. 3. Weigh the client weekly, first thing in the morning. 4. Change the IV tubing every three (3) days. 5. Monitor intake and output every shift.

1. TPN is a hypertonic solution that has enough calories, proteins, lipids, electrolytes, and trace elements to sustain life. It is administered via a pump to prevent too rapid infusion. 2. TPN contains 50% dextrose solution; therefore, the client is monitored to ensure that the pancreas is adapting to the high glucose levels. 5. Intake and output are monitored to observe for fluid balance.

3

1. The ED nurse is caring for a client diagnosed with multiple rib fractures. Which data should the nurse include in the assessment? 1. Level of orientation to time and place. 2. Current use and last dose of medication. 3. Symmetrical movement of the chest. 4. Time of last meal the client ate.

37. Which situation warrants the nurse obtaining information from a material safety data sheet (MSDS)? 1. The custodian spilled a chemical solvent in the hallway. 2. A visitor slipped and fell on the floor that had just been mopped. 3. A bottle of antineoplastic agent broke on the client's floor. 4. The nurse was stuck with a contaminated needle in the client's room.

1. The MSDS provides chemical information regarding specific agents, health information, and spill information for a variety of chemicals. It is required for every chemical found in the hospital. 2. This situation requires an occurrence or accident report. 3. Any facility administering antineoplastic agents (medications used to treat cancer) is required to have specific chemotherapy spill kits available and a policy and procedure included; in this situation the nurse already knows the chemical involved. 4. This requires a hospital variance report and notifying the employee health or infection control nurse. TEST-TAKING HINT: If the test taker were not aware of an MSDS, the name tells the test taker to look for content in the answer options addressing materials; therefore, options "2" and "4" could be eliminated as possible answers.

12. The nurse is aware the Patient Self-Determination Act of 1991 requires the health-care facility to implement which action? 1. Make available an AD on admission to the facility. 2. Assist the client with legally completing a will. 3. Provide ethically and morally competent care to the client. 4. Discuss the importance of understanding consent forms.

1. The Patient Self-Determination Act of 1991 requires health-care facilities which receive Medicare or Medicaid funding to make ADs available to clients on admission into the facility

9. The nurse and unlicensed nursing assistant are caring for clients on a medical floor. Which nursing task would be most appropriate for the nurse to delegate? 1. Collect a clean voided midstream urine specimen. 2. Evaluate the client's 8-hour intake and output. 3. Assist in checking a unit of blood prior to hanging. 4. Administer a cation-exchange resin enema.

1. The assistant can collect specimens. Collecting a midstream urine specimen requires the client to clean the perineal area, to urinate a little, and then collect the rest of the urine output in a sterile container.

A client is being seen in the clinic to R/O mitral valve stenosis. Which assessment data would be most significant? 1. The client complains of shortness of breath when walking. 2. The client has jugular vein distention and 3+ pedal edema. 3. The client complains of chest pain after eating a large meal. 4. The client's liver is enlarged and the abdomen is edematous.

1. The client complains of shortness of breath when walking. 1. Dyspnea on exertion (DOE) is typically the earliest manifestation of mitral valve stenosis. 2. Jugular vein distention (JVD) and 3+ pedal edema are signs/symptoms of right-sided heart failure and indicate worsening of the mitral valve stenosis. These signs would not be expected in a client with early manifestations of mitral valve stenosis. 3. Chest pain rarely occurs with mitral valve stenosis. 4. An enlarged liver and edematous abdomen are late signs of right-sided heart failure that can occur with long-term untreated mitral valve stenosis. TEST-TAKING HINT: Whenever the test taker reads "rule out," the test taker should look for data that would not indicate a severe condition of the body system that is affected. Chest pain, JVD, and pedal edema are late signs of heart problems.

The nurse on the telemetry unit has just received the a.m. shift report. Which client should the nurse assess first? 1. The client diagnosed with myocardial infarction who has an audible S3 heart sound. 2. The client diagnosed with congestive heart failure who has 4+ sacral pitting edema. 3. The client diagnosed with pneumonia who has a pulse oximeter reading of 94%. 4. The client with chronic renal failure who has an elevated creatinine level.

1. The client diagnosed with myocardial infarction who has an audible S3 heart 1. An S3 heart sound indicates left ventricular failure, and the nurse must assess this client first because it is an emergency situation. 2. The nurse would expect a client with CHF to have sacral edema of 4+; the client with an S3 would be in a more life-threatening situation. 3. A pulse oximeter reading of greater than 93% is considered normal. 4. An elevated creatinine level is expected in a client diagnosed with chronic renal failure. TEST-TAKING HINT: Because the nurse will be assessing each client, the test taker must determine which client is a priority. A general guideline for this type of question is for the test taker to ask "Is this within normal limits?" or "Is this expected for the disease process?" If the answer is yes to either question, then the test taker can eliminate these options and look for abnormal data that would make that client a priority.

76. The client diagnosed with cancer of the bladder is undergoing intravesical chemotherapy. Which instruction should the nurse provide the client about the pre-therapy routine? 1. Instruct the client to remain NPO after midnight before the procedure. 2. Explain the use of chemotherapy in bladder cancer. 3. Teach the client to administer Neupogen, a biologic response modifier. 4. Have the client take Tylenol, an analgesic, before coming to the clinic.

1. The client will have medication instilled in the bladder that must remain in the bladder for a prescribed length of time. For this reason, the client must remain NPO before the procedure.

The nurse writes a client problem of "spiritual distress" for the client who is dying. Which statement is an appropriate goal? 1. The client will reconcile self and the higher power of his or her beliefs. 2. The client will be able to express anger at the terminal diagnosis 3. The client will reconcile self to estranged family member 4. The client will have a dignified and pain-free death

1. The client will reconcile self and the higher power of his or her beliefs. The primary goal of spiritual care is to allow the client to be able to reconcile themselves with higher being. maybe God. This goal is based on the belief that life comes from God, and to some degree for many people the process of living includes some separation from God. In the Western world, 95% of the people claim some belief in God.

23. Which action by the primary nurse would require the unit manager to intervene? 1. The nurse uses a correction fluid to correct a charting mistake. 2. The nurse is shredding the worksheet at the end of the shift. 3. The nurse circles an omitted medication time on the MAR. 4. The nurse documents narcotic wastage with another nurse.

1. The client's chart is a legal document, and if a mistake occurs, it should be corrected by marking one line through the entry in such a way the entry can still be read in a court of law. Erasing, using a correction fluid, or obliterating the entry is illegal.

19. The hospice care nurse is planning the care of an elderly client diagnosed with end-stage renal disease. Which interventions should be included in the plan of care? Select all that apply. 1. Discuss financial concerns. 2. Assess any comorbid conditions. 3. Monitor increased visual or auditory abilities. 4. Note any spiritual distress. 5. Encourage euphoria at the time of death.

1. The elderly are frequently on fixed incomes, and financial concerns are important for the nurse to address. A social services referral may be needed. 2. The elderly may have many comorbid conditions, which affect the type and amount of medications the client can tolerate and the client's quality of life. 4. The client may feel some spiritual distress at the terminal diagnosis. Even if the client possesses a strong faith, the unknown can be frightening.

The HCP has notified the family of a client in a persistent vegetative state on a ventilator of the need to "pull the plug." The client does not have an AD or a durable power of attorney for health care, and the family does not want their loved one removed from the ventilator. Which action should the nurse implement? 1. Refer the case to the hospital ethics committee. 2. Tell the family they must do what the HCP orders. 3. Follow the HCP's order and "pull the plug." 4. Determine why the client did not complete an AD.

1. The ethics committee is composed of health-care workers and laypeople from the community to objectively review the situation and make a recommendation which is fair to both the client and healthcare system. The family has the right to be present and discuss their feelings

8. The HCP has notified the family of a client in a persistent vegetative state on a ventilator of the need to "pull the plug." The client does not have an AD or a durable power of attorney for health care, and the family does not want their loved one removed from the ventilator. Which action should the nurse implement? 1. Refer the case to the hospital ethics committee. 2. Tell the family they must do what the HCP orders. 3. Follow the HCP's order and "pull the plug." 4. Determine why the client did not complete an AD.

1. The ethics committee is composed of health-care workers and laypeople from the community to objectively review the situation and make a recommendation which is fair to both the client and healthcare system. The family has the right to be present and discuss their feelings.

65. The client diagnosed with renal calculi is scheduled for a 24-hour urine specimen collection. Which interventions should the nurse implement? Select all that apply. 1. Check for the ordered diet and medication modifications. 2. Instruct the client to urinate, and discard this urine when starting collection. 3. Collect all urine during 24 hours and place in appropriate specimen container. 4. Insert a Foley catheter in client after having the client empty the bladder. 5. Post notices on the client's door to save all urine output.

1. The health-care provider may order certain foods and medications when obtaining 24- hour urine collection to evaluate for calcium oxalate or uric acid. 2. When the collection begins, the client should completely empty the bladder and discard that urine. 3. All urine for 24 hours should be saved and put in a container with preservative, refrigerated, or put on ice as indicated. Not following specific instructions will result in an inaccurate test result. 5. Posting signs will help ensure that all the urine is saved during the 24-hour period. If any urine is discarded, the test may result in inaccurate information or the need to start the test over.

16. The nurse is teaching a class on chronic pain to new graduates. Which information is most important for the nurse to discuss? 1. The nurse must believe the client's report of pain. 2. Clients in chronic pain may not show objective signs. 3. Alternate pain-control therapies are used for chronic pain. 4. Referral to a pain clinic may be necessary.

1. The most important information for a nurse caring for a client with acute or chronic pain is to believe the client. Pain is subjective, and the nurse should not be judgmental.

5. The hospice nurse is making the final visit to the wife whose husband died a little more than a year ago. The nurse realizes the husband's clothes are still in the closet and chest of drawers. Which action should the nurse implement first? 1. Discuss what the wife is going to do with the clothes. 2. Refer the wife to a grief recovery support group. 3. Do not take any action because this is normal grieving. 4. Remove the clothes from the house and dispose of them.

1. The nurse must first confront the wife about moving on through the grieving process. After one (1) year, the wife should be seriously thinking about what to do with her husband's belongings.

24. Which action should the nurse implement for the Chinese client's family who are requesting to light incense around the dying client? 1. Suggest the family bring potpourri instead of incense. 2. Tell the client the door must be shut at all times. 3. Inform the family the scent will make the client nauseated. 4. Explain fire code does not allow any burning in a hospital.

1. The nurse must support the client's culture. Potpourri provides the scent without having the burning incense, which is against fire code, and thus is a compromise which supports the client's culture.

9. Which interventions should the nurse implement at the time of a client's death? Select all that apply. 1. Allow gaps in the conversation at the client's bedside. 2. Avoid giving the family advice about how to grieve. 3. Tell the family the nurse understands their feelings. 4. Explain this is God's will to prevent further suffering. 5. Allow the family time with the body in private.

1. The nurse needs to be sensitive to the family, and simply being present to support the family emotionally is important; the nurse does not have to talk. 2. The nurse should avoid the impulse to give advice; each person grieves in his or her own way. 5. The family needs time for closure, and allowing the family to stay at the bedside is meeting the family's need to say good-bye.

59. The client returned from surgery after having a TURP and has a P 110, R 24, B/P 90/40, and cool and clammy skin. Which interventions should the nurse implement? Select all that apply. 1. Assess the red urine in the continuous irrigation drainage bag. 2. Increase the irrigation fluid in the continuous irrigation catheter. 3. Lower the head of the bed while raising the foot of the bed. 4. Contact the surgeon to give an update in the client's condition. 5. Monitor the client's postoperative hematocrit and hemoglobin.

1. The nurse should assess the drain postoperative. 2. The nurse should increase the irrigation fluid to clear the red urine. 3. The head of the bed should be lowered and the foot should be elevated to protect the brain. 4. The surgeon needs to be notified of the change in condition. 5. These laboratory values should be assessed for bleeding.

21. The client is on the ventilator and has been declared brain dead. The spouse refuses to allow the ventilator to be discontinued. Which collaborative action by the nurse is most appropriate? 1. Discuss referral of the case to the ethics committee. 2. Pull the plug when the spouse is not in the room. 3. Ask the HCP to discuss the futile situation with the spouse. 4. Inform the spouse what is happening is cruel.

1. The nurse should discuss using the ethics committee with the HCP to assist the family in making the decision to terminate life support. Many families feel there may be a racial or financial reason the HCP wants to discontinue life support.

18. The nurse writes a client problem of "spiritual distress" for the client who is dying. Which statement is an appropriate goal? 1. The client will reconcile self and the higher power of his or her beliefs. 2. The client will be able to express anger at the terminal diagnosis. 3. The client will reconcile self to estranged members of the family. 4. The client will have a dignified and pain-free death.

1. The primary goal of spiritual care is to allow the client to be able to reconcile himself or herself with a higher being, maybe God. This goal is based on the belief that life comes from God, and to some degree for many people the process of living includes some separation from God. In the Western world, 95% of the people claim some belief in God

13. The spouse of a client dying from lung cancer states, "I don't understand this death rattle. She has not had anything to drink in days. Where is the fluid coming from?" Which is the hospice care nurse's best response? 1. "The body produces about two (2) teaspoons of fluid every minute on its own." 2. "Are you sure someone is not putting ice chips in her mouth?" 3. "There is no reason for this, but it does happen from time to time." 4. "I can administer a patch to her skin to dry up the secretions if you wish."

1. The respiratory tract cells produce liquid as a defense mechanism against bacteria and other invaders. About nine (9) mL a minute are produced. The "death rattle" can be disturbing to family members, and the nurse should intervene but not with suctioning, which will increase secretions and the need to suction more.

40. The nurse is obtaining the client's signature on a surgical permit form. The nurse determines the client does not understand the surgical procedure and possible risks. Which action should the nurse take first? 1. Notify the client's surgeon. 2. Document the information in the chart. 3. Contact the operating room staff. 4. Explain the procedure to the client.

1. The surgeon is responsible for explaining the surgical procedure to the client; therefore, the nurse should first notify the surgeon

The client is admitted into the emergency department with diaphoresis, pale clammy skin, and BP of 90/70. Which intervention should the nurse implement first? 1. Start an IV with an 18-gauge catheter. 2. Administer dopamine intravenous infusion. 3. Obtain arterial blood gases (ABGs). 4. Insert an indwelling urinary catheter.

1. There are many types of shock, but the one common intervention which should be done first in all types of shock is to establish an intravenous line with a large-bore catheter. The low blood pressure and cold, clammy skin indicate shock.

84. The female client diagnosed with bladder cancer with a cutaneous urinary diversion states, "Will I be able to have children now?" Which statement is the nurse's best response? 1. "Cancer does not make you sterile, but sometimes the therapy can." 2. "Are you concerned that you can't have children?" 3. "You will be able to have as many children as you want." 4. "Let me have the chaplain come to talk with you about this."

1. This client is asking for information and should be given factual information. The surgery will not make the client sterile, but chemotherapy can induce menopause and radiation therapy to the pelvis can render a client sterile.

58. The experienced medical-surgical nurse is being oriented to the transplant unit. Which client should the charge nurse assign to this nurse? 1. The client who donated a kidney to a relative three (3) days ago and will be discharged in the morning. 2. The client who had a liver transplantation three (3) days ago and was transferred from the intensive care unit two (2) hours ago. 3. The client who received a corneal transplant four (4) hours ago and has developed a cough and is vomiting. 4. The client who had a pancreas transplantation and has a fever, chills, and a blood glucose monitor reading of 342.

1. This client is ready for discharge and is presumably stable. The client donated the kidney and still has one functioning kidney. An experienced medical-surgical nurse could care for this client

11. The client has been declared brain dead and is an organ donor. The nurse is preparing the wife of the client to enter the room to say good-bye. Which information is most important for the nurse to discuss with the wife? 1. Inform the wife the client will still be on the ventilator. 2. Instruct the wife to only stay a few minutes at the bedside. 3. Tell the wife it is all right to talk to the client. 4. Allow another family member to go in with the wife.

1. This is the most important action because, when the wife walks in the room, the client's chest will be rising and falling, the monitor will show a heartbeat, and the client will be warm. Many family members do not realize this and think the client is still alive. The organs must be perfused until retrieved for organ donation

47. The father of a child brought to the emergency department is yelling at the staff and obviously intoxicated. Which approach should the nurse take with the father? 1. Talk to the father in a calm and low voice. 2. Tell the father to wait in the waiting room. 3. Notify the child's mother to come to the ED. 4. Call the police department to come and arrest him.

1. This will help diffuse the escalating situation and attempt to keep the father calm. 2. Sending the father to the waiting room does not help his behavior and could possibly make his behavior worse; loud and obnoxious behavior can become violent. 3. This will not help the current situation and could make it worse because the nurse doesn't know the home situation. 4. The nurse should notify hospital security before calling the police department. TEST-TAKING HINT: The rule concerning dealing with anger is to directly address the client and diffuse the situation. There is only one option addressing this rule, option "1."

55. The nurse is caring for a client who received a kidney transplant from an unrelated cadaver donor. Which interventions should be included in the plan of care? Select all that apply. 1. Collect a urine culture every other day. 2. Prepare the client for dialysis three (3) times a week. 3. Monitor urine osmolality studies. 4. Monitor intake and output every shift. 5. Check abdominal dressing every four (4) hours.

1. Urine cultures are performed frequently because of the bacteriuria present in the early stages of transplantation. 2. A cadaver kidney may have undergone acute tubular necrosis and may not function for two (2) to three (3) weeks, during which time the client may experience anuria, oliguria, or polyuria and require dialysis.

The nurse is assisting with a synchronized cardioversion on a client in atrial fibrillation. When the machine is activated, there is a pause. What action should the nurse take? 1. Wait until the machine discharges. 2. Shout "all clear" and don't touch the bed. 3. Make sure the client is all right. 4. Increase the joules and redischarge.

1. Wait until machine discharges Cardioversion involves the delivery of a timed electrical current. The electrical impulse discharges during ventricular depolarization and therefore there might be a short delay. The nurse should wait until it discharges.

12. The nurse is developing a care plan for the client diagnosed with type 1 diabetes. The nurse identifies the problem "high risk for hyperglycemia related to noncompliance with the medication regimen." Which statement is an appropriate short-term goal for the client? 1. The client will have a blood glucose level between 90 and 140 mg/dL. 2. The client will demonstrate appropriate insulin injection technique. 3. The nurse will monitor the client's blood glucose levels four (4) times a day. 4. The client will maintain normal kidney function with 30-mL/hr urine output.

12. 1. The short-term goal must address the response part of the nursing diagnosis, which is "high risk for hyperglycemia," and this blood glucose level is within acceptable ranges for a client who is noncompliant.

13. The occupational nurse for a mining company is planning a class on the risks of working with toxic substances to comply with the "Right to Know" law. Which information should the nurse include in the presentation? Select all that apply. 1. A client who smokes cigarettes has a drastically increased risk for lung cancer. 2. Floors need to be clean and dust needs to be wet to prevent transfer of dust. 3. The air needs to be monitored at specific times to evaluate for exposure. 4. Surface areas need to be painted every year to prevent the accumulation of dust. 5. Employees should wear the appropriate personal protective equipment.

13. 1. Clients who smoke cigarettes and work with toxic substances have increased risk of lung cancer because many of the substances are carcinogenic. 2. When floors and surfaces are kept clean, toxic dust particles, such as asbestos and silica, are controlled and this decreases exposure. Covering areas with water controls dust. 3. The quality of air is monitored to determine what toxic substances are present and in what amount. The information is then used in efforts to minimize the amount of exposure. 5. Employees must wear protective coverings, goggles, and other equipment needed to eliminate exposure to the toxic substances.

The elderly client is admitted to the intensive care department diagnosed with severe HHNS. Which collaborative intervention should the nurse include in the plan of care? 1. Infuse 0.9% normal saline intravenously. 2. Administer intermediate-acting insulin. 3. Perform blood glucometer checks daily. 4. Monitor arterial blood gas results.

14. 1. The initial fluid replacement is 0.9% normal saline (an isotonic solution) intravenously, followed by 0.45% saline. The rate depends on the client's fluid volume status and physical health, especially of the heart.

15. The nurse is providing discharge teaching to the client diagnosed with polycystic kidney disease. Which statement made by the client indicates the teaching has been effective? 1. "I need to avoid any activity causing a risk for injury to my kidney." 2. "I should avoid taking medications for high blood pressure." 3. "When I urinate there may be blood streaks in my urine." 4. "I may have occasional burning when I urinate with this disease."

15. 1. Polycystic kidney disease poses an increased risk for rupture of the kidney, and therefore sports activities or occupations with risks for trauma should be avoided.

3

15. The ED nurse is caring for a client with fractured pelvis and bladder trauma secondary to a motor-vehicle accident. Which data are most important for the nurse to assess? 1. Monitor the creatinine and BUN. 2. Check urine output hourly. 3. Note the amount and color of the urine. 4. Assess for bladder distention.

3

15. The nurse is teaching a class on bioterrorism and is discussing personal protective equipment (PPE). Which statement is the most important fact for the nurse to share with the participants? 1. Health-care facilities should keep masks at entry doors. 2. The respondent should be trained in the proper use of PPE. 3. No single combination of PPE protects against all hazards. 4. The EPA has divided PPE into four levels of protection.

3

15. Which action by the unlicensed assistive personnel (UAP) would warrant immediate intervention by the nurse? 1. The UAP is holding the phone to the ear of a client who is a quadriplegic. 2. The UAP refuses to discuss the client's condition with the visitor in the room. 3. The UAP put a vest restraint on an elderly client found wandering in the hall. 4. The UAP is assisting the client with arthritis to open up personal mail.

17. The client diagnosed with type 1 diabetes is found lying unconscious on the floor of the bathroom. Which intervention should the nurse implement first? 1. Administer 50% dextrose IVP. 2. Notify the health-care provider. 3. Move the client to the ICU. 4. Check the serum glucose level.

17. 1. The nurse should assume the client is hypoglycemic and administer IVP dextrose, which will rouse the client immediately. If the collapse is the result of hyperglycemia, this additional dextrose will not further injure the client.

2

17. The ED receives a client involved in a motor-vehicle accident. The nurse notes a large hematoma on the right flank. Which intervention should the nurse implement first? 1. Insert an indwelling urinary catheter. 2. Take the vital signs every 15 minutes. 3. Monitor the skin turgor every hour. 4. Mark the edges of the bruised area.

4

17. The client with chronic low back pain is having trouble sleeping at night. Which nonpharmacological therapy should the nurse teach the client? 1. Acupuncture. 2. Massage therapy. 3. Herbal remedies. 4. Progressive relaxation techniques.

3

17. The triage nurse in a large trauma center has been notified of an explosion in a major chemical manufacturing plant. Which action should the nurse implement first when the clients arrive at the emergency department? 1. Triage the clients and send them to the appropriate areas. 2. Thoroughly wash the clients with soap and water and then rinse. 3. Remove the clients' clothing and have them shower. 4. Assume the clients have been decontaminated at the plant.

18. The nurse is caring for the client recovering from a percutaneous renal biopsy. Which data indicate the client is complying with client teaching? 1. The client is lying flat in the supine position. 2. The client continues oral fluids restriction while on bedrest. 3. The client uses the bedside commode to urinate. 4. The client refuses to ask for any pain medication.

18. 1. The client needs to lie flat on the back to apply pressure to prevent bleeding.

2

18. The client diagnosed with cancer is unable to attain pain relief despite receiving large amounts of narcotic medications. Which intervention should be included in the plan of care? 1. Ask the HCP to increase the medication. 2. Assess for any spiritual distress. 3. Change the client's position every two (2) hours. 4. Turn on the radio to soothing music.

3

18. Which expected outcome is priority for the nurse who is caring for a client with chest trauma from a gunshot injury? 1. The client will have an absence of pain. 2. The client will maintain a BP of 90/60. 3. The client will have symmetrical chest expansion. 4. The client will maintain urine output of 30 mL/hr.

Which client would the nurse suspect of having a mitral valve prolapse? 1. A 60-year-old female with congestive heart failure. 2. A 23-year-old male with Marfan's syndrome. 3. An 80-year-old male with atrial fibrillation. 4. A 33-year-old female with Down syndrome.

2. A 23-year-old male with Marfan's syndrome. 1. Congestive heart failure does not predispose the female client to having a mitral valve prolapse. 2. Clients with Marfan's syndrome have life-threatening cardiovascular problems, including mitral valve prolapse, progressive dilation of the aortic valve ring, and weakness of the arterial walls, and they usually do not live past the age of 40 because of dissection and rupture of the aorta. 3. Atrial fibrillation does not predispose a client to mitral valve prolapse. 4. A client with Down syndrome may have congenital heart anomalies but not mitral valve prolapse. TEST-TAKING HINT: The test taker could eliminate options "1" and "3" based on knowledge that these are commonly occurring cardiovascular problems, and the nurse should know that possible complications of these problems do not include mitral valve prolapse.

Which assessment data would the nurse expect to auscultate in the client diagnosed with mitral valve insufficiency? 1. A loud S1, S2 split, and a mitral opening snap. 2. A holosystolic murmur heard best at the cardiac apex. 3. A midsystolic ejection click or murmur heard at the base. 4. A high-pitched sound heard at the third left intercostal space.

2. A holosystolic murmur heard best at the cardiac apex. 1. This would be expected with mitral valve stenosis. 2. The murmur associated with mitral valve insufficiency is loud, highpitched, rumbling, and holosystolic (occurring throughout systole) and is heard best at the cardiac apex. 3. This would be expected with mitral valve prolapse. 4. This would be expected with aortic regurgitation. TEST-TAKING HINT: This is a knowledgebased question and there is no Test- Taking Hint to help the test taker rule out distracters.

75. The client diagnosed with cancer of the bladder is scheduled to have a cutaneous urinary diversion procedure. Which preoperative teaching intervention specific to the procedure should be included? 1. Demonstrate turn, cough, and deep breathing. 2. Explain that a bag will drain the urine from now on. 3. Instruct the client on the use of a PCA pump. 4. Take the client to the ICD so that he or she can become familiar with it

2. A urinary diversion procedure involves the removal of the bladder. In a cutaneous procedure the ureters are implanted in some way to allow for stoma formation on the abdominal wall, and the urine then drains into a pouch. There are numerous methods used for creating the stoma.

The client tells the nurse, "Every time I come in the hospital you hand me one of these advance directives (AD). Why should I fill one of these out?" Which statement by the nurse is most appropriate? 1. "You must fill out this form because Medicare laws require it." 2. "An AD lets you participate in decisions about your health care." 3. "This paper will ensure no one can override your decisions." 4. "It is part of the hospital admission packet and I have to give it to you."

2. ADs allow the client to make personal health-care decisions about end-of-life issues, including cardiopulmonary resuscitation (CPR), ventilators, feeding tubes, and other issues concerning the client's death. TEST-TAKING HINT: The test taker could eliminate option "1" because the nurse cannot make the client do anything. The client has a right to say no. Option "3" is an absolute, and unless the test taker knows for sure this is correct information, the test taker should not select this option. Content - Medical: Category of Health Alteration - Patient Advocacy: Integrated Nursing Process - Implementation: Client Needs - Safe Effective Care Environment, Management of Care: Cognitive Level - Application.

The nurse is caring for a client diagnosed with a myocardial infarction who is experiencing chest pain. Which interventions should the nurse implement? Select all that apply. 1. Administer morphine intramuscularly. 2. Administer an aspirin orally. 3. Apply oxygen via a nasal cannula. 4. Place the client in a supine position. 5. Administer nitroglycerin subcutaneously.

2. Administer an aspirin orally. 3. Apply oxygen via a nasal cannula. 1. Morphine should be administered intravenously, not intramuscularly. 2. Aspirin is an antiplatelet medication and should be administered orally. 3. Oxygen will help decrease myocardial ischemia, thereby decreasing pain. 4. The supine position will increase respiratory effort, which will increase myocardial oxygen consumption; the client should be in the semi-Fowler's position. 5. Nitroglycerin, a coronary vasodilator, is administered sublingually, not subcutaneously. TEST-TAKING HINT: This is an alternatetype question that requires the test taker to select all options that are applicable. The test taker must identify all correct answer options to receive credit for a correct answer; no partial credit is given. Remember to read the question carefully—it is not meant to be tricky.

The client is in complete heart block. Which intervention should the nurse implement first? 1. Prepare to insert a pacemaker. 2. Administer atropine, an antidysrhythmic. 3. Obtain a STAT electrocardiogram (ECG). 4. Notify the health-care provider

2. Administer atropine, an antidysrhythmic. 1. A pacemaker will have to be inserted, but it is not the first intervention. 2. Atropine will decrease vagal stimulation and increase the heart rate. Therefore, it is the first intervention. 3. A STAT ECG may be done, but the telemetry reading shows complete heart block, which is a life-threatening dysrhythmia and must be treated. 4. The HCP will need to be notified but not prior to administering a medication. The test taker must assume the nurse has the order to administer medication. Many telemetry departments have standing protocols. TEST-TAKING HINT: The test taker must select the intervention that should be implemented first and will directly affect the dysrhythmia. Medication is the first intervention, and then pacemaker insertion. The test taker should not eliminate an option because the test taker thinks there

The male client has made himself a DNR is in pain. The client's vital signs are P 88, R 8, and BP 108/70. Which intervention should be the nurse's priority action? 1. Refuse to give the medication because it could kill the client 2. Administer the medication as orderer and access for relief from pain 3. Wait until the client' respirations improve and then administer the medication 4. Notify the HCP that the client is unstable and pain medication is being held

2. Administer the medication as orderer and access for relief from pain. The client is in pain. The American Nurse's Code of Ethics states that clients have the right to die as comfortably as possible even if the measures used to control pain indirectly hasten the impending death. The Dying Client's Bill of Right reiterates this position. The client should be allowed to die with dignity and with as much comfort as the nurse can provide

42. Which intervention should the nurse implement for the client experiencing bronchospasms? 1. Administer intravenous epinephrine, a bronchodilator. 2. Administer Albuterol, a bronchodilator, via nebulizer. 3. Request a STAT portable chest x-ray at the bedside. 4. Insert a small nasal trumpet in the right nostril.

2. Albuterol given via nebulizer is administered to stop the bronchospasms. If the client continues to have the bronchospasms, intubation may be needed.

The client is one (1) day postoperative coronary artery bypass surgery. The client complains of chest pain. Which intervention should the nurse implement first? 1. Medicate the client with intravenous morphine. 2. Assess the client's chest dressing and vital signs. 3. Encourage the client to turn from side to side. 4. Check the client's telemetry monitor.

2. Assess the client's chest dressing and vital signs. 1. The nurse should medicate the client as needed, but it is not the first intervention. 2. The nurse must always assess the client to determine if the chest pain that is occurring is expected postoperatively or if it is a complication of the surgery. 3. Turning will help decrease complications from immobility, such as pneumonia, but it will not help relieve the client's pain. 4. The nurse, not a machine, should always take care of the client. TEST-TAKING HINT: The stem asks the nurse to identify the first intervention that should be implemented. Therefore, the test taker should apply the nursing process and select an assessment intervention. Both options "2" and "4" involve assessment, but the nurse—not a machine or diagnostic test— should always assess the client.

The client is scheduled for a right femoral cardiac catheterization. Which nursing intervention should the nurse implement after the procedure? 1. Perform passive range-of-motion exercises. 2. Assess the client's neurovascular status. 3. Keep the client in high Fowler's position. 4. Assess the gag reflex prior to feeding the client.

2. Assess the client's neurovascular status. 1. The client's right leg should be kept straight to prevent arterial bleeding from the femoral insertion site for the catheter used to perform the catheterization. 2. The nurse must make sure that blood is circulating to the right leg, so the client should be assessed for pulses, paresthesia, paralysis, coldness, and pallor. 3. The head of the bed should be elevated no more than 10 degrees. The client should be kept on bedrest, flat with the affected extremity straight, to help decrease the chance of femoral artery bleeding. 4. The gag reflex is assessed if a scope is inserted down the trachea (bronchoscopy) or esophagus (endoscopy) because the throat is numbed when inserting the scope. A catheter is inserted in the femoral or brachial artery when performing a cardiac catheterization. TEST-TAKING HINT: The nurse should apply the nursing process when determining the correct answer. Therefore, either option "2" or option "4" could possibly be the correct answer. The test taker then should apply anatomy concepts—where is the left femoral artery? Neurovascular assessment is performed on extremities. Content - Surgical: Category of Health Alteration -

Which intervention should the nurse implement when administering a loop diuretic to a client diagnosed with coronary artery disease? 1. Assess the client's radial pulse. 2. Assess the client's serum potassium level. 3. Assess the client's glucometer reading. 4. Assess the client's pulse oximeter reading.

2. Assess the client's serum potassium level. 1. The nurse should always assess the apical (not radial) pulse, but the pulse is not affected by a loop diuretic. 2. Loop diuretics cause potassium to be lost in the urine output. Therefore, the nurse should assess the client's potassium level, and if the client is hypokalemic, the nurse should question administering this medication. 3. The glucometer provides a glucose level, which is not affected by a loop diuretic. 4. The pulse oximeter reading evaluates peripheral oxygenation and is not affected by a loop diuretic. TEST-TAKING HINT: Knowing that diuretics increase urine output would lead the test taker to eliminate glucose level and oxygenation (options "3" and "4"). In very few instances does the nurse assess the radial pulse; the apical pulse is assessed.

11. Which priority intervention should the nurse implement for the client diagnosed with coal workers' pneumoconiosis? 1. Monitor the client's intake and output. 2. Assess for black-streaked sputum. 3. Monitor the white blood cell count daily. 4. Assess the client's activity level every shift.

2. Black-streaked sputum is a classic sign of coal workers' pneumoconiosis (black lung), and the sputum should be assessed for color and amount. Remember Maslow's hierarchy of needs when answering priority questions.

60. Which expected outcome would indicate that the client's condition following a TURP is improving? 1. The client is using the maximum amount allowed by the PCA pump. 2. The client's bladder spasms are relieved by medication. 3. The client's scrotum is swollen and tender with movement. 4. The client has passed a large, hard, brown stool this morning.

2. Bladder spasms are common, but being relieved with medication indicates the condition is improving.

47. The home health nurse is admitting a client diagnosed with cancer of the pancreas. Which information is the most important for the nurse to discuss with the client? 1. Determine the client's food preferences. 2. Ask the client if there is an advance directive. 3. Find out about insurance/Medicare reimbursement. 4. Explain the client should eat as much as possible.

2. Cancer of the pancreas has a poor prognosis; the nurse should determine if the client has executed an advance directive regarding his or her wishes.

Which potential complication should the nurse assess for in the client with infective endocarditis who has embolization of vegetative lesions from the mitral valve? 1. Pulmonary embolus. 2. Cerebrovascular accident. 3. Hemoptysis. 4. Deep vein thrombosis.

2. Cerebrovascular accident. 1. Pulmonary embolus would occur with an embolization of vegetative lesions from the tricuspid valve on the right side of the heart. 2. Bacteria enter the bloodstream from invasive procedures, and sterile platelet-fibrin vegetation forms on heart valves. The mitral valve is on the left side of the heart and, if the vegetation breaks off, it will go through the left ventricle into the systemic circulation and may lodge in the brain, kidneys, or peripheral tissues. 3. Coughing up blood (hemoptysis) occurs when the vegetation breaks off the tricuspid valve in the right side of the heart and enters the pulmonary artery. 4. Deep vein thrombosis is a complication of immobility, not of a vegetative embolus from the left side of the heart. TEST-TAKING HINT: If the test taker does not know the answer, knowledge of anatomy may help determine the answer. The mitral valve is on the left side of the heart and any emboli would not enter the lung first, thereby eliminating options "1" and "3" as possible correct answers.

64. The adolescent female comes to the school nurse of an intermediate school and tells the nurse she thinks she is pregnant. During the interview, the client states her father is the baby's father. Which intervention should the nurse implement first? 1. Complete a rape kit. 2. Notify Child Protective Services 3. Call the parents to come to the school. 4. Arrange for the client to go to a free clinic.

2. Child Protective Services should be notified to protect the child from further abuse and to initiate charges against the father. An intermediate school nurse cares for children in the 4th, 5th, 6th, or 7th grades, depending on the school district. 1. The school nurse is not a Sexual Assault Nurse Examiner (SANE) nurse, and this child thinks she is pregnant, suggesting the abuse has been occurring for a period of time or at least in some months past. The child should be taken to a hospital for examination. 3. This action brings the abuser to the school. 4. Sending the child to a free clinic does not negate the nurse's responsibility to report suspected child abuse. TEST-TAKING HINT: All 50 states require the nurse to report suspected child abuse. Child Protective Services (CPS) is the advocate to notify. Nurses in a school clinic do not have the appropriate facilities to perform rape examinations. Option "4" does not address the abuse.

What is the priority problem in the client diagnosed with congestive heart failure? 1. Fluid volume overload. 2. Decreased cardiac output. 3. Activity intolerance. 4. Knowledge deficit.

2. Decreased cardiac output Decreased cardiac output is responsible for all the signs/symptoms associated with CHF and eventually causes death, which is why it is the priority problem.

Which client problem has priority for the client with a cardiac dysrhythmia? 1. Alteration in comfort. 2. Decreased cardiac output. 3. Impaired gas exchange. 4. Activity intolerance.

2. Decreased cardiac output. 1. Not every cardiac dysrhythmia causes alteration in comfort; angina is caused by decreased oxygen to the myocardium. 2. Any abnormal electrical activity of the heart causes decreased cardiac output. 3. Impaired gas exchange is the result of pulmonary complications, not cardiac dysrhythmias. 4. Not all clients with cardiac dysrhythmias have activity intolerance. TEST-TAKING HINT: Option "2" has the word "cardiac," which refers to the heart. Therefore, even if the test taker had no idea what the correct answer was, this would be an appropriate option. The test taker should use medical terminology to help identify the correct option.

Along with persistent, crushing chest pain, which signs/symptoms would make the nurse suspect that the client is experiencing a myocardial infarction? 1. Midepigastric pain and pyrosis. 2. Diaphoresis and cool clammy skin. 3. Intermittent claudication and pallor. 4. Jugular vein distention and dependent edema.

2. Diaphoresis and cool clammy skin. 1. Midepigastric pain would support a diagnosis of peptic ulcer disease; pyrosis is belching. 2. Diaphoresis (sweating) is a systemic reaction to the MI. The body vasoconstricts to shunt blood from the periphery to the trunk of the body; this, in turn, leads to cold, clammy skin. 3. Intermittent claudication is leg pain secondary to decreased oxygen to the muscle, and pallor is paleness of the skin as a result of decreased blood supply. Neither is an early sign of MI. 4. Jugular vein distension (JVD) and dependent edema are signs/symptoms of congestive heart failure, not of MI. TEST-TAKING HINT: The stem already addresses chest pain; therefore, the test taker could eliminate option "1" as a possible answer. Intermittent claudication, option "3," is the classic sign of arterial occlusive disease, and JVD is very specific to congestive heart failure. The nurse must be able to identify at least two or three signs/symptoms of disease processes.

Which intervention should the nurse implement with the client diagnosed with dilated cardiomyopathy? 1. Keep the client in the supine position with the legs elevated. 2. Discuss a heart transplant, which is the definitive treatment. 3. Prepare the client for coronary artery bypass graft. 4. Teach the client to take a calcium channel blocker in the morning.

2. Discuss a heart transplant, which is the definitive treatment. Without a heart transplant, this client will end up in end-stage heart failure. A transplant is the only treatment for a client with dilated cardiomyopathy.

The female client in the oncology clinic tells the nurse that she has a great deal of paint does not like to take pain medication. Which action should the nurse implement first? 1. Tell the client that it is important for her to take the medication 2. Find out how the client has been dealing with the pain 3. Have the HCP tell the client to take the medication 4. Instruct the client not to worry the pain will resolve itself

2. Find out how the client has been dealing with the pain. The nurse should asses the situation fully. The client may be afraid for becoming addicted or may have been using alternative forms of treatment, such as music therapy, distraction techniques, acupuncture, or guide imagery

28. The nurse writes the nursing problem of "fluid volume excess" (FVE). Which intervention should be included in the plan of care? 1. Change the IV fluid from 0.9% NS to D5W. 2. Restrict the client's sodium in the diet. 3. Monitor blood glucose levels. 4. Prepare the client for hemodialysis

2. Fluid volume excess refers to an isotonic expansion of the extracellular fluid by an abnormal expansion of water and sodium. Therefore sodium is restricted to allow the body to excrete the extra volume

The nurse is developing a nursing care plan for a client diagnosed with congestive heart failure. A nursing diagnosis of "decreased cardiac output related to inability of the heart to pump effectively" is written. Which short-term goal would be best for the client? The client will: 1. Be able to ambulate in the hall by date of discharge. 2. Have an audible S1 and S2 with no S3 heard by end of shift. 3. Turn, cough, and deep breathe every two (2) hours. 4. Have a pulse oximeter reading of 98% by day two (2) of care.

2. Have an audible S1 and S2 with no S3 heard by end of shift. 1. Ambulating in the hall by day of discharge would be a more appropriate goal for an activity-intolerance nursing diagnosis. 2. Audible S1 and S2 sounds are normal for a heart with adequate output. An audible S3 sound might indicate left ventricular failure which could be life threatening. 3. This is a nursing intervention, not a short-term goal, for this client. 4. A pulse oximeter reading would be a goal for impaired gas exchange, not for cardiac output. TEST-TAKING HINT: When reading a nursing diagnosis or problem, the test taker must be sure that the answer selected addresses the problem. An answer option may be appropriate care for the disease process but may not fit with the problem or etiology. Remember, when given an etiology in a nursing diagnosis, the answer will be doing something about the problem (etiology). In this question the test taker should look for an answer that addresses the ability of the heart to pump blood.

50. The nurse observes red urine and several large clots in the tubing of the normal saline continuous irrigation catheter for the client who is 1 day postoperative TURP. Which intervention should the nurse implement? 1. Remove the indwelling catheter. 2. Titrate the NS irrigation to run faster. 3. Administer protamine sulfate IVP. 4. Administer vitamin K slowly.

2. Increasing the irrigation fluid will flush out the clots and blood.

The client has been in a persistent vegetative state for several years. The family, who has decided to withhold tube feeding because there is no hope of recovery, asks the nurse, "Will death be painful?" Which intervention should the nurse implement? 1. Tell the family that the death will be painful but the HCP can order medications 2. Inform the family that dehydration provides a form of euphoria 3. Relate other cases where the clients have died in excruciating pain 4. Ask the family why they are concerned because they want the client to die anyway

2. Inform the family that dehydration provides a form of euphoria. Death from dehydration occurs when the client is unable to take in fluid. A natural euphoria occurs with dehydration. This is the body's way of allowing comfort at the time of death.

6. The nurse suspects the client admitted with a near-drowning is developing acute respiratory distress syndrome (ARDS). Which data support the nurse's suspicion? 1. The client's arterial blood gases are within normal limits. 2. The client appears anxious, has dyspnea, and is tachypneic. 3. The client has intercostal retractions and is using accessory muscles. 4. The client's bilateral lung sounds have crackles and rhonchi.

2. Initial clinical manifestations of ARDS usually develop 24 to 48 hours after the initial insult leading to hypoxia and include anxiety, dyspnea, and tachypnea.

38. The nurse is inserting an indwelling catheter into a female client. Which interventions should be implemented? Select all that apply. 1. Explain the procedure to the significant other. 2. Set up the sterile field. 3. Inflate the catheter bulb. 4. Place absorbent pads under the client. 5. Clean the perineum from clean to dirty with Betadine.

2. Inserting an indwelling catheter is a sterile procedure. 3. The bulb of the catheter should be tested to make sure it will inflate and deflate prior to inserting the catheter into the client. 4. Incontinence pads should be placed under the client before beginning the sterile part of the procedure. 5. During the procedure the perineum is swiped with Betadine swabs from front to back and also down the middle, then side to side with new swabs (clean to dirty).

68. The 84-year-old female client is admitted with multiple burn marks on the torso and under the breasts along with contusions in various stages of healing. When questioned by the nurse, the woman denies any problems have occurred. The woman lives with her son and does the housework. Which is the most probable reason the woman denies being abused? 1. There has not been any abuse to report. 2. The client is ashamed to admit being abused. 3. The client has Alzheimer's disease and can't remember. 4. The client has engaged in consensual sex.

2. Many times the elderly are ashamed to report abuse because they raised the abuser and feel responsible for their child becoming an abuser. The elder parent may feel financially dependent on the child or be afraid of being placed in a long-term care facility. Forty-seven states have Adult Protective Services (APS) created by the states to protect elder citizens. 1. This client has signs of ongoing abuse such as multiple burns and contusions in different stages of healing. 3. There is no evidence provided in the stem of the client is not being mentally competent, and there is evidence in the stem of physical abuse. This client is performing activities of daily living. 4. Consensual sex does not involve the physical abuse noted in the assessment. TEST-TAKING HINT: The test taker could eliminate options "1," "3," and "4" by examining the stem and noting the physical abuse occurring, and by the fact the client is functioning by performing activities of daily living.

64. The nurse is discharging a client diagnosed with diabetes insipidus. Which statement made by the client warrants further intervention? 1. "I will keep a list of my medications in my wallet and wear a Medic Alert bracelet." 2. "I should take my medication in the morning and leave it refrigerated at home." 3. "I should weigh myself every morning and record any weight gain." 4. "If I develop a tightness in my chest, I will call my health-care provider."

2. Medication for DI is usually taken every 8 to 12 hours, depending on the client. The client should keep the medication close at hand.

The client diagnosed with congestive heart failure is complaining of leg cramps at night. Which nursing interventions should be implemented? 1. Check the client for peripheral edema and make sure the client takes a diuretic early in the day. 2. Monitor the client's potassium level and assess the client's intake of bananas and orange juice. 3. Determine if the client has gained weight and instruct the client to keep the legs elevated. 4. Instruct the client to ambulate frequently and perform calf-muscle stretching exercises daily.

2. Monitor the client's potassium level and assess the client's intake of bananas and orange juice. 1. The client with peripheral edema will experience calf tightness but would not have leg cramping, which is the result of low potassium levels. The timing of the diuretic will not change the side effect of leg cramping resulting from low potassium levels. 2. The most probable cause of the leg cramping is potassium excretion as a result of diuretic medication. Bananas and orange juice are foods that are high in potassium. 3. Weight gain is monitored in clients with CHF, and elevating the legs would decrease peripheral edema by increasing the rate of return to the central circulation, but these interventions would not help with leg cramps. 4. Ambulating frequently and performing leg-stretching exercises will not be effective in alleviating the leg cramps. TEST-TAKING HINT: The timing "at night" in this question was not important in answering the question, but it could have made the test taker jump at option "1." Be sure to read all answer options before deciding on an answer. Answering this question correctly requires knowledge of the side effects of treatments used for CHF.

28. The client diagnosed with cancer is experiencing severe pain. Which regimen would the nurse teach the client about to control the pain? 1. Nonsteroidal anti-inflammatory drugs (NSAIDs) around the clock with narcotics used for severe pain. 2. Morphine sustained release, a narcotic, routinely with a liquid morphine preparation for breakthrough pain. 3. Extra-Strength Tylenol, a nonnarcotic analgesic, plus therapy to learn alternative methods of pain control. 4. Demerol, an opioid narcotic, every six (6) hours orally with a suppository when the pain is not controlled.

2. Morphine is the drug of choice for cancer pain. There is no ceiling effect, it metabolizes without harmful byproducts, and it is relatively inexpensive. A sustained-release formulation, such as MS Contin, is administered every six (6) to eight (8) hours, and a liquid fast-acting form is administered sublingually for any pain which is not controlled.

15. Which assessment data indicate to the nurse the client diagnosed with Legionnaires' disease is experiencing a complication? 1. The client has an elevated body temperature. 2. The client has <30 mL urine output an hour. 3. The client has a decrease in body aches. 4. The client has an elevated white blood cell count.

2. Multiple organ failure is a common complication of Legionnaires' disease. Renal failure should be suspected as a complication if the client does not have a urine output of 30 mL/hr.

72. The client with infective endocarditis is admitted to the medical department. Which health-care provider's order should be implemented first? 1. Administer intravenous antibiotic. 2. Obtain blood cultures times two (2). 3. Schedule an echocardiogram. 4. Encourage bedrest with bathroom privileges.

2. Obtain blood cultures times two (2). 1. The nurse must obtain blood cultures prior to administering antibiotics. 2. Blood cultures must be done before administering antibiotics so that an adequate number of organisms can be obtained to culture and identify. 3. An echocardiogram allows visualization of vegetations and evaluation of valve function. However, antibiotic therapy is priority before diagnostic tests, and blood cultures must be obtained before administering medication. 4. Bedrest should be implemented, but the first intervention should be obtaining blood cultures so that antibiotic therapy can be started as soon as possible. TEST-TAKING HINT: The test taker must identify the first of the HCP's orders to be implemented. "Infective" should indicate that this is an infection, which requires antibiotics, but the nurse should always assess for allergies and obtain cultures prior to administering any antibiotic.

18. The client diagnosed with cancer is unable to attain pain relief despite receiving large amounts of narcotic medications. Which intervention should be included in the plan of care? 1. Ask the HCP to increase the medication. 2. Assess for any spiritual distress. 3. Change the client's position every two (2) hours. 4. Turn on the radio to soothing music.

2. Pain has many components, and spiritual distress or psychosocial needs will affect the client's perception of pain; remember, assessment is the first step of the nursing process

19. The client diagnosed with chronic pain is laughing and joking with visitors. When the nurse asks the client to rate the pain on a 1-to-10 scale, the client rates the pain as 10. According to the pain scale, how would the nurse chart the client's pain? 1. The client's pain is between a zero (0) and two (2) on the faces scale. 2. The client's pain is a "10" on a 1-to-10 pain scale. 3. The client is unable to accurately rate the pain on a scale. 4. The client's pain is moderate on the pain scale. CHAPTER 18 END-OF-LIFE ISSUES 703 0 No hurt 0 1 Hurts little bit 2 2 Hurts little more 4 3 Hurts even more 6 4 Hurts whole lot 8 5 Hurts worst 10 0 No Pain Unbearable Pain Severe Pain Moderate Pain Mild Pain 1 2 3 4 5 6 7 8 9 10 Alternate coding

2. Pain is whatever the client says it is and occurs whenever the client says it does. Pain is a wholly subjective symptom, and the nurse should not question the client's perception of pain. The client's pain is a 10.

51. The client diagnosed with septicemia expired, and the family tells the nurse the client is an organ donor. Which intervention should the nurse implement? 1. Notify the organ and tissue organizations to make the retrieval. 2. Explain a systemic infection prevents the client from being a donor. 3. Call and notify the health-care provider of the family's request. 4. Take the body to the morgue until the organ bank makes a decision.

2. Septicemia is a systemic infection and will prevent the client from donating tissues or organs.

48. The client diagnosed with chronic pyelonephritis is given a prescription for Bactrim, trimethoprim sulfa, a sulfa antibiotic, twice a day for 90 days. Which statement is the scientific rationale for prescribing this medication? 1. The antibiotic will treat the bladder spasms that accompany a urinary tract infection. 2. If the urine cannot be made bacteria free, the Bactrim will suppress bacterial growth. 3. In three (3) months the client should be rid of all bacteria in the urinary tract. 4. The HCP is providing the client with enough medication to treat future infections.

2. Some clients develop a chronic infection and must receive antibiotic therapy as a routine daily medication to suppress the bacterial growth. The prescription will be refilled after the 90 days and continued.

The nurse is discussing angina with a client who is diagnosed with coronary artery disease. Which action should the client take first when experiencing angina? 1. Put a nitroglycerin tablet under the tongue. 2. Stop the activity immediately and rest. 3. Document when and what activity caused angina. 4. Notify the health-care provider immediately.

2. Stop the activity immediately and rest. 1. The client should take the coronary vasodilator nitroglycerin sublingually, but it is not the first intervention. 2. Stopping the activity decreases the heart's need for oxygen and may help decrease the angina (chest pain). 3. The client should keep a diary of when angina occurs, what activity causes it, and how many nitroglycerin tablets are taken before chest pain is relieved. 4. If the chest pain (angina) is not relieved with three (3) nitroglycerin tablets, the client should call 911 or have someone take him to the emergency department. Notifying the HCP may take too long. TEST-TAKING HINT: The question is asking which action the client should take first. This implies that more than one of the answer options could be appropriate for the chest pain, but that only one is done first. The test taker should select the answer that will help the client directly and quickly—and that is stopping the activity.

The nurse is developing a discharge-teaching plan for the client diagnosed with congestive heart failure. Which interventions should be included in the plan? Select all that apply. 1. Notify health-care provider of a weight gain of more than one (1) pound in a week. 2. Teach client how to count the radial pulse when taking digoxin, a cardiac glycoside. 3. Instruct client to remove the saltshaker from the dinner table. 4. Encourage client to monitor urine output for change in color to become dark. 5. Discuss the importance of taking the loop diuretic furosemide at bedtime.

2. Teach client how to count the radial pulse when taking digoxin, a cardiac glycoside. 1. The client should notify the HCP of weight gain of more than two (2) or three (3) pounds in one (1) day. 2. The client should not take digoxin if the radial pulse is less than 60. 3. The client should be on a low-sodium diet to prevent water retention. 4. The color of the urine should not change to a dark color; if anything, it might become lighter and the amount will increase with diuretics. 5. Instruct the client to take the diuretic in the morning to prevent nocturia. TEST-TAKING HINT: This is an alternativetype question—in this case, "select all that apply." If the test taker missed this statement, it is possible to jump at the first correct answer. This is one reason that it is imperative to read all options before deciding on the correct one(s). This could be a clue to reread the question for clarity. Another hint that this is an alternative question is the number of options. The other questions have four potential answers; this one has five. Numbers in an answer option are always important. Is one (1) enough pounds to indicate a problem that should be brought to the attention of the health-care provider?

The health-care provider has ordered an angiotensin-converting enzyme (ACE) inhibitor for the client diagnosed with congestive heart failure. Which discharge instructions should the nurse include? 1. Instruct the client to take a cough suppressant if a cough develops. 2. Teach the client how to prevent orthostatic hypotension. 3. Encourage the client to eat bananas to increase potassium level. 4. Explain the importance of taking the medication with food.

2. Teach the client how to prevent orthostatic hypotension. 1. If a cough develops, the client should notify the health-care provider because this is an adverse reaction and the HCP will discontinue the medication. 2. Orthostatic hypotension may occur with ACE inhibitors as a result of vasodilation. Therefore, the nurse should instruct the client to rise slowly and sit on the side of the bed until equilibrium is restored. 3. ACE inhibitors may cause the client to retain potassium; therefore, the client should not increase potassium intake. 4. An ACE inhibitor should be taken one (1) hour before meals or two (2) hours after a meal to increase absorption of the medication. TEST-TAKING HINT: If the test taker knows that an ACE inhibitor is also given for hypertension, then looking at answer options referring to hypotension would be appropriate.

37. The nurse is preparing to administer warfarin (Coumadin), an oral anticoagulant, to a client diagnosed with a pulmonary embolus. Which data would cause the nurse to question administering the medication? 1. The client's partial thromboplastin time (PTT) is 38. 2. The client's international normalized ratio (INR) is 5. 3. The client's prothrombin time (PT) is 22. 4. The client's erythrocyte sedimentation rate (ESR) is 10.

2. The INR therapeutic range is 2 to 3 for a client receiving warfarin. The INR may be allowed to go to 3.5 if the client has a mechanical cardiac valve, but nothing in the stem of the question indicates this.

37. The nurse is assessing a client with complaints of vague upper abdominal pain worse at night but relieved by sitting up and leaning forward. Which assessment question should the nurse ask next? 1. "Have you noticed a yellow haze when you look at things?" 2. "Does the pain get worse when you eat a meal or snack?" 3. "Have you had your amylase and lipase checked recently?" 4. "How much weight have you gained since you saw an HCP?"

2. The abdominal pain is often made worse by eating and lying supine in clients diagnosed with cancer of the pancreas.

52. The client is admitted to rule out Cushing's syndrome. Which laboratory tests should the nurse anticipate being ordered? 1. Plasma drug levels of quinidine, digoxin, and hydralazine. 2. Plasma levels of ACTH and cortisol. 3. A 24-hour urine for metanephrine and catecholamine. 4. Spot urine for creatinine and white blood cells.

2. The adrenal gland secretes cortisol and the pituitary gland secretes adrenocorticotropic hormone (ACTH), a hormone used by the body to stimulate the production of cortisol.

The nurse is preparing to administer a beta blocker to the client diagnosed with coronary artery disease. Which assessment data would cause the nurse to question administering the medication? 1. The client has a BP of 110/70. 2. The client has an apical pulse of 56. 3. The client is complaining of a headache. 4. The client's potassium level is 4.5 mEq/L.

2. The client has an apical pulse of 56. 1. This blood pressure is normal and the nurse would administer the medication. 2. A beta blocker decreases sympathetic stimulation to the heart, thereby decreasing the heart rate. An apical rate less than 60 indicates a lower-thannormal heart rate and should make the nurse question administering this medication because it will further decrease the heart rate. 3. A headache will not affect administering the medication to the client. 4. The potassium level is within normal limits, but it is usually not monitored prior to administering a beta blocker. TEST-TAKING HINT: If the test taker does not know when to question the use of a certain medication, the test taker should evaluate the options to determine if any options include abnormal data based on normal parameters. This would make the test taker select option "2" because the normal apical pulse in an adult is 60 to 100.

34. The client has received IV solutions for three (3) days through a 20-gauge IV catheter placed in the left cephalic vein. On morning rounds the nurse notes the IV site is tender to palpation and a red streak has formed. Which action should the nurse implement first? 1. Start a new IV in the right hand. 2. Discontinue the intravenous line. 3. Complete an incident record. 4. Place a warm washrag over the site.

2. The client has signs of phlebitis and the IV must be removed to prevent further complications.

45. The client had a total pancreatectomy and splenectomy for cancer of the body of the pancreas. Which discharge instructions should the nurse teach? Select all that apply. 1. Keep a careful record of intake and output. 2. Use a stool softener or bulk laxative regularly. 3. Use correct insulin injection technique. 4. Take the pain medication before the pain gets too bad. 5. Sleep with the head of the bed on blocks.

2. The client has undergone a radical and extensive surgery and will need narcotic pain medication, and a bowel regimen should be in place to prevent constipation. 3. Removal of the pancreas will create a diabetic state for the client. The client will need insulin and pancreatic enzyme replacement. 4. The client should not allow pain to reach above a "5" before taking pain medication or it will be more difficult to get the pain under control.

15. Which electrolyte replacement should the nurse anticipate being ordered by the health-care provider in the client diagnosed with DKA who has just been admitted to the ICU? 1. Glucose. 2. Potassium. 3. Calcium. 4. Sodium.

2. The client in DKA loses potassium from increased urinary output, acidosis, catabolic state, and vomiting. Replacement is essential for preventing cardiac dysrhythmias secondary to hypokalemia.

31. The male client who has made himself a do not resuscitate (DNR) order is in pain. The client's vital signs are P 88, R 8, and BP 108/70. Which intervention should be the nurse's priority action? 1. Refuse to give the medication because it could kill the client. 2. Administer the medication as ordered and assess for relief from pain. 3. Wait until the client' respirations improve and then administer the medication. 4. Notify the HCP the client is unstable and pain medication is being held.

2. The client is in pain. The American Nurses Association Code of Ethics states clients have the right to die as comfortably as possible even if the measures used to control the pain indirectly hasten the impending death. The Dying Client's Bill of Rights reiterates this position. The client should be allowed to die with dignity and with as much comfort as the nurse can provide.

25. The nurse is preparing the discharge teaching plan for the male client with a left-sided nephrectomy. Which statement indicates the teaching is effective? 1. "I can't wait to start back to work next week, I really need the money." 2. "I will take my temperature and if it is above 101 I will call my doctor." 3. "I am glad I won't have to keep track of how much I urinate in the day." 4. "I am happy I will be able eat what I usually eat, I don't like this food."

2. The client or family needs to contact the surgeon if the client develops chills, flank pain, decreased urinary output, or fever.

The client with coronary artery disease is prescribed transdermal nitroglycerin, a coronary vasodilator. Which behavior indicates the client understands the discharge teaching concerning this medication? 1. The client places the medication under the tongue. 2. The client removes the old patch before placing the new one. 3. The client applies the patch to a hairy area. 4. The client changes the patch every 36 hours.

2. The client removes the old patch before placing the new one. This behavior indicates the client understands the discharge teaching.

12. Which statement indicates the client with a total laryngectomy requires more teaching concerning the care of the tracheostomy? 1. "I must avoid hair spray and powders." 2. "I should take a shower instead of a tub bath." 3. "I will need to cleanse around the stoma daily." 4. "I can use an electric larynx to speak."

2. The client should not allow water to enter the stoma; therefore, the client should take a tub bath, not a shower.

The charge nurse is making shift assignments. Which client would be most appropriate for the charge nurse to assign to a new graduate who just completed orientation to the medical floor? 1. The client admitted for diagnostic tests to rule out valvular heart disease. 2. The client three (3) days post-myocardial infarction being discharged tomorrow. 3. The client exhibiting supraventricular tachycardia (SVT) on telemetry. 4. The client diagnosed with atrial fibrillation who has an INR of five (5).

2. The client three (3) days post-myocardial infarction being discharged tomorrow. 1. This client requires teaching and an understanding of the preprocedure interventions for diagnostic tests; therefore, a more experienced nurse should be assigned to this client. 2. Because this client is being discharged, it would be an appropriate assignment for the new graduate. 3. Supraventricular tachycardia (SVT) is not life threatening, but the client requires intravenous medication and close monitoring and therefore should be assigned to a more experienced nurse. 4. A client with atrial fibrillation is usually taking the anticoagulant warfarin (Coumadin), and the therapeutic INR is 2 to 3. An INR of 5 is high and the client is at risk for bleeding. TEST-TAKING HINT: The test taker must realize that a new graduate must be assigned the least critical client. Remember, teaching is a primary responsibility of the nurse; physical care is not always the criterion that should be used when making client assignments.

19. The UAP on the medical floor tells the nurse the client diagnosed with DKA wants something else to eat for lunch. Which intervention should the nurse implement? 1. Instruct the UAP to get the client additional food. 2. Notify the dietitian about the client's request. 3. Request the HCP increase the client's caloric intake. 4. Tell the UAP the client cannot have anything else.

2. The client will not be compliant with the diet if he or she is still hungry. Therefore, the nurse should request the dietitian talk to the client to try to adjust the meals so the client will adhere to the diet.

18. Which assessment data indicate the client diagnosed with diabetic ketoacidosis is responding to the medical treatment? 1. The client has tented skin turgor and dry mucous membranes. 2. The client is alert and oriented to date, time, and place. 3. The client's ABG results are pH 7.29, PaCO2 44, HCO3 15. 4. The client's serum potassium level is 3.3 mEq/L.

2. The client's level of consciousness can be altered because of dehydration and acidosis. If the client's sensorium is intact, the client is getting better and responding to the medical treatment.

28. Which clinical manifestation indicates to the nurse the child has cystic fibrosis? 1. Wheezing with a productive cough. 2. Excessive salty sweat secretions. 3. Multiple vitamin deficiencies. 4. Clubbing of all fingers.

2. The excessive excretion of salt from the sweat glands is specific to cystic fibrosis. Repeated values greater than 60 mEq/L of sweat chloride is diagnostic for CF.

72. The nurse writes a nursing diagnosis of "risk for injury as a result of physical abuse by spouse" for a client. Which is an appropriate goal for this client? 1. The client will learn not to trust anyone. 2. The client will admit the abuse is happening and get help. 3. The client will discuss the nurse's suspicions with the spouse. 4. The client will choose to stay with the spouse.

2. The first step in helping a client who has been abused is to get the client to admit the abuse is happening. 1. The nurse should attempt to develop a relationship in which the client feels he or she can trust the nurse (males are abused by significant others too). 3. This could cause the abuse to escalate. 4. This is what the nurse is trying to get the client to avoid. TEST-TAKING HINT: Option "1" could be eliminated because it is the opposite of what the nurse tries to establish in a nurse-client relationship. Option "4" places the client in harm's way.

41. The client diagnosed with cancer of the pancreas is being discharged to start chemotherapy in the HCP's office. Which statement made by the client indicates the client understands the discharge instructions? 1. "I will have to see the HCP every day for six (6) weeks for my treatments." 2. "I should write down all my questions so I can ask them when I see the HCP." 3. "I am sure this is not going to be a serious problem for me to deal with." 4. "The nurse will give me an injection in my leg and I will get to go home."

2. The most important person in the treatment of the cancer is the client. Research has proved the more involved a client becomes in his or her care, the better the prognosis. Clients should have a chance to ask questions.

35. The nurse is caring for a client on a ventilator and the alarm goes off. Which action should the nurse implement first? 1. Notify the respiratory therapist immediately. 2. Check the ventilator to determine the cause. 3. Elevate the head of the client's bed. 4. Assess the client's oxygen saturation.

2. The nurse must determine what is causing the alarm; a high or low alarm will make a difference in the nurse's action.

62. The elderly male client is admitted to the medical unit with a diagnosis of senile dementia. The client is 74 inches tall and weighs 54.5 kg. The client lives with his son and daughter-in-law, both of whom work outside the house. Which referral is most important for the nurse to implement? 1. Adult Protective Services. 2. Social worker. 3. Medicare ombudsman. 4. Dietitian.

2. The nurse should arrange for the social worker to see the client and family to determine if some arrangements could be made to provide for the client's safety and for the client to be provided with nutritious meals while the adult children are at work. A long-term care facility or adult day care may be needed. 1. Adult protective services should be called only if it is determined willful neglect or abuse of the client is occurring. 3. The Medicare ombudsman is a person who represents a Medicare client in a long-term care facility. 4. The dietitian could see this client to determine eating preferences (74 inches = 6 foot 2 inches and 54.5 kg = 120 pounds), but the most appropriate intervention is safety. TEST-TAKING HINT: The question asks for the test taker to determine a priority intervention. The client is diagnosed with senile dementia and is being left alone for hours of the day. Safety is priority.

36. The female client in the oncology clinic tells the nurse she has a great deal of pain but does not like to take pain medication. Which action should the nurse implement first? 1. Tell the client it is important for her to take her medication. 2. Find out how the client has been dealing with the pain. 3. Have the HCP tell the client to take the pain medications. 4. Instruct the client not to worry—the pain will resolve itself.

2. The nurse should assess the situation fully. The client may be afraid of becoming addicted or may have been using alternative forms of treatment, such as music therapy, distraction techniques, acupuncture, or guided imagery

38. The nurse is caring for a client diagnosed with a pneumothorax who had chest tubes inserted four (4) hours ago. There is no fluctuating (tidaling) in the water-seal compartment of the closed chest drainage system. Which action should the nurse implement first? 1. Milk the chest tube. 2. Check the tubing for kinks. 3. Instruct the client to cough. 4. Assess the insertion site.

2. The nurse should implement the least invasive intervention first. The nurse should check to see if the tubing is kinked, causing a blockage between the pleural space and the water-seal bottle.

8. The Hispanic client who has terminal cancer is requesting a curandero to come to the bedside. Which intervention should the nurse implement? 1. Tell the client it is against policy to allow faith healers. 2. Assist with planning the visit from the curandero. 3. Refer the client to the pastoral care department. 4. Determine the reason the client needs the curandero.

2. The nurse should support the client's culture as long as it is not contraindicated in the client's care. This client is terminal; therefore, allowing the curandero, who is a folk healer and religious person in the Hispanic culture, would be appropriate.

62. The nurse is admitting a client to the neurological intensive care unit who is postoperative transsphenoidal hypophysectomy. Which data warrant immediate intervention? 1. The client is alert to name but is unable to tell the nurse the location. 2. The client has an output of 2,500 mL since surgery and an intake of 1,000 mL . 3. The client's vital signs are T 97.6ºF, P 88, R 20, and BP 130/80. 4. The client has a 3-cm amount of dark-red drainage on the turban dressing.

2. The output is more than double the intake in a short time. This client could be developing diabetes insipidus, a complication of trauma to the head.

64. Which clinical manifestations would the nurse expect to assess for the client diagnosed with a ureteral renal stone? 1. Dull, aching flank pain and microscopic hematuria. 2. Nausea; vomiting; pallor; and cool, clammy skin. 3. Gross hematuria and dull suprapubic pain with voiding. 4. No symptoms.

2. The severe flank pain associated with a stone in the ureter often causes a sympathetic response with associated nausea; vomiting; pallor; and cool, clammy skin.

13. The intensive care nurse is caring for a deceased client who is an organ donor, and the organ donation team is in route to the hospital. Which statement would be an appropriate goal of treatment for the client? 1. The urinary output is 20 mL/hr via a Foley catheter. 2. The systolic blood pressure is greater than 90 mm Hg. 3. The pulse oximeter reading remains between 88% and 90%. 4. The telemetry shows the client in sinus tachycardia.

2. The systolic blood pressure must be maintained at this rate to keep the client's organs perfused until removal.

6. The nurse is preparing the plan of care for the client diagnosed with a neurogenic flaccid bladder. Which expected outcome is appropriate for this client? 1. The client has conscious control over bladder activity. 2. The client's bladder does not become overdistended. 3. The client has bladder sensation and no discomfort. 4. The client demonstrates how to check for bladder distention.

2. The treatment goal of the flaccid bladder is to prevent overdistention.

38. The triage nurse is working in the emergency department. Which client should be assessed first? 1. The 10-year-old child whose dad thinks the child's leg is broken. 2. The 45-year-old male who is diaphoretic and clutching his chest. 3. The 58-year-old female complaining of a headache and seeing spots. 4. The 25-year-old male who cut his hand with a hunting knife.

2. The triage nurse should see this client first because these are symptoms of a myocardial infarction, which is potentially life threatening. 1. The child needs an x-ray to confirm the fracture, but the client is stable and does not have a life-threatening problem. 3. These are symptoms of a migraine headache and are not life threatening. 4. A laceration on the hand is priority, but not over a client having a myocardial infarction. TEST-TAKING HINT: The test taker should evaluate each option on a scale of one to ten, with one being the least critical client and 10 being life threatening. Option "2" rates a score of 10.

31. The client post-thyroidectomy complains of numbness and tingling around the mouth and the tips of the fingers. Which intervention should the implement first? 1. Notify the health-care provider immediately. 2. Tap the cheek about two (2) cm anterior to the ear lobe. 3. Check the serum calcium and magnesium levels. 4. Prepare to administer calcium gluconate IVP

2. These are signs and symptoms of hypocalcemia, and the nurse can confirm this by tapping the cheek to elicit the Chvostek's sign. If the muscles of the cheek begin to twitch, then the HCP should be notified immediately because hypocalcemia is a medical emergency.

11. The unlicensed nursing assistant tells the nurse that the client with ARF has a white layer on top of the skin that looks like crystals. Which intervention should the nurse implement? 1. Have the assistant apply a moisture barrier cream to the skin. 2. Instruct the nursing assistant to bathe the client in cool water. 3. Tell the nursing assistant not to turn the client in this condition. 4. Explain that this is normal and do not do anything to the client.

2. These crystals are uremic frost resulting from irritating toxins deposited in the client's tissues. Bathing in cool water will remove the crystals, promote client comfort, and decrease the itching that occurs from uremic frost.

42. The nurse in a disaster is triaging the following clients. Which client should be triaged as an Expectant Category, priority 4, and color black? 1. The client with a sucking chest wound who is alert. 2. The client with a head injury who is unresponsive. 3. The client with an abdominal wound and stable vital signs. 4. The client with a sprained ankle which may be fractured.

2. This client has a very poor prognosis, and even with treatment, survival is unlikely. 1. This client should be classified as an Immediate Category, Priority 1, and color red. If not treated STAT, a tension pneumothorax will occur. 3. This client should be classified as a Delayed Category, Priority 2, and color yellow. This client receives treatment after the casualties requiring immediate treatment are treated. 4. This client is a Minimal Category, Priority 3, and color green. This client can wait days for treatment. TEST-TAKING HINT: If the test taker did not know the definition of the categories, looking at the word "black," which has a connotation of death, and the word "expectant" might lead the test taker to select the worst-case scenario.

14. The nurse in the dialysis center is initiating the morning dialysis run. Which client should the nurse assess first? 1. The client who has hemoglobin of 9.8 mg/dL and hematocrit of 30%. 2. The client who does not have a palpable thrill or auscultated bruit. 3. The client who is complaining of being exhausted and is sleeping. 4. The client who did not take antihypertensive medication this morning.

2. This client's dialysis access is compromised and should be assessed first.

1. The 38-year-old client was brought to the emergency department with CPR in progress and expired 15 minutes after arrival. Which intervention should the nurse implement for postmortem care? 1. Do not allow significant others to see the body. 2. Do not remove any tubes from the body. 3. Prepare the body for the funeral home. 4. Send the client's clothing to the hospital laundry.

2. This death should be reported to the medical examiner because the death occurred less than 24 hours after hospital admission and an autopsy may be required. Therefore, the nurse must leave all tubes in place; the medical examiner will remove the tubes.

44. The client diagnosed with cancer of the head of the pancreas is two (2) days postpancreatoduodenectomy (Whipple's procedure). Which nursing problem has the highest priority? 1. Anticipatory grieving. 2. Fluid volume imbalance. 3. Alteration in comfort. 4. Altered nutrition.

2. This is a major abdominal surgery, and massive fluid volume shifts occur when this type of trauma is experienced by the body. Maintaining the circulatory system without overloading it requires extremely close monitoring.

11. Which situation would cause the nurse to question the validity of an AD when caring for the elderly client? 1. The client's child insists the client make his or her own decisions. 2. The nurse observes the wife making the husband sign the AD. 3. A nurse encouraged the client to think about end-of-life decisions. 4. A friend witnesses the client's signature on the AD form.

2. This is coercion and is illegal when signing an AD. The AD must be signed by the client's own free will; an AD signed under duress may not be valid.

The client has just signed an AD at the bedside. Which intervention should the nurse implement first? 1. Notify the client's health-care provider about the AD. 2. Instruct the client to discuss the AD with significant others. 3. Place a copy of the advance directive in the client's chart. 4. Give the original advance directive to the client.

2. This is the most important intervention because the legality of the document is sometimes not honored if the family members disagree and demand other action. If the client's family is aware of the client's wishes, then the health-care team can sup

7. The client has just signed an AD at the bedside. Which intervention should the nurse implement first? 1. Notify the client's health-care provider about the AD. 2. Instruct the client to discuss the AD with significant others. 3. Place a copy of the advance directive in the client's chart. 4. Give the original advance directive to the client.

2. This is the most important intervention because the legality of the document is sometimes not honored if the family members disagree and demand other action. If the client's family is aware of the client's wishes, then the health-care team can support and honor the client's final wishes.

32. The charge nurse is making assignments on an oncology floor. Which client should be assigned to the most experienced nurse? 1. The client diagnosed with leukemia who has a hemoglobin of 6.0 g/dL. 2. The client diagnosed with lung cancer with a pulse oximeter reading of 89%. 3. The client diagnosed with colon cancer who needs the colostomy irrigated. 4. The client diagnosed with Kaposi's sarcoma who is yelling at the staff.

2. This represents an arterial blood gas of less than 60%; this client should be assigned to the most experienced nurse

23. The nurse is preparing a teaching care plan for the client diagnosed with nephrotic syndrome. Which intervention should the nurse include? 1. Stop steroids if a moon face develops. 2. Provide teaching for taking diuretics. 3. Increase the intake of dietary sodium. 4. Report a decrease in daily weight.

2. Treatment includes diuretics to eliminate dependent edema, usually in the ankles and sacrum. Medication teaching is an appropriate intervention.

1. An 18-year-old female client, 5′4′′tall, weighing 113 kg, comes to the clinic for a nonhealing wound on her lower leg, which she has had for two (2) weeks. Which disease process should the nurse suspect the client has developed? 1. Type 1 diabetes. 2. Type 2 diabetes. 3. Gestational diabetes. 4. Acanthosis nigricans.

2. Type 2 diabetes is a disorder usually occurring around the age of 40, but it is now being detected in children and young adults as a result of obesity and sedentary lifestyles. Nonhealing wounds are a hallmark sign of type 2 diabetes. This client weighs 248.6 pounds and is short.

37. The nurse is teaching an in-service on legal issues in nursing. Which situation is an example of battery, an intentional tort? 1. The nurse threatens the client who is refusing to take a hypnotic medication. 2. The nurse forcibly inserts a Foley catheter in a client who refused it. 3. The nurse tells the client a nasogastric tube insertion is not painful. 4. The nurse gives confidential information over the telephone.

2. When a mentally competent adult is forced to have a treatment he or she has refused, battery occurs.

9. The nurse is caring for an elderly client who has an indwelling catheter. Which data warrant further investigation? 1. The client's temperature is 98.0˚F. 2. The client has become confused and irritable. 3. The client's urine is clear and light yellow. 4. The client feels the need to urinate.

2. When an elderly client's mental status changes to confused and irritable, the nurse should seek the etiology, which may be a UTI secondary to an indwelling catheter. Elderly clients often do not present with classic signs and symptoms of infection.

53. The client has developed iatrogenic Cushing's disease. Which statement is the scientific rationale for the development of this diagnosis? 1. The client has an autoimmune problem causing the destruction of the adrenal cortex. 2. The client has been taking steroid medications for an extended period for another disease process. 3. The client has a pituitary gland tumor causing the adrenal glands to produce too much cortisol. 4. The client has developed an adrenal gland problem for which the health-care provider does not have an explanation.

2."Iatrogenic" means a problem has been caused by a medical treatment or procedure—in this case, treatment with steroids for another problem. Clients taking steroids over a period of time develop the clinical manifestations of Cushing's disease. Disease processes for which long-term steroids are prescribed include chronic obstructive pulmonary disease, cancer, and arthritis.

20. The nurse is preparing the plan of care for a client with fluid volume deficit. Which interventions should the nurse include in the plan of care? Select all that apply. 1. Monitor vital signs every two (2) hours until stable. 2. Measure the client's oral intake and urinary output daily. 3. Administer mouth care when bathing the client. 4. Weigh the client weekly in the same clothing at the same time. 5. Assess skin turgor and mucous membranes every shift.

20. 1. Vital signs should be monitored every two (2) hours until stable and more frequently if the client is unstable. 5. Skin turgor and mucous membranes should be assessed every shift or more often depending on the client's condition.

4

20. The client diagnosed with diabetes mellitus type 2 wants to be an organ donor and asks the nurse, "Which organs can I donate?" Which statement is the nurse's best response? 1. "It is wonderful you want to be an organ donor. Let's discuss this." 2. "You can donate any organ in your body, except the pancreas." 3. "You have to donate your body to science to be an organ donor." 4. "You cannot donate any organs, but you can donate some tissues."

1

20. The client has expired secondary to smallpox. Which information about funeral arrangements is most important for the nurse to provide to the client's family? 1. The client should be cremated. 2. Suggest an open casket funeral. 3. Bury the client within 24 hours. 4. Notify the public health department.

1

20. The nurse is discharging a client from the ED with a sutured laceration on the right knee. Which information is most important for the nurse to obtain? 1. The date of the client's last tetanus injection. 2. The name of the client's regular health-care provider. 3. Explain the sutures must be removed in 10 to 14 days. 4. Determine if the client has any drug or food allergies.

2

21. A chemical exposure has just occurred at an airport. An off-duty nurse, knowledgeable about biochemical agents, is giving directions to the travelers. Which direction should the nurse provide to the travelers? 1. Hold their breath as much as possible. 2. Stand up to avoid heavy exposure. 3. Lie down to stay under the exposure. 4. Attempt to breathe through their clothing.

4

21. The client with multiple sclerosis who is becoming very debilitated tells the home health nurse the Hemlock Society sent information on euthanasia. Which question should the nurse ask the client? 1. "Why did you get in touch with the Hemlock Society?" 2. "Did you know this is an illegal organization?" 3. "Who do you know who has committed suicide?" 4. "What religious beliefs do you practice?"

2

21. The nurse working in an outpatient clinic is caring for a client who is experiencing epistaxis. Which intervention should the nurse implement first? 1. Take the client's blood pressure in both arms. 2. Hold the nose with thumb and finger for 15 minutes. 3. Have the client sit with the head tilted back and hold a tissue. 4. Prepare to administer silver nitrate, a cauterizing agent, with a packing applicator

3

22. The nurse is caring for a client in the prodromal phase of radiation exposure. Which signs/symptoms should the nurse assess in the client? 1. Anemia, leukopenia, and thrombocytopenia. 2. Sudden fever, chills, and enlarged lymph nodes. 3. Nausea, vomiting, and diarrhea. 4. Flaccid paralysis, diplopia, and dysphagia.

4

22. Which intervention should the nurse implement to provide culturally sensitive health care to the European-American Caucasian elderly client who is terminal? 1. Discuss health-care issues with the oldest male child. 2. Determine if the client will be cremated or have an earth burial. 3. Do not talk about death and dying in front of the client. 4. Encourage the client's autonomy and answer questions truthfully.

23. The nurse is caring for the client diagnosed with bacterial pneumonia. Which priority intervention should the nurse implement? 1. Assess respiratory rate and depth. 2. Provide for adequate rest period. 3. Administer oxygen as prescribed. 4. Teach slow abdominal breathing.

23. 1. The assessment of respiratory rate and depth is the priority intervention because tachypnea and dyspnea may be early indicators of respiratory compromise.

3

23. The ED nurse is caring for the client who has taken an overdose of cocaine. Which intervention should the nurse delegate to the unlicensed assistive personnel (UAP)? 1. Evaluate the airway and breathing. 2. Monitor the rate of intravenous fluids. 3. Place the cardiac monitor on the client. 4. Transfer the client to the intensive care unit.

1,2,3,4

23. Which cultural issues should the nurse consider when caring for clients during a bioterrorism attack? Select all that apply. 1. Language difficulties. 2. Religious practices. 3. Prayer times for the people. 4. Rituals for handling the dead. 5. Keeping the family in the designated area.

24. Which intervention is most important for the nurse to implement for the client with a left nephrectomy? 1. Assess the intravenous fluids for rate and volume. 2. Change surgical dressing every day at the same time. 3. Monitor the client's PT/PTT/INR level daily. 4. Monitor the percentage of each meal eaten.

24. 1. Assessing the rate and volume of intravenous fluid is the most important intervention for the client who has one (1) kidney because an overload of fluids can result in pulmonary edema.

24. The client is admitted to the ICU diagnosed with DKA. Which interventions should the nurse implement? Select all that apply. 1. Maintain adequate ventilation. 2. Assess fluid volume status. 3. Administer intravenous potassium. 4. Check for urinary ketones. 5. Monitor intake and output.

24. 1. The nurse should always address the airway when a client is seriously ill. 2. The client must be assessed for fluid volume deficit and then for fluidvolume excess after fluid replacement is started. 3. The electrolyte imbalance of primary concern is depletion of potassium. 4. Ketones are excreted in the urine; levels are documented from negative to large amount. Ketones should be monitored frequently. 5. The nurse must ensure the client's fluid intake and output are equal. TEST-TAKING HINT: The test taker must select all answer options that apply. Do not try to outguess the item writer. In some instances all options are correct.

3

24. The client has been brought to the ED by ambulance following a motor-vehicle accident with a flail chest, an intravenous line, and a Heimlich valve. Which intervention should the nurse implement first? 1. Start a large-bore intravenous access. 2. Request a portable chest x-ray. 3. Prepare to insert chest tubes. 4. Assess the cardiac rhythm on the monitor.

4

24. The off-duty nurse hears on the television of a bioterrorism act in the community. Which action should the nurse take first? 1. Immediately report to the hospital emergency room. 2. Call the American Red Cross to find out where to go. 3. Pack a bag and prepare to stay at the hospital. 4. Follow the nurse's hospital policy for responding.

1

24. Which action should the nurse implement for the Chinese client's family who are requesting to light incense around the dying client? 1. Suggest the family bring potpourri instead of incense. 2. Tell the client the door must be shut at all times. 3. Inform the family the scent will make the client nauseated. 4. Explain fire code does not allow any burning in a hospital.

3

25. The ED nurse is completing the initial assessment on a client who becomes unresponsive. Which intervention should the nurse implement first? 1. Assess the rate and site of the intravenous fluid. 2. Administer an ampule of sodium bicarbonate. 3. Assess the cardiac rhythm shown on the monitor. 4. Prepare to cardiovert the client into sinus rhythm.

3

25. The nurse is caring for the client who has active tuberculosis of the lungs. The client does not have a DNR order. The client experiences a cardiac arrest, and there is no resuscitation mask at the bedside. The nurse waits for the crash cart before beginning resuscitation. According to the ANA Code of Ethics for Nurses (see below), which disciplinary action should be taken against the nurse? 1. Report the action to the State Board of Nurse Examiners. 2. The nurse should be terminated for failure to perform duties. 3. No disciplinary action should be taken against the nurse. 4. Refer the nurse to the American Nurses Association.

26. The client diagnosed with a fluid and electrolyte disturbance in the emergency department is exhibiting peaked T waves on the STAT electrocardiogram. Which interventions should the nurse implement? List in order of priority. 1. Assess the client for leg and muscle cramps. 2. Check the serum potassium level. 3. Notify the health-care provider. 4. Arrange for a transfer to the telemetry floor. 5. Administer Kayexalate, a cation resin.

26. In order of priority: 1, 2, 3, 5, 4. 1. The nurse should assess to determine if the client is symptomatic of hyperkalemia. 2. A peaked T wave is indicative of hyperkalemia; therefore, the nurse should obtain a potassium level. 3. Hyperkalemia is a life-threatening situation because of the risk of cardiac dysrhythmias; therefore, the nurse should notify the health-care provider. 5. Kayexalate is a medication that will help remove potassium through the gastrointestinal system and should be administered to decrease the potassium level. 4. The client should be monitored continuously for cardiac dysrhythmias, so a transfer to the telemetry unit is warranted.

4,2,3,1,5

26. The ED nurse is caring for a female client with a greenstick fracture of the left forearm and multiple contusions on the face, arms, trunk, and legs. The significant other is in the treatment area with the client. Which nursing interventions should the nurse implement? List in order of priority. 1. Determine if the client has a plan for safety. 2. Assess the pulse, temperature, and capillary refill of the left wrist and hand. 3. Ask the client if she feels safe in her own home. 4. Request the significant other wait in the waiting room during the examination. 5. Notify the social worker to consult on the case.

4,3,2,1,5

26. The wife of a client receiving hospice care being cared for at home calls the nurse to report the client is restless and agitated. Which interventions should the nurse implement? List in order of priority. 1. Request an order from the health-care provider for antianxiety medications. 2. Call the medical equipment company and request oxygen for the client. 3. Go to the home and assess the client and address the wife's concerns. 4. Reassure and calm the wife over the telephone. 5. Notify the chaplain about the client's change in status.

27. Which information should the nurse include in the teaching plan for the mother of a child diagnosed with cystic fibrosis (CF)? Select all that apply. 1. Perform postural drainage and percussion every four (4) hours. 2. Modify activities to accommodate daily physiotherapy. 3. Increase fluid intake to one (1) liter daily to thin secretions. 4. Recognize and report signs and symptoms of respiratory infections. 5. Avoid anyone suspected of having an upper respiratory infection.

27. 1. Clients and family members should be taught chest physiotherapy, including postural drainage, chest percussion, and vibration and breathing techniques to keep the lungs clear of the copious secretions. 2. Daily activities should be modified to accommodate the client's treatments. 4. Clients should be taught the signs and symptoms of infections to report to the health-care provider. 5. Clients with CF are susceptible to respiratory infections and should avoid anyone who is suspected of having an infection.

30. The client diagnosed with intractable pain is receiving an IV constant infusion of morphine, a narcotic opioid. The concentration is 50 mg of morphine in 250 mL of normal saline. The IV is infusing at 10 mL/hr. The client has required bolus administration of two (2) mg IVP × two (2) during the 12-hour shift. How much morphine has the client received during the shift? _________

28 mg of morphine. First, determine how many milligrams of morphine are in each milliliter of saline: 50 ÷ 250 mL = 0.2 mg/mL Then determine how many milliliters are given in a shift: 10 mL/hr × 12 hour = 120 mL infused 1 shift = 120 mL infused If each milliliter contains 0.2 milligram of morphine, then 0.2 mg × 120 mL = 24 mg by constant infusion Then determine the amount given IVP: 2 × 2 = 4 mg given IVP Finally, add the bolus amount to the amount constantly infused: 24 + 4 = 28 mg

Which client would most likely be misdiagnosed for having a myocardial infarction? 1. A 55-year-old Caucasian male with crushing chest pain and diaphoresis. 2. A 60-year-old Native American male with an elevated troponin level. 3. A 40-year-old Hispanic female with a normal electrocardiogram. 4. An 80-year-old Peruvian female with a normal CK-MB at 12 hours.

3. A 40-year old Hispanic female with a normal electrocardiogram The clients who are misdiagnosed concerning MIs usually present with atypical symptoms. They tend to be female, be younger than 55 years old, be members of a minority group, and have normal electrocardiograms.

49. Which intervention should the nurse implement first when administering the first dose of intravenous antibiotic to the client diagnosed with a respiratory infection? 1. Monitor the client's current temperature. 2. Monitor the client's white blood cells. 3. Determine if a culture has been collected. 4. Determine the compatibility of fluids.

3. A culture needs to be collected prior to the first dose of antibiotic, or the culture and sensitivity will be skewed and the appropriate antibiotic needed to treat the respiratory infection may not be identified.

70. The client with a history of renal calculi calls the clinic and reports having burning on urination, chills, and an elevated temperature. Which instruction should the nurse discuss with the client? 1. Increase water intake for the next 24 hours. 2. Take two Tylenol to help decrease the temperature. 3. Come to the clinic and give a urinalysis specimen. 4. Use a sterile 4x4 gauze to strain the client's urine.

3. A urinalysis can assess for hematuria (red blood cells in the urine), the presence of white blood cells, crystal fragments, or all three, which can determine if the client has a urinary tract infection or possibly a renal stone, with accompanying signs/symptoms of UTI.

32. Which collaborative intervention should the nurse implement when caring for the client diagnosed with bronchiectasis? 1. Prepare the client for an emergency tracheostomy. 2. Discuss postoperative teaching for a lobectomy. 3. Administer bronchodilators with postural drainage. 4. Obtain informed consent form for chest tube insertion.

3. Administering bronchodilators is a collaborative intervention (requiring an order from a health-care provider) appropriate for this client.

21. The nurse is preparing the plan of care for the client who had a pleurodesis. Which collaborative intervention should the nurse include? 1. Monitor the amount and color of drainage from the chest tube. 2. Perform a complete respiratory assessment every two (2) hours. 3. Administer morphine sulfate, an opioid analgesic, intravenously. 4. Keep a sterile dressing and bottle of sterile normal saline at the bedside.

3. Administering medication is a collaborative intervention because it requires a health-care provider's order.

24. The nurse is caring for a client diagnosed with pneumonia who is having shortness of breath and difficulty breathing. Which intervention should the nurse implement first? 1. Take the client's vital signs. 2. Check the client's pulse oximeter reading. 3. Administer oxygen via a nasal cannula. 4. Notify the respiratory therapist STAT.

3. After elevating the head of the bed, the nurse should administer oxygen to the client who is in respiratory difficulty.

56. The nurse writes a problem of "altered body image" for a 34-year-old client diagnosed with Cushing's disease. Which intervention should be implemented? 1. Monitor blood glucose levels prior to meals and at bedtime. 2. Perform a head-to-toe assessment on the client every shift. 3. Use therapeutic communication to allow the client to discuss feelings. 4. Assess bowel sounds and temperature every four (4) hours.

3. Allowing the client to ventilate feelings about the altered body image is the most appropriate intervention. The nurse cannot do anything to help the client's buffalo hump or moon face.

40. The client diagnosed with a cold is taking an antihistamine. Which statement indicates to the nurse the client needs more teaching concerning the medication? 1. "If my mouth gets dry I will suck on hard candy." 2. "I will not drink beer or any type of alcohol." 3. "I need to be careful when I drive my car." 4. "This medication will make me sleepy."

3. Antihistamines cause drowsiness, so the client should not drive or operate any type of machinery.

The client diagnosed with pericarditis is complaining of increased pain. Which intervention should the nurse implement first? 1. Administer oxygen via nasal cannula. 2. Evaluate the client's urinary output. 3. Assess the client for cardiac complications. 4. Encourage the client to use the incentive spirometer.

3. Assess the client for cardiac complications. 1. Oxygen may be needed, but it is not the first intervention. 2. This would be appropriate to determine if the urine output is at least 30 mL/hr, but it is not the first intervention. 3. The nurse must assess the client to determine if the pain is expected secondary to pericarditis or if the pain is indicative of a complication that requires intervention from the health-care provider. 4. Using the incentive spirometer will increase the client's alveolar ventilation and help prevent atelectasis, but it is not the first intervention. TEST-TAKING HINT: The test taker must apply the nursing process when determining the correct answer and select the option that addresses the first step in the nursing process—assessment.

The client is exhibiting ventricular tachycardia. Which intervention should the nurse implement first? 1. Administer lidocaine, an antidysrhythmic, IVP. 2. Prepare to defibrillate the client. 3. Assess the client's apical pulse and blood pressure. 4. Start basic cardiopulmonary resuscitation.

3. Assess the client's apical pulse and blood pressure. 1. Lidocaine is the drug of choice for ventricular tachycardia, but it is not the first intervention. 2. Defibrillation may be needed, but it is not the first intervention. 3. The nurse must assess the apical pulse and blood pressure to determine if the client is in cardiac arrest and then treat as ventricular fibrillation. If the client's heart is beating, the nurse would then administer lidocaine. 4. CPR is only performed on a client who is not breathing and does not have a pulse. The nurse must establish if this is occurring first, prior to taking any other action. TEST-TAKING HINT: When the stem asks the test taker to select the first intervention, all answer options could be plausible interventions, but only one is implemented first. The test taker should use the nursing process to answer the question and select the intervention that addresses assessment, which is the first step in the nursing process.

The male client requested a DNR per the AD, and the HCP wrote the order. The client's death is imminent and the client's wife tells the nurse, "Help him please. Do something. I am not ready to let him go." Which action should the nurse take? 1. Ask the wife if she would like to revoke her husband's AD. 2. Leave the wife at the bedside and notify the hospital chaplain. 3. Sit with the wife at the bedside and encourage her to say good-bye. 4. Request the client to tell the wife he is ready to die, and don't do anything.

3. At the time of death, loved ones become scared and find it difficult to say good-bye. The nurse should support the client's decision and acknowledge the wife's psychological state. Research states hearing is the last sense to go, and talking to the dying client is therapeutic for the client and the family

10. The male client requested a DNR per the AD, and the HCP wrote the order. The client's death is imminent and the client's wife tells the nurse, "Help him please. Do something. I am not ready to let him go." Which action should the nurse take? 1. Ask the wife if she would like to revoke her husband's AD. 2. Leave the wife at the bedside and notify the hospital chaplain. 3. Sit with the wife at the bedside and encourage her to say good-bye. 4. Request the client to tell the wife he is ready to die, and don't do anything.

3. At the time of death, loved ones become scared and find it difficult to say good-bye. The nurse should support the client's decision and acknowledge the wife's psychological state. Research states hearing is the last sense to go, and talking to the dying client is therapeutic for the client and the family.

49. The nurse is admitting a client diagnosed with primary adrenal cortex insufficiency (Addison's disease). Which clinical manifestations should the nurse expect to assess? 1. Moon face, buffalo hump, and hyperglycemia. 2. Hirsutism, fever, and irritability. 3. Bronze pigmentation, hypotension, and anorexia. 4. Tachycardia, bulging eyes, and goiter.

3. Bronze pigmentation of the skin, particularly of the knuckles and other areas of skin creases, occurs in Addison's disease. Hypotension and anorexia also occur with Addison's disease.

The client shows ventricular fibrillation on the telemetry at the nurse's station. Which action should the telemetry nurse implement first? 1. Administer epinephrine IVP. 2. Prepare to defibrillate the client. 3. Call a STAT code. 4. Start cardiopulmonary resuscitation.

3. Call a STAT code. 1. There are many interventions that should be implemented prior to administering medication. 2. The treatment of choice for ventricular fibrillation is defibrillation, but it is not the first action. 3. The nurse must call a code that activates the crash cart being brought to the room and a team of health-care providers that will care for the client according to an established protocol. 4. The first person at the bedside should start cardiopulmonary resuscitation (CPR), but the telemetry nurse should call a code so that all necessary equipment and personnel are at the bedside. TEST-TAKING HINT: The test taker must realize that ventricular fibrillation is life threatening and immediate action must be implemented. Remember, when the question asks "first," all options could be appropriate interventions but only one should be implemented first.

7. The client diagnosed with ARF is admitted to the intensive care department and placed on a therapeutic diet. Which diet would be most appropriate for the client? 1. A high-potassium and low-calcium diet. 2. A low-fat and low-cholesterol diet. 3. A high-carbohydrate and restricted-protein diet. 4. A regular diet with six (6) small feedings a day.

3. Carbohydrates are increased to provide for the client's caloric intake and protein is restricted to minimize protein breakdown and to prevent accumulation of toxic end products.

The client who has had a myocardial infarction is admitted to the telemetry unit from intensive care. Which referral would be most appropriate for the client? 1. Social worker. 2. Physical therapy. 3. Cardiac rehabilitation. 4. Occupational therapy.

3. Cardiac rehabilitation. 1. The social worker addresses financial concerns or referrals after discharge, which are not indicated for this client. 2. Physical therapy addresses gait problems, lower extremity strength building, and assisting with transfer, which are not required for this client. 3. Cardiac rehabilitation is the most appropriate referral. The client can start rehabilitation in the hospital and then attend an outpatient cardiac rehabilitation clinic, which includes progressive exercise, diet teaching, and classes on modifying risk factors. 4. Occupational therapy assists the client in regaining activities of daily living and covers mainly fine motor activities. TEST-TAKING HINT: The test taker must be familiar with the responsibilities of the other members of the health-care team. If the test taker had no idea which would be the most appropriate referral, the word "cardiac," which means "heart," should help the test taker in deciding that this is the most sensible option because the client had a myocardial infarction, a "heart attack."

16. The hospice care nurse is conducting a spiritual care assessment. Which statement is the scientific rationale for this intervention? 1. The client will ask all of his or her spiritual questions and get answers. 2. The nurse is able to explain to the client how death will affect the spirit. 3. Spirituality provides a sense of meaning and purpose for many clients. 4. The nurse is the expert when assisting the client with spiritual matters.

3. Clients facing death may wish to find meaning and purpose in life through a higher power. This gives the clients hope, even if the life on earth will be temporary

53. The client received a liver transplant and is preparing for discharge. Which discharge instruction should the nurse teach? 1. The immune-suppressant drugs must be tapered off when discontinuing them. 2. There may be slight foul-smelling drainage on the dressing for a few days. 3. Notify the HCP immediately if a cough or fever develops. 4. The skin will turn yellow from the anti-rejection drugs.

3. Clients should be taught to notify the HCP immediately of any signs of an infection. The immune-suppressant drugs will mask the sign of an infection and superinfections can develop

The telemetry nurse is unable to read the telemetry monitor at the nurse's station. Which intervention should the telemetry nurse implement first? 1. Go to the client's room to check the client. 2. Instruct the primary nurse to assess the client. 3. Contact the client on the client call system. 4. Request the nursing assistant to take the crash cart to the client's room.

3. Contact the client on the client call system. 1. The telemetry nurse should not leave the monitors unattended at any time. 2. The telemetry nurse must have someone go assess the client, but this is not the first intervention. 3. If the client answers the call light and is not experiencing chest pain, then there is probably a monitor artifact, which is not a life-threatening emergency. After talking with the client, send a nurse to the room to check the monitor. 4. The crash cart should be taken to a room when the client is experiencing a code. TEST-TAKING HINT: When the test taker sees the word "first," the test taker must realize that more than one answer option may be a possible intervention but that only one should be implemented first. The test taker should try to determine which intervention directly affects the client.

26. The client who has undergone an exploratory laparotomy and subsequent removal of a large intestinal tumor has a nasogastric tube (NGT) in place and an IV running at 150 mL/hr via an IV pump. Which data should be reported to the HCP? 1. The pump keeps sounding an alarm that the high pressure has been reached. 2. Intake is 1800 mL, NGT output is 550 mL, and Foley output 950 mL. 3. On auscultation, crackles and rales in all lung fields are noted. 4. Client has negative pedal edema and an increasing level of consciousness.

3. Crackles and rales in all lung fields indicate that the body is not able to process the amounts of fluids being infused. This should be brought to the HCP's attention.

2. The primary nurse caring for the client who died is crying with the family at the bedside. Which action should the charge nurse implement? 1. Request the primary nurse to come out in the hall. 2. Refer the nurse to the employee assistance program. 3. Allow the nurse and family this time to grieve. 4. Ask the chaplain to relieve the nurse at the bedside.

3. Crying was once considered unprofessional, but today it is recognized as simply an expression of empathy and caring

Which preprocedure information should be taught to the female client having an exercise stress test in the morning? 1. Wear open-toed shoes to the stress test. 2. Inform the client not to wear a bra. 3. Do not eat anything for four (4) hours. 4. Take the beta blocker one (1) hour before the test.

3. Do not eat anything for four hours. NPO decreases the chance of aspiration in case of emergency. In addition, if the client has just had a meal, the blood sup- ply will be shunted to the stomach for digestion and away from the heart, per- haps leading to an inaccurate test result.

The nurse is discussing the importance of exercise with the client diagnosed with coronary artery disease. Which intervention should the nurse implement? 1. Perform isometric exercises daily. 2. Walk for 15 minutes three (3) times a week. 3. Do not walk outside if it is less than 40˚F. 4. Wear open-toed shoes when ambulating.

3. Do not walk outside if it is less than 40˚F. 1. Isometric exercises are weight lifting-type exercises. A client with CAD should perform isotonic exercises, which increase muscle tone, not isometric exercises. 2. The client should walk at least 30 minutes a day to increase collateral circulation. 3. When it is cold outside, vasoconstriction occurs, and this will decrease oxygen to the heart muscle. Therefore, the client should not exercise when it is cold outside. 4. The client should wear good supportive tennis shoes when ambulating, not sandals or other open-toed shoes. TEST-TAKING HINT: The test taker should be aware of adjectives such as "isometric," which makes option "1" incorrect, and "open-toed," which makes option "4" incorrect.

2. Which statement indicates the client diagnosed with asthma needs more teaching concerning the medication regimen? 1. "I will take Singulair, a leukotriene, every day to prevent allergic asthma attacks." 2. "I need to use my Intal, cromolyn, inhaler 15 minutes before I begin my exercise." 3. "I need to take oral glucocorticoids every day to prevent my asthma attacks." 4. "If I have an asthma attack, I need to use my Albuterol, a beta 2 agonist, inhaler."

3. Glucocorticoids are administered orally or intravenously during acute exacerbations of asthma, not on a daily basis because of the long-term complications of steroid therapy.

The client diagnosed with rule-out myocardial infarction is experiencing chest pain while walking to the bathroom. Which action should the nurse implement first? 1. Administer sublingual nitroglycerin. 2. Obtain a STAT electrocardiogram. 3. Have the client sit down immediately. 4. Assess the client's vital signs.

3. Have the client sit down immediately. 1. The nurse must assume the chest pain is secondary to decreased oxygen to the myocardium and administer a sublingual nitroglycerin tablet, which is a coronary vasodilator, but this is not the first action. 2. An ECG should be ordered, but it is not the first intervention. 3. Stopping all activity will decrease the need of the myocardium for oxygen and may help decrease the chest pain. 4. Assessment is often the first nursing intervention, but when the client has chest pain and a possible MI, the nurse must first take care of the client. Taking vital signs would not help relieve chest pain. TEST-TAKING HINT: Whenever the test taker wants to select an assessment intervention, be sure to think about whether that intervention will help the client, especially if the client is experiencing pain. Do not automatically select the answer option that is assessment.

10. Which clinical manifestation would the nurse assess in the client newly diagnosed with intrinsic lung cancer? 1. Dysphagia. 2. Foul-smelling breath. 3. Hoarseness. 4. Weight loss.

3. Hoarseness is an early clinical manifestation of intrinsic lung cancer. "Intrinsic" means the tumor is on the vocal cord.

21. The nurse is discussing ways to prevent diabetic ketoacidosis with the client diagnosed with type 1 diabetes. Which instruction is most important to discuss with the client? 1. Refer the client to the American Diabetes Association. 2. Do not take any over-the-counter medications. 3. Take the prescribed insulin even when unable to eat because of illness. 4. Explain the need to get the annual flu and pneumonia vaccines.

3. Illness increases blood glucose levels; therefore, the client must take insulin and consume high-carbohydrate foods such as regular Jell-O, regular popsicles, and orange juice.

The client with pericarditis is prescribed a nonsteroidal anti-inflammatory drug (NSAID). Which teaching instruction should the nurse discuss with the client? 1. Explain the importance of tapering off the medication. 2. Discuss that the medication will make the client drowsy. 3. Instruct the client to take the medication with food. 4. Tell the client to take the medication when the pain level is around "8."

3. Instruct the client to take the medication with food. 1. Steroids, such as prednisone, not NSAIDs, must be tapered off to prevent adrenal insufficiency. 2. NSAIDs will not make clients drowsy. 3. NSAIDs must be taken with food, milk, or antacids to help decrease gastric distress. NSAIDs reduce fever, inflammation, and pericardial pain. 4. NSAIDs should be taken regularly around the clock to help decrease inflammation, which, in turn, will decrease pain. TEST-TAKING HINT: The test taker must remember NSAIDs and steroids cause gastric distress to the point of causing peptic ulcer disease. These medications are administered for a variety of conditions and diseases.

14. Which data are significant when assessing a client diagnosed with rule-out Legionnaires' disease? 1. The amount of cigarettes smoked a day and the age when started. 2. Symptoms of aching muscles, high fever, malaise, and coughing. 3. Exposure to a saprophytic water bacterium transmitted into the air. 4. Decreased bilateral lung sounds in the lower lobes.

3. Legionnaires' disease is caused by a saprophytic water bacterium that is transmitted through the air from places where these bacteria are found: rivers, lakes, evaporative condensers, respiratory apparatuses, or water distribution centers.

39. The nurse is planning a program for clients at a health fair regarding the prevention and early detection of cancer of the pancreas. Which self-care activity should the nurse discuss which is an example of a primary nursing intervention? 1. Monitor for elevated blood glucose at random intervals. 2. Inspect the skin and sclera of the eyes for a yellow tint. 3. Limit meat in the diet and eat a diet low in fat. 4. Instruct the client with hyperglycemia about insulin injections.

3. Limiting the intake of meat and fats in the diet is an example of primary interventions. Risk factors for the development of cancer of the pancreas are cigarette smoking and eating a high-fatdiet. By changing these behaviors, the client could possibly prevent the development of cancer of the pancreas. Other risk factors include genetic predisposition and exposure to industrial chemicals. 4. Instructing

22. The charge nurse is making client assignments in the intensive care unit. Which client should be assigned to the most experienced nurse? 1. The client with type 2 diabetes who has a blood glucose level of 348 mg/dL. 2. The client diagnosed with type 1 diabetes who is experiencing hypoglycemia. 3. The client with DKA who has multifocal premature ventricular contractions. 4. The client with HHNS who has a plasma osmolarity of 290 mOsm/L.

3. Multifocal PVCs, which are secondary to hypokalemia and can occur in clients with DKA, are a potentially life-threatening emergency. This client needs an experienced nurse.

66. The nurse is caring for clients on a medical floor. Which client should be assessed first? 1. The client diagnosed with syndrome of inappropriate antidiuretic hormone (SIADH) who has a weight gain of 1.5 pounds since yesterday. 2. The client diagnosed with a pituitary tumor who has developed diabetes insipidus (DI) and has an intake of 1,500 mL and an output of 1,600 mL in the last 8 hours. 3. The client diagnosed with syndrome of inappropriate antidiuretic hormone (SIADH) who is having muscle twitching. 4. The client diagnosed with diabetes insipidus (DI) who is complaining of feeling tired after having to get up at night.

3. Muscle twitching is a sign of early sodium imbalance. If an immediate intervention is not made, the client could begin to seize.

31. The nurse is discussing complications of chronic pancreatitis with a client diagnosed with the disease. Which complication should the nurse discuss with the client? 1. Diabetes insipidus . 2. Crohn's disease. 3. Narcotic addiction. 4. Peritonitis.

3. Narcotic addiction is related to the frequent, severe pain episodes often occurring with chronic pancreatitis which require narcotics for relief.

10. The male client asks the nurse, "Should I designate my wife as durable power of attorney for health care?" Which statement would be the nurse's best response? 1. "Yes, she should be because she is your next of kin." 2. "Most people don't allow their spouse to do this." 3. "Will your wife be able to support your wishes?" 4. "Your children are probably the best ones for the job."

3. No matter who the client selects as the power of attorney, the most important aspect is to make sure the person, whether it be the wife, child, or friend, will honor the client's wishes no matter what happens.

26. The client had a mastectomy and lymph node dissection three (3) years ago and has experienced postmastectomy pain (PMP) since. Which intervention should the nurse implement? 1. Have the client see a psychologist because the pain is not real. 2. Tell the client the pain is the cancer coming back. 3. Refer the client to a physical therapist to prevent a frozen shoulder. 4. Discuss changing the client to a more potent narcotic medication.

3. PMP is characterized as a constriction accompanied by a burning sensation or prickling in the chest wall, axilla, or posterior arm resulting from movement of the arm. Because of this, the client limits movement of the arm and the shoulder becomes frozen.

27. The male client diagnosed with chronic pain since a construction accident which broke several vertebrae tells the nurse he has been referred to a pain clinic and asks, "What good will it do? I will never be free of this pain." Which statement is the nurse's best response? 1. "Are you afraid of the pain never going away?" 2. "The pain clinic will give you medication to cure the pain." 3. "Pain clinics work to help you achieve relief from pain." 4. "I am not sure. You should discuss this with your HCP."

3. Pain clinics use a variety of methods to help the client to achieve relief from pain. Some measures include guided imagery, transcutaneous electrical nerve stimulation (TENS) units, nerve block surgery or injections, or medications.

36. The client diagnosed with deep vein thrombosis (DVT) suddenly complains of severe chest pain and a feeling of impending doom. Which complication should the nurse suspect the client has experienced? 1. Myocardial infarction. 2. Pneumonia. 3. Pulmonary embolus. 4. Pneumothorax.

3. Part of the clot in the deep veins of the legs dislodges and travels up the inferior vena cava, lodges in the pulmonary artery, and causes the chest pain; the client often feels as if he or she is going to die.

8. The nurse is planning the activities for the client diagnosed with asbestosis. Which activity should the nurse schedule at 0900 if breakfast is served at 0800? 1. Assist with the client's bath and linen change. 2. Administer an inhalation bronchodilator treatment. 3. Provide the client with a one (1)-hour rest period. 4. Have respiratory therapy perform chest physiotherapy.

3. Periods of rest should be alternated with periods of activity.

The client diagnosed with a myocardial infarction is on bedrest. The unlicensed assistive personnel (UAP) is encouraging the client to move the legs. Which action should the nurse implement? 1. Instruct the UAP to stop encouraging the leg movements. 2. Report this behavior to the charge nurse as soon as possible. 3. Praise the UAP for encouraging the client to move legs. 4. Take no action concerning the UAP's behavior.

3. Praise the UAP for encouraging the client to move legs. 1. Leg movement is an appropriate action, and the UAP should not be told to stop encouraging it. 2. This behavior is not unsafe or dangerous and should not be reported to the charge nurse. 3. The nurse should praise and encourage UAPs to participate in the client's care. Clients on bedrest are at risk for deep vein thrombosis, and moving the legs will help prevent this from occurring. 4. The nurse should praise subordinates for appropriate behavior, especially when it is helping to prevent life-threatening complications. TEST-TAKING HINT: This is a management question. The test taker must know the chain of command and when to report behavior. The test taker could eliminate options "1" and "2" with the knowledge that moving the legs is a safe activity for the client. When having to choose between options "3" and "4," the test taker should select doing something positive, instead of taking no action. This is a management concept.

The client is exhibiting sinus bradycardia, is complaining of syncope and weakness, and has a BP of 98/60. Which collaborative treatment should the nurse anticipate being implemented? 1. Administer a thrombolytic medication. 2. Assess the client's cardiovascular status. 3. Prepare for insertion of a pacemaker. 4. Obtain a permit for synchronized cardioversion.

3. Prepare for insertion of a pacemaker. 1. A thrombolytic medication is administered for a client experiencing a myocardial infarction. 2. Assessment is an independent nursing action, not a collaborative treatment. 3. The client is symptomatic and will require a pacemaker. 4. Synchronized cardioversion is used for ventricular tachycardia with a pulse or atrial fibrillation. TEST-TAKING HINT: The key to answering this question is the adjective "collaborative," which means the treatment requires obtaining a health-care provider's order or working with another member of the health-care team. This would cause the test taker to eliminate option "2" as a possible correct answer.

66. The nurse is teaching a class about rape prevention to a group of women at a community center. Which information is not a myth about rape? 1. Women who are raped asked for it by dressing provocatively. 2. If a woman says no, it is a come on and she really does not mean it. 3. Rape is an attempt to exert power and control over the client. 4. All victims of sexual assault are women; men can't be raped.

3. Rape is an act of violence motivated by the rapist desiring to overpower and control the victim. 1. This is a myth believed by some people. Many individuals are raped, ranging in age from infants to the 90s, male and female, heterosexuals and homosexuals. No one asks to be raped. 2. If a person says he or she is not interested in any type of sexual activity, it means "no" and anything else is forced and it is rape. "No" means "no." It is considered rape if a prostitute says "no." 4. Men and children can be victims of rape. Sexual arousal and orgasm do not imply consent; it may be a pathological response to stimulation. TEST-TAKING HINT: This is an "except" question, which means three of the options will contain correct information. In this question, there are three false statements about rape; this is a double negative type of question.

6. The nurse is developing a plan of care for a client diagnosed with ARF. Which statement would be an appropriate outcome for the client? 1. Monitor intake and output every shift. 2. Decrease of pain by 3 levels on a 1-10 scale. 3. Electrolytes are within normal limits. 4. Administer enemas to decrease hyperkalemia.

3. Renal failure causes an imbalance of electrolytes (potassium, sodium, calcium, phosphorus). Therefore the desired client outcome would be that all the electrolytes are within normal limits.

56. The client is three (3) hours post-heart transplantation. Which data would support a complication of this procedure? 1. The client has nausea after taking the oral anti-rejection medication. 2. The client has difficulty coming off the heart-lung bypass machine. 3. The client has saturated three (3) ABD dressing pads in one (1) hour. 4. The client complains of pain at a "6" on a 1-to-10 scale.

3. Saturating three (3) dressing pads in one (1) hour would indicate hemorrhage.

Pancreatitis 25. The client is admitted to the medical department with a diagnosis of R/O acute pancreatitis. Which laboratory values should the nurse monitor to confirm this diagnosis? 1. Creatinine and BUN. 2. Troponin and CK-MB. 3. Serum amylase and lipase. 4. Serum bilirubin and calcium.

3. Serum amylase increases within two (2) to 12 hours of the onset of acute pancreatitis to two (2) to three (3) times normal and returns to normal in three (3) to four (4) days; lipase elevates and remains elevated for seven (7) to 14 days.

29. The nurse is completing discharge teaching to the client diagnosed with acute pancreatitis. Which instruction should the nurse discuss with the client? 1. Instruct the client to decrease alcohol intake. 2. Explain the need to avoid all stress. 3. Discuss the importance of stopping smoking. 4. Teach the correct way to take pancreatic enzymes.

3. Smoking stimulates the pancreas to release pancreatic enzymes and should be stopped.

The charge nurse is making assignments for clients on a cardiac unit. Which client should the charge nurse assign to a new graduate nurse? 1. The 44-year-old client diagnosed with a myocardial infarction. 2. The 65-year-old client admitted with unstable angina. 3. The 75-year-old client scheduled for a cardiac catheterization. 4. The 50-year-old client complaining of chest pain.

3. The 75-year-old client scheduled for a cardiac catheterization. 1. This client is at high risk for complications related to necrotic myocardial tissue and will need extensive teaching, so this client should not be assigned to a new graduate. 2. Unstable angina means this client is at risk for life-threatening complications and should not be assigned to a new graduate. 3. A new graduate should be able to complete a preprocedure checklist and get this client to the catheterization laboratory. 4. Chest pain means this client could be having a myocardial infarction and should not be assigned to a new graduate. TEST-TAKING HINT: "New graduate" is the key to answering this question correctly. What type of client should be assigned to an inexperienced nurse? The test taker should not assign the new graduate a client who is unstable or at risk for a lifethreatening complication. Content - Medical: Category of Health Alteration - Cardiovascular: Integrated Nursing Process - Planning: Client Needs - Safe Effective Care Environment, Management of Care: Cognitive Level - Synthesis.

25. The nurse is caring for the client who has active tuberculosis of the lungs. The client does not have a DNR order. The client experiences a cardiac arrest, and there is no resuscitation mask at the bedside. The nurse waits for the crash cart before beginning resuscitation. According to the ANA Code of Ethics for Nurses (see below), which disciplinary action should be taken against the nurse? 1. Report the action to the State Board of Nurse Examiners. 2. The nurse should be terminated for failure to perform duties. 3. No disciplinary action should be taken against the nurse. 4. Refer the nurse to the American Nurses Association.

3. The Code states, "The nurse owes the same duty to self as to others, including the responsibility to preserve integrity and safety." Therefore, if the nurse realizes he or she could contract TB if unprotected mouth-to-mouth resuscitation is performed, then not doing this action does not violate the Code of Ethics.

34. The client diagnosed with chronic back pain is being placed on a transcutaneous electrical nerve stimulation (TENS) unit. Which information should the nurse teach? 1. The TENS unit will deaden the nerve endings, and the client will not feel pain. 2. The TENS unit could cause paralysis if the client gets the unit wet. 3. The TENS unit stimulates the nerves in the area, blocking the pain sensation. 4. The TENS unit should be left on for an hour, and then taken off for an hour.

3. The TENS unit works on the gate control theory of pain control and works by flooding the area with stimulation and blocking the pain impulses from reaching the brain.

The client diagnosed with chronic back pain is being placed on a TENS unit. Which information should the nurse teach? 1. The TENs will deaden the nerve endings, and the client will not feel pain 2. The TENs unit could cause paralysis if the client gets the unit wet 3. The TENs unit stimulates the nerves in the area, blocking the pain sensation 4. The TENs unit should be left on for an hour, and then taken off for an hour

3. The TENs unit stimulates the nerves in the area, blocking the pain sensation. The TENs unit works as a gate control theory of pain control and works by flooding the area with stimulation and blocking the pain impulses from reaching the brain

The nurse is caring for clients on a cardiac floor. Which client should the nurse assess first? 1. The client with three (3) unifocal PVCs in one (1) minute. 2. The client diagnosed with coronary artery disease who wants to ambulate. 3. The client diagnosed with mitral valve prolapse with an audible S3. 4. The client diagnosed with pericarditis who is in normal sinus rhythm.

3. The client diagnosed with mitral valve prolapse with an audible S3 An audible S3 indicates the client is developing left-sided heart failure and needs to be assessed immediately.

The client diagnosed with a myocardial infarction is six (6) hours post-right femoral percutaneous transluminal coronary angioplasty (PTCA), also known as balloon surgery. Which assessment data would require immediate intervention by the nurse? 1. The client is keeping the affected extremity straight. 2. The pressure dressing to the right femoral area is intact. 3. The client is complaining of numbness in the right foot. 4. The client's right pedal pulse is 3+ and bounding.

3. The client is complaining of numbness in the right foot. 1. After PTCA, the client must keep the right leg straight for at least six (6) to eight (8) hours to prevent any arterial bleeding from the insertion site in the right femoral artery. 2. A pressure dressing is applied to the insertion site to help prevent arterial bleeding. 3. Any neurovascular assessment data that are abnormal require intervention by the nurse; numbness may indicate decreased blood supply to the right foot. 4. A bounding pedal pulse indicates that adequate circulation is getting to the right foot; therefore, this would not require immediate intervention. TEST-TAKING HINT: This question requires the test taker to identify abnormal, unexpected, or life-threatening data. The nurse must know that a PTCA is performed by placing a catheter in the femoral artery and that internal or external bleeding is the most common complication.

65. The client is admitted to the medical unit with a diagnosis of rule-out diabetes insipidus (DI). Which instructions should the nurse teach regarding a fluid deprivation test? 1. The client will be asked to drink 100 mL of fluid as rapidly as possible and then will not be allowed fluid for 24 hours. 2. The client will be administered an injection of antidiuretic hormone, and urine output will be measured for four (4) to six (6) hours. 3. The client will be NPO, and vital signs and weights will be done hourly until the end of the test. 4. An IV will be started with normal saline, and the client will be asked to try to hold the urine in the bladder until a sonogram can be done.

3. The client is deprived of all fluids, and if the client has DI the urine production will not diminish. Vital signs and weights are taken every hour to determine circulatory status. If a marked decrease in weight or vital signs occurs, the test is immediately terminated.

6. The client with an AD tells the nurse, "I have changed my mind about my AD. I really want everything possible done if I am near death since I have a grandchild." Which action should the nurse implement? 1. Notify the health information systems department to talk to the client. 2. Remove the AD from the client's chart and shred the document. 3. Inform the client he or she has the right to revoke the AD at any time. 4. Explain this document cannot be changed once it is signed.

3. The client must be informed the AD can be rescinded or revoked at any time for any reason verbally, in writing, or by destroying his or her own AD. The nurse cannot destroy the client's AD, but the client can destroy his or her own.

The client with an AD tells the nurse, "I have changed my mind about my AD. I really want everything possible done if I am near death since I have a grandchild." Which action should the nurse implement? 1. Notify the health information systems department to talk to the client. 2. Remove the AD from the client's chart and shred the document. 3. Inform the client he or she has the right to revoke the AD at any time. 4. Explain this document cannot be changed once it is signed.

3. The client must be informed the AD can be rescinded or revoked at any time for any reason verbally, in writing, or by destroying his or her own AD. The nurse cannot destroy the client's AD, but the client can destroy his or her own.

In which client situation would the AD be consulted and used in decision making? 1. The client diagnosed with Guillain-Barré who is on a ventilator. 2. The client with a C6 spinal cord injury in the rehabilitation unit. 3. The client in end-stage renal disease who is in a comatose state. 4. The client diagnosed with cancer who has Down syndrome.

3. The client must have lost decisionmaking capacity as a result of a condition which is not reversible or must be in a condition specified under state law, such as a terminal, persistent vegetative state; an irreversible coma; or as speci- fied in the AD.

The client has just returned from a cardiac catheterization. Which assessment data would warrant immediate intervention from the nurse? 1. The client's BP is 110/70 and pulse is 90. 2. The client's groin dressing is dry and intact. 3. The client refuses to keep the leg straight. 4. The client denies any numbness and tingling.

3. The client refuses to keep the leg straight. 1. These vital signs are within normal limits and would not require any immediate intervention. 2. The groin dressing should be dry and intact. 3. If the client bends the leg, it could cause the insertion site to bleed. This is arterial blood and the client could bleed to death very quickly, so this requires immediate intervention. 4. The nurse must check the neurovascular assessment, and paresthesia would warrant immediate intervention, but no numbness and tingling is a good sign. TEST-TAKING HINT: "Warrants immediate intervention" means the nurse should probably notify the health-care provider or do something independently because a complication may occur. Therefore, the test taker must select an answer option that is abnormal or unsafe. In the data listed, there are three normal findings and one abnormal finding.

42. The client is being admitted to the outpatient department prior to an endoscopic retrograde cholangiopancreatogram (ERCP) to rule out cancer of the pancreas. Which preprocedure instruction should the nurse teach? 1. Prepare to be admitted to the hospital after the procedure for observation. 2. If something happens during the procedure, then emergency surgery will be done. 3. Do not eat or drink anything after midnight the night before the test. 4. If done correctly, this procedure will correct the blockage of the stomach.

3. The client should be NPO after midnight to make sure the stomach is empty to reduce the risk of aspiration during the procedure.

71. The emergency department nurse writes the problem of "ineffective coping" for a client who has been raped. Which intervention should the nurse implement? 1. Encourage the client to take the "morning-after" pill. 2. Allow the client to admit guilt for causing the rape. 3. Provide a list of rape crisis counselors. 4. Discuss reporting the case to the police.

3. The client should be provided the phone number of a rape crisis counseling center or counselor to help the client deal with the psychological feelings of being raped. 1. This plan for the client to take RU 486, or the "morning-after" pill, prevents pregnancy from occurring, but it does not directly address coping skills. 2. The client may talk about "what if I had not done...," but the client is not guilty of causing the rape. 4. This is a legal issue TEST-TAKING HINT: The test taker should read the stem "ineffective coping" and eliminate the physiological problem in option "1" and the legal problem in option "4."

45. The nurse is discharging a client with a nosocomial urinary tract infection. Which information should the nurse include in the discharge teaching? 1. Limit fluid intake so the urinary tract can heal. 2. Collect a routine urine specimen for culture. 3. Take all antibiotics as prescribed. 4. Be sure to void every five (5) to six (6) hours.

3. The client should be taught to take all the prescribed medication any time a prescription is written for antibiotics.

The client has just received a mechanical valve replacement. Which behavior by the client indicates the client needs more teaching? 1. The client takes prophylactic antibiotics. 2. The client uses a soft-bristle toothbrush. 3. The client takes an enteric-coated aspirin daily. 4. The client alternates rest with activity.

3. The client takes an enteric-coated aspirin daily. 1. Prophylactic antibiotics before invasive procedures prevent infectious endocarditis. 2. The client is undergoing anticoagulant therapy and should use a soft-bristle toothbrush to help prevent gum trauma and bleeding. 3. Aspirin and nonsteroidal antiinflammatory drugs (NSAIDs) interfere with clotting and may potentiate the effects of the anticoagulant therapy, which the client with a mechanical valve will be prescribed. Therefore, the client should not take aspirin daily. 4. The client should alternate rest with activity to prevent fatigue to help decrease the workload of the heart. TEST-TAKING HINT: The stem asks the test taker to identify which behavior means the client does not understand the teaching. Therefore, the test taker should select the distracter that does not agree with the condition. There is no condition for which alternating rest with activity would not be recommended.

30. The nurse is discharging the client diagnosed with bronchiolitis obliterans. Which priority intervention should the nurse include? 1. Refer the client to the American Lung Association. 2. Notify the physical therapy department to arrange for activity training. 3. Arrange for oxygen therapy to be used at home. 4. Discuss advance directives with the client.

3. The client with bronchiolitis obliterans will need long-term use of oxygen.

21. Which outcome should the nurse identify for the client diagnosed with fluid volume excess? 1. The client will void a minimum of 30 mL per hour. 2. The client will have elastic skin turgor. 3. The client will have no adventitious breath sounds. 4. The client will have a serum creatinine of 1.4 mg/dL.

3. The client with fluid volume excess has too much fluid. Excess fluid is reflected by adventitious breath sounds. Therefore, an expected outcome is to have no excess fluid, as evidenced by normal, clear breath sounds.

75. The client diagnosed with hypothyroidism is prescribed the thyroid hormone levothyroxine (Synthroid). Which assessment data indicate the medication has been effective? 1. The client has a three (3)-pound weight gain. 2. The client has a decreased pulse rate. 3. The client's temperature is WNL. 4. The client denies any diaphoresis.

3. The client with hypothyroidism frequently has a subnormal temperature, so a temperature WNL indicates the medication is effective.

10. The nurse is observing the UAP providing direct care to a client with an indwelling catheter. Which data warrant immediate intervention by the nurse? 1. The UAP secures the tubing to the client's leg with tape. 2. The UAP provides catheter care with the client's bath. 3. The UAP puts the collection bag on the client's bed. 4. The UAP cares for the catheter after washing the hands.

3. The drainage bag should be kept below the level of the bladder to prevent reflux of urine into the renal system; it should not be placed on the bed.

70. The unlicensed assistive personnel (UAP) complains to the nurse she has filled the water pitcher four (4) times during the shift for a client diagnosed with a closed head injury and the client has asked for the pitcher to be filled again. Which intervention should the nurse implement first? 1. Tell the UAP to fill the pitcher with ice cold water. 2. Instruct the UAP to start measuring the client's I & O. 3. Assess the client for polyuria and polydipsia. 4. Check the client's BUN and creatinine levels.

3. The first action should be to determine if the client is experiencing polyuriaand polydipsia as a result of developing diabetes insipidus, a complication of the head trauma.

32. The client is immediate postprocedure endoscopic retrograde cholangiopancreatogram (ERCP). Which intervention should the nurse implement? 1. Assess for rectal bleeding. 2. Increase fluid intake. 3. Assess gag reflex. 4. Keep in supine position.

3. The gag reflex will be suppressed as a result of the local anesthesia applied to the throat to insert the endoscope into the esophagus; therefore, the gag reflex must be assessed prior to allowing the client to resume eating or drinking.

35. The nurse and an unlicensed nursing assistant are caring for a group of clients. Which nursing intervention should the nurse perform? 1. Measure the client's output from the indwelling catheter. 2. Record the client's intake and output on the I & O sheet. 3. Instruct the client on appropriate fluid restrictions. 4. Provide water for a client diagnosed with diabetes insipidus

3. The nurse cannot delegate teaching.

4

3. The nurse is discussing advance directives with the client. The client asks the nurse, "Why is this so important to do?" Which statement would be the nurse's best response? 1. "The federal government mandates this form must be completed by you." 2. "This will make sure your family does what you want them to do." 3. "Don't you think it is important to let everyone know your final wishes?" 4. "Because of technology, there are many options for end-of-life care."

7. Which nursing intervention is most important before attempting to catheterize a client? 1. Determine the client's history of catheter use. 2. Evaluate the level of anxiety of the client. 3. Verify the client is not allergic to latex. 4. Assess the client's sensation level and ability to void.

3. The nurse should always assess for allergies to latex prior to inserting a latex catheter or using a drainage system because, if the client is allergic to latex, use of it could cause a life-threatening reaction.

25. The post-anesthesia care nurse is caring for the client diagnosed with lung cancer who had a thoracotomy and is experiencing frequent premature ventricular contractions (PVCs). Which intervention should the nurse implement first? 1. Request STAT arterial blood gases. 2. Administer lidocaine intravenous push. 3. Assess for possible causes. 4. Request a STAT electrocardiogram.

3. The nurse should assess for possible causes of the PVCs; these causes may include hypoxia or hypokalemia.

23. The family is dealing with the imminent death of the client. Which information is most important for the nurse to discuss when planning interventions for the grieving process? 1. How angry are the family members about the death? 2. Which family member will be making decisions? 3. What previous coping skills have been used? 4. What type of funeral service has been planned?

3. The nurse should assess previous coping skills used by the family and build on those to assist the family in dealing with their loss. Coping mechanisms are learned behaviors and should be supported if they are healthy behaviors. If the client and family use unhealthy coping behaviors, then the nurse should attempt to guide the family to a counselor or support group.

4. Which intervention should the nurse implement first for the client who has had an incontinent episode? 1. Palpate the client's bladder to assess for urinary retention. 2. Obtain a bedside commode for the client. 3. Assist the client with changing the wet clothes. 4. Request the UAP to change the client's linens.

3. The nurse should first assist the client in getting out of the wet clothes prior to any other action. Wet clothes are embarrassing to the client and can lead to skin breakdown.

19. Which intervention should the nurse implement for the client who has had an ileal conduit? 1. Pouch the stoma with a one (1)-inch margin around the stoma. 2. Refer the client to the United Ostomy Association for discharge teaching. 3. Report to the health-care provider any decrease in urinary output. 4. Monitor the stoma for signs and symptoms of infection every shift. 20.

3. The output should be monitored to detect a decreased amount indicating an obstruction from edema or ureteral stenosis. Any decrease should be reported to the health-care provider.

18. The nurse is preparing the client for a polysomnography to confirm sleep apnea. Which preprocedure instruction should the nurse include? 1. The client should not eat or drink past midnight. 2. The client will receive a sedative for relaxation. 3. The client will sleep in a laboratory for evaluation. 4. The client will wear a monitor at home for this test.

3. The polysomnography is completed in a sleep laboratory to observe all the stages of sleep. Equipment is attached to the client to monitor depth and stage of sleep and movement, respiratory effort, and oxygen saturation level during sleep.

39. The family has requested a client with terminal cancer not be told of the diagnosis. The client tells the nurse, "I think something is really wrong with me, but the doctor says everything is all right. Do you know if there is something wrong with me?" Which response by the nurse would support the ethical principle of veracity? 1. "I think you should talk to your doctor about your concerns." 2. "What makes you think something is really wrong?" 3. "Your family has requested you not be told your diagnosis." 4. "The doctor would never tell you incorrect information."

3. The principle of veracity is the duty to tell the truth. This response is telling the client the truth.

46. The triage nurse has placed a disaster tag on the client. Which action warrants immediate intervention by the nurse? 1. The nurse documents the tag number in the disaster log. 2. The unlicensed assistive personnel documents vital signs on the tag. 3. The health-care provider removes the tag to examine the limb. 4. The LPN securely attaches the tag to the client's foot.

3. The tag should never be removed from the client until the disaster is over or the client is admitted and the tag becomes a part of the client's record. The HCP needs to be informed immediately of the action. 1. This is the correct procedure when tagging a client and does not warrant intervention. 2. Vital signs should be documented on the tag. The tag takes the place of the client's chart, so this does not warrant intervention. 4. The tag can be attached to any part of the client's body. TEST-TAKING HINT: This question is asking the test taker to identify an incorrect option for the situation. Sometimes asking which action is appropriate helps identify the correct answer.

54. The pregnant client asks the nurse about banking the cord blood. Which information should the nurse teach the client? 1. The procedure involves a lot of pain with a very poor result. 2. The client must deliver at a large public hospital to do this. 3. The client will be charged a yearly storage fee on the cells. 4. The stem cells can be stored for about four (4) years before they ruin.

3. There is an initial fee to process the stem cells and a yearly fee to maintain the stored stem cells until needed. Stem cells may be used by the infant in case of a devastating illness or can be donated at the discretion of the owner.

40. The nurse is examining a 15-year-old female who is complaining of pain, frequency, and urgency when urinating. After asking the parent to leave the room, which question should the nurse ask the client? 1. "When was your last menstrual cycle?" 2. "Have you noticed any change in the color of the urine?" 3. "Are you sexually active?" 4. "What have you taken for the pain?"

3. These are symptoms of cystitis, a bladder infection that may be caused by sexual intercourse resulting from the introduction of bacteria into the urethra during the physical act. A teenager may not want to divulge this information in front of the parent.

55. The charge nurse of an intensive care unit is making assignments for the night shift. Which client should be assigned to the most experienced intensive care nurse? 1. The client diagnosed with respiratory failure who is on a ventilator and requires frequent sedation. 2. The client diagnosed with lung cancer and iatrogenic Cushing's disease with ABGs of pH 7.35, PaO2 88, PaCO2 44, and HCO3 22. 3. The client diagnosed with Addison's disease who is lethargic and has a BP of 80/45, P 124, and R 28. 4. The client diagnosed with hyperthyroidism who has undergone a thyroidectomy two (2) days ago and has a negative Trousseau's sign.

3. This client has a low blood pressure and tachycardia. This client may be experiencing an addisonian crisis, a potentially life-threatening condition. The most experienced nurse should care for this client.

33. The client diagnosed with acute pancreatitis is in pain. Which position should the nurse assist the client to assume to help decrease the pain? 1. Recommend lying in the prone position with legs extended. 2. Maintain a tripod position over the bedside table. 3. Place in side-lying position with knees flexed. 4. Encourage a supine position with a pillow under the knees.

3. This fetal position decreases pain caused by the stretching of the peritoneum as a result of edema.

72. The male client diagnosed with syndrome of inappropriate antidiuretic hormone (SIADH) secondary to cancer of the lung tells the nurse he wants to discontinue the fluid restriction and does not care if he dies. Which action by the nurse is an example of the ethical principle of autonomy? 1. Discuss the information the client told the nurse with the health-care provider and significant other. 2. Explain it is possible the client could have a seizure if he drank fluid beyond the restrictions. 3. Notify the health-care provider of the client's wishes and give the client fluids as desired. 4. Allow the client an extra drink of water and explain the nurse could get into trouble if the client tells the health-care provider.

3. This is an example of autonomy (the client has the right to decide for himself).

80. The client diagnosed with cancer of the bladder states, "I have young children. I am too young to die." Which statement is the nurse's best response? 1. "This cancer is treatable and you should not give up." 2. "Cancer occurs at any age. It is just one of those things." 3. "You are afraid of dying and what will happen to your children." 4. "Have you talked to your children about your dying?"

3. This is an example of restating, a therapeutic technique used to clarify the client's feelings and encourage a discussion of those feelings.

13. The client diagnosed with end-stage renal disease (ESRD) is experiencing metabolic acidosis. Which statement best describes the scientific rationale for metabolic acidosis in this client? 1. There is an increased excretion of phosphates and organic acids, which leads to an increase in arterial blood pH. 2. A shortened life span of red blood cells because of damage secondary to dialysis treatments. This, in turn, leads to metabolic acidosis. 3. The kidney cannot excrete increased levels of acid because the kidneys cannot excrete ammonia or cannot reabsorb sodium bicarbonate. 4. An increase in nausea and vomiting causes a loss of hydrochloric acid and the respiratory system cannot compensate adequately.

3. This is the correct scientific rationale for metabolic acidosis occurring in the client with ESRD.

35. The nurse is caring for clients on a medical floor. Which client should the nurse assess first after the shift report? 1. The client with arterial blood gases of pH 7.36, PaCO2 40, HCO3 26, PaO2 90. 2. The client with vital signs of T 99˚F, P 101, R 28, and BP 120/80. 3. The client complaining of pain at a "10" on a 1-to-10 scale who can't localize it. 4. The client who is postappendectomy with pain at a "3" on a 1-to-10 scale.

3. This is typical of clients with chronic pain. They cannot localize the pain and frequently describe the pain as always being there, as disturbing rest, and as demoralizing. This client should be seen, and appropriate pain-control measures should be taken

57. The client who is postoperative TURP asks the nurse, "When will I know if I will be able to have sex after my TURP?" Which response would be most appropriate by the nurse? 1. "You seem anxious about your surgery." 2. "Tell me about your fears of impotency." 3. "Potency can return in six (6) to eight (8) weeks." 4. "Did you ask your doctor about your concern?"

3. This is usually the length of time clients need to wait prior to having sexual intercourse; this is the information that the client wants to know.

2. The client diagnosed with type 1 diabetes has a glycosylated hemoglobin (A1c ) of 8.1%. Which interpretation should the nurse make based on this result? 1. This result is below normal levels. 2. This result is within acceptable levels. 3. This result is above recommended levels. 4. This result is dangerously high.

3. This result parallels a serum blood glucose level of approximately 180 to 200 mg/dL. An A1c is a blood test reflecting average blood glucose levels over a period of three (3) months; clients with elevated blood glucose levels are at risk for developing long-term complications.

24. The client receiving hemodialysis is being discharged home from the dialysis center. Which instruction should the nurse teach the client? 1. Notify the HCP if oral temperature is 102F or greater. 2. Apply ice to the access site if it starts bleeding at home. 3. Keep fingernails short and try not to scratch the skin. 4. Encourage significant other to make decisions for the client.

3. Uremic frost, which results when the skin attempts to take over the function of the kidneys, causes itching, which can lead to scratching that results in a break in the skin.

72. The client is diagnosed with a uric acid stone. Which foods should the client eliminate from the diet to help prevent reoccurrence? 1. Beer and colas. 2. Asparagus and cabbage. 3. Venison and sardines. 4. Cheese and eggs.

3. Venison, sardines, goose, organ meats, and herrings are high purine foods, which should be eliminated from the diet to help prevent uric acid stones.

31. The client admitted for recurrent aspiration pneumonia is at risk for bronchiectasis. Which intervention should the nurse anticipate the health-care provider to order? 1. Administer intravenous antibiotics for seven (7) days. 2. Insert a subclavian line and initiate total parenteral nutrition. 3. Provide a low-calorie and low-sodium restricted diet. 4. Encourage the client to turn, cough, and deep breathe frequently.

31. 1. Antibiotics should be administered intravenously for seven (7) to 10 days. Bronchiectasis is an irreversible condition caused by repeated damage to the bronchial walls secondary to repeated aspiration of gastric contents.

Which population is at a higher risk for dying from a myocardial infarction? 1. Caucasian males. 2. Hispanic females. 3. Asian males. 4. African American females.

4. African American Females African American females are 35% more likely to die from coronary artery disease than any other population. This population has significantly higher rates of hypertension and it occurs at a younger age. The higher risk of death from an MI is also attributed to a delay in seeking emergency care—an average of 11 hours.

23. The client diagnosed with ESRD is receiving peritoneal dialysis. Which assessment data warrant immediate intervention by the nurse? 1. Inability to auscultate a bruit over the fistula. 2. The client's abdomen is soft, is nontender, and has bowel sounds. 3. The dialysate being removed from the client's abdomen is clear. 4. The dialysate instilled into the client was 1500 mL and that removed was 1500 mL.

4. Because the client is in ESRD, fluid must be removed from the body so the output should be more than the amount instilled. This assessment data requires intervention by the nurse.

29. The client is admitted with a serum sodium level of 110 mEq/L. Which nursing intervention should be implemented? 1. Encourage fluids orally. 2. Administer 10% saline solution IVPB. 3. Administer antidiuretic hormone intranasally. 4. Place on seizure precautions.

4. Clients with sodium levels less than 120 mEq/L are at risk for seizures as a complication. The lower the sodium level, the greater the risk of a seizure.

The client is three (3) hours post-myocardial infarction. Which data would warrant immediate intervention by the nurse? 1. Bilateral peripheral pulses 2+. 2. The pulse oximeter reading is 96%. 3. The urine output is 240 mL in the last four (4) hours. 4. Cool, clammy, diaphoretic skin.

4. Cool, clammy, diaphoretic skin Cold, clammy skin is an indicator of cardiogenic shock, which is a complication of MI and warrants all immediate interventions.

The client who is one (1) day postoperative coronary artery bypass surgery is exhibiting sinus tachycardia. Which intervention should the nurse implement? 1. Assess the apical heart rate for one (1) full minute. 2. Notify the client's cardiac surgeon. 3. Prepare the client for synchronized cardioversion. 4. Determine if the client is having pain.

4. Determine if the client is having pain. 1. The telemetry reading is accurate, and there is no need for the nurse to assess the client's heart rate. 2. There is no reason to notify the surgeon for a client exhibiting sinus tachycardia. 3. Synchronized cardioversion is prescribed for clients in acute atrial fibrillation or ventricular fibrillation with a pulse. 4. Sinus tachycardia means the sinoatrial node is the pacemaker, but the rate is greater than 100 because of pain, anxiety, or fever. The nurse must determine the cause and treat appropriately. There is no specific medication for sinus tachycardia. TEST-TAKING HINT: The test taker must use the nursing process to determine the correct option and select an option that addresses assessment, the first step of the nursing process. Because both option "1" and option "4" address assessment, the test taker must determine which option is more appropriate. How will taking the apical pulse help treat sinus tachycardia? Determining the cause for sinus tachycardia is the most appropriate intervention.

1. Which diagnostic test should the nurse anticipate the health-care provider ordering to rule out the diagnosis of asthma? 1. A bronchoscopy. 2. An immunoglobulin E. 3. An arterial blood gas. 4. A bronchodilator reversibility test.

4. During a bronchodilator reversibility test, the client's positive response to a bronchodilator confirms the diagnosis of asthma.

41. The client with a cold asks the nurse, "Is it all right to take echinacea for my cold?" Which statement is the nurse's best response? 1. "You should discuss that with your health-care provider." 2. "No, you should not take any type of herbal medicine." 3. "Yes, but do not take it for more than 3 days." 4. "Echinacea may help with the symptoms of your cold."

4. Echinacea is an herb that may reduce the duration and symptoms of the common cold, but nothing cures the common cold. If the client does not have comorbid conditions, is not taking other medications, and is not pregnant, herbal medications may be helpful in treating the common

16. The nurse is developing a nursing care plan for the client diagnosed with ESRD. Which nursing problem would have priority for the client? 1. Low self-esteem. 2. Knowledge deficit. 3. Activity intolerance. 4. Excess fluid volume.

4. Excess fluid volume is priority because of the stress placed on the heart and vessels, which could lead to heart failure, pulmonary edema, and death.

17. The nurse is caring for a dying client and the family. The male client is Muslim. Which intervention should the female nurse implement at the time of death? 1. Allow the wife to stay in the room during postmortem care. 2. Call the client's iman to perform last rites when the client dies. 3. Place incense around the bed, but do not allow anyone to light it. 4. Do not touch the body, and have the male family members perform care.

4. Females, including the spouse, are not allowed to touch a male's body after death. The nurse should respect this and allow the male members of the family or mosque to perform postmortem care.

50. The nurse is developing a plan of care for the client diagnosed with acquired immunodeficiency syndrome (AIDS) who has developed an infection in the adrenal gland. Which client problem is highest priority? 1. Altered body image. 2. Activity intolerance. 3. Impaired coping. 4. Fluid volume deficit.

4. Fluid volume deficit (dehydration) can lead to circulatory impairment and hyperkalemia. TEST-TAKING HINT: Assuming all of the problems listed apply to the client diagnosed with Addison's disease, two are psychosocial problems and two are physiological. Applying Maslow's hierarchy of needs, the two psychological problems can be ruled out as the highest priority. Of the two options remaining, activity intolerance is not life altering or threatening.

The unlicensed assistive personnel (UAP) tells the primary nurse that the client diagnosed with coronary artery disease is having chest pain. Which action should the nurse take first? 1. Tell the UAP to go take the client's vital signs. 2. Ask the UAP to have the telemetry nurse read the strip. 3. Notify the client's health-care provider. 4. Go to the room and assess the client's chest pain.

4. Go to the room and assess the client's chest pain. Assessment is the first step in the nursing process and should be implemented first; chest pain is priority.

The nurse and an unlicensed assistive personnel (UAP) are caring for four clients on a telemetry unit. Which nursing task would be best for the nurse to delegate to the UAP? 1. Assist the client to go down to the smoking area for a cigarette. 2. Transport the client to the intensive care unit via a stretcher. 3. Provide the client going home discharge-teaching instructions. 4. Help position the client who is having a portable x-ray done.

4. Help position the client who is having a portable x-ray done. 1. Allowing the UAP to take a client down to smoke is not cost effective and is not supportive of the medical treatment regimen that discourages smoking. 2. The client going to the ICU would be unstable, and the nurse should not delegate to a UAP any nursing task that involves an unstable client. 3. The nurse cannot delegate teaching. 4. The UAP can assist the x-ray technician in positioning the client for the portable x-ray. This does not require judgment. TEST-TAKING HINT: The test taker must be knowledgeable about the individual state's Nurse Practice Act regarding what a nurse may delegate to unlicensed assistive personnel. Generally, the answer options that require higher level of knowledge or ability are reserved for licensed staff.

The client is diagnosed with pericarditis. Which are the most common signs/symptoms the nurse would expect to find when assessing the client? 1. Pulsus paradoxus. 2. Complaints of fatigue and arthralgias. 3. Petechiae and splinter hemorrhages. 4. Increased chest pain with inspiration.

4. Increased chest pain with inspiration. 1. Pulsus paradoxus is the hallmark of cardiac tamponade; a paradoxical pulse is markedly decreased in amplitude during inspiration. 2. Fatigue and arthralgias are nonspecific signs/symptoms that usually occur with myocarditis. 3. Petechiae on the trunk, conjunctiva, and mucous membranes and hemorrhagic streaks under the fingernails or toenails occur with endocarditis. 4. Chest pain is the most common symptom of pericarditis, usually has an abrupt onset, and is aggravated by respiratory movements (deep inspiration, coughing), changes in body position, and swallowing. TEST-TAKING HINT: The test taker who has no idea what the answer is should apply the test-taking strategy of asking which body system is affected. In this case, it is the cardiac system, specifically the outside of the heart. The test taker should select the option that has something to do with the heart, which is either option "1" or option "4."

47. The nurse must be knowledgeable of ethical principles. Which is an example of the ethical principle of justice? 1. The nurse administers a placebo, and the client asks if it will help the pain. 2. The nurse accepts a work assignment in an area in which he or she is not experienced. 3. The nurse refuses to tell a family member the client has a positive HIV test. 4. The nurse provides an indigent client with safe and appropriate nursing care.

4. Justice involves the duty to treat all clients fairly, without regard to age, socioeconomic status, or any other variables. Providing safe and appropriate nursing care to all clients is an example of justice.

The client who has just had a percutaneous balloon valvuloplasty is in the recovery room. Which intervention should the recovery room nurse implement? 1. Assess the client's chest tube output. 2. Monitor the client's chest dressing. 3. Evaluate the client's endotracheal (ET) lip line. 4. Keep the client's affected leg straight.

4. Keep the client's affected leg straight. 1. Percutaneous balloon valvuloplasty is not an open-heart surgery; therefore, the chest will not be open and the client will not have a chest tube. 2. This is not an open-heart surgery; therefore, the client will not have a chest dressing. 3. The endotracheal (ET) tube is inserted if the client is on a ventilator, and this surgery does not require putting the client on a ventilator. 4. In this invasive procedure, performed in a cardiac catheterization laboratory, the client has a catheter inserted into the femoral artery. Therefore, the client must keep the leg straight to prevent hemorrhaging at the insertion site. TEST-TAKING HINT: If the test taker knows that the word "percutaneous" means "via the skin," then options "1" and "2" could be eliminated as possible correct answers.

48. The nurse caring for a client diagnosed with cancer of the pancreas writes the nursing diagnosis of "risk for altered skin integrity related to pruritus." Which intervention should the nurse implement? 1. Assess tissue turgor. 2. Apply antifungal creams. 3. Monitor bony prominences for breakdown. 4. Have the client keep the fingernails short.

4. Keeping the fingernails short will reduce the chance of breaks in the skin from scratching. TEST-TAKING HINT: The problem is "risk for skin breakdown." The etiology is "pruritus." Interventions address the etiology. Goals address the problem.

6. The nurse is giving an in-service on end-of life-issues. Which activity should the nurse encourage the participants to perform? 1. Discuss with another participant the death of a client. 2. Review the hospital postmortem care policy. 3. Justify not putting the client in a shroud after dying. 4. Write down their own beliefs about death and dying.

4. Many nurses are reluctant to discuss death openly with their clients because of their own anxieties about death. Therefore, coming face to face with the nurse's own mortality will address the fear of death.

1. The nurse is admitting a client diagnosed with acute renal failure (ARF). Which question would be most important for the nurse to ask during the admission interview? 1. "Have you recently traveled outside the United States?" 2. "Did you recently begin a vigorous exercise program?" 3. "Is there a chance you have been exposed to a virus?" 4. "What over-the-counter medications do you take regularly?"

4. Medications such as nonsteroidal antiinflammatory drugs (NSAIDs) and some herbal remedies are nephrotoxic; therefore, asking about medications is appropriate.

The nurse and PCA are caring for a group of clients in a pain clinic. Which intervention would be inappropriate to delegate to the PCA? 1. Assist to the bathroom the client diagnosed with intractable pain 2. Elevate the head of the bed for the client diagnosed with back pain 3. Perform passive range of motion for the client that is bed fast. 4. Monitor the potassium levels on a client about to recieve medication

4. Monitor the potassium levels on a client about to recieve medication. The nurse should monitor any lab work needed to administer a medication safely.

41. Which statement best describes the role of the medical-surgical nurse during a disaster? 1. The nurse may be assigned to ride in the ambulance. 2. The nurse may be assigned as a first assistant in the operating room. 3. The nurse may be assigned to crowd control. 4. The nurse may be assigned to the emergency department.

4. New settings and atypical roles for nurses may be required during disasters; medical-surgical nurses can provide first aid and may be required to work in unfamiliar settings. 1. The nurse should not leave the hospital area; the nurse must wait for the casualties to come to the facility. 2. This is a position requiring knowledge of instruments and procedures not common to the medical-surgical floor. 3. The people in this area are usually chaplains or social workers, not direct client care personnel. In a disaster, direct care personnel cannot be spared for this duty. TEST-TAKING HINT: The test taker should look at traditional nursing roles requiring nursing expertise and eliminate crowd control or riding in an ambulance.

5. The client diagnosed with ARF has a serum potassium level of 6.8 mEq/L. Which collaborative treatment would the nurse anticipate for the client? 1. Administer a phosphate binder. 2. Type and cross-match for whole blood. 3. Assess the client for leg cramps. 4. Prepare the client for dialysis.

4. Normal potassium level is 3.5-5.5 mEq/L. A level of a 6.8 mEq/L is life threatening and could lead to cardiac dysrhythmias. Therefore, the client may be dialyzed to decrease the potassium level quickly. This would be done with an order from a healthcare provider, so it is a collaborative intervention.

26. The nurse is caring for the postoperative client diagnosed with lung cancer recovering from a thoracotomy. Which data require immediate intervention by the nurse? 1. The client refuses to perform shoulder exercises. 2. The client complains of a sore throat and is hoarse. 3. The client has crackles that clear with cough. 4. The client is coughing up pink frothy sputum.

4. Pink frothy sputum indicates pulmonary edema and would require immediate intervention.

The nurse has written an outcome goal "demonstrates tolerance for increased activity" for a client diagnosed with congestive heart failure. Which intervention should the nurse implement to assist the client to achieve this outcome? 1. Measure intake and output. 2. Provide two (2)-g sodium diet. 3. Weigh client daily. 4. Plan for frequent rest periods.

4. Plan for frequent rest periods. 1. Measuring the intake and output is an appropriate intervention to implement for a client with CHF, but it does not address getting the client to tolerate activity. 2. Dietary sodium is restricted in clients with CHF, but this is an intervention for decreasing fluid volume, not for increasing tolerance for activity. 3. Daily weighing monitors fluid volume status, not activity tolerance. 4. Scheduling activities and rest periods allows the client to participate in his or her own care and addresses the desired outcome. TEST-TAKING HINT: With questions involving nursing diagnoses or goals and outcomes, the test taker should realize that all activities referred to in the answer options may be appropriate for the disease but may not be specific for the desired outcome. TEST-TAKING HINT: With questions involving nursing diagnoses or goals and outcomes, the test taker should realize that all activities referred to in the answer options may be appropriate for the disease but may not be specific for the desired outcome.

17. The client with chronic low back pain is having trouble sleeping at night. Which nonpharmacological therapy should the nurse teach the client? 1. Acupuncture. 2. Massage therapy. 3. Herbal remedies. 4. Progressive relaxation techniques.

4. Progressive relaxation techniques involve visualizing a specific muscle group and mentally relaxing each muscle; this can be taught to the client, and it will allow the client to relax, which will foster sleep.

Which intervention should the nurse implement when defibrillating a client who is in ventricular fibrillation? 1. Defibrillate the client at 50, 100, and 200 joules. 2. Do not remove the oxygen source during defibrillation. 3. Place petroleum jelly on the defibrillator pads. 4. Shout "all clear" prior to defibrillating the client.

4. Shout "all clear" prior to defibrillating the client. 1. The adult client should be defibrillated at 360 joules. 2. The oxygen source should be removed to prevent any type of spark during defibrillation. 3. The nurse should use defibrillator pads or defibrillator gel to prevent any type of skin burns while defibrillating the client. 4. If any member of the health-care team is touching the client or the bed during defibrillation, that person could possibly be shocked. Therefore, the nurse should shout "all clear." TEST-TAKING HINT: The test taker should always consider the safety of the client and the health-care team. Options "2" and "3" put the client at risk for injury during defibrillation.

3. The nurse is discussing advance directives with the client. The client asks the nurse, "Why is this so important to do?" Which statement would be the nurse's best response? 1. "The federal government mandates this form must be completed by you." 2. "This will make sure your family does what you want them to do." 3. "Don't you think it is important to let everyone know your final wishes?" 4. "Because of technology, there are many options for end-of-life care."

4. Technology now allows for the body to maintain life functions indefinitely in some futile situations. ADs allow clients to make decisions which hopefully will be honored at the time of their death.

43. Which document is the best professional source to provide direction for a nurse when addressing ethical issues and behavior? 1. The Hippocratic Oath. 2. The Nuremberg Code. 3. Home Health Care Bill of Rights. 4. ANA Code of Ethics.

4. The American Nurses Association (ANA) Code of Ethics outlines to society the values, concerns, and goals of the nursing profession. The code provides direction for ethical decisions and behavior by emphasizing the obligations and responsibilities which are entailed in the nurse-client relationship.

4

4. The ED nurse is caring for a client who suffered a near-drowning. Which expected outcome should the nurse include in the plan of care for this client? 1. Maintain the client's cardiac function. 2. Promote a continued decrease in lung surfactant. 3. Warm rapidly to minimize the effects of hypothermia. 4. Keep the oxygen saturation level above 93%.

38. Which act protects the nurse against a malpractice claim when the nurse stops at a motor-vehicle accident and renders emergency care? 1. The Health Insurance Portability and Accountability Act. 2. The State Nurse Practice Act. 3. The Emergency Rendering Aid Act. 4. The Good Samaritan Act.

4. The Good Samaritan Act protects health-care practitioners against malpractice claims for care provided in emergency situations.

The nurse is presenting an in-service discussing do not resuscitate (DNR) orders and advance directives. Which statement should the nurse discuss with the class? 1. Advance directives must be notarized by a notary public. 2. The client must use an attorney to complete the advanced directive. 3. Once the DNR is written, it can be used for every hospital admission. 4. The health-care provider must write the DNR order in the client's chart.

4. The HCP writes the DNR order in the client's chart, and the client completes the AD.

The nurse is presenting an in-service discussing do not resuscitate (DNR) orders and advance directives. Which statement should the nurse discuss with the class? 1. Advance directives must be notarized by a notary public. 2. The client must use an attorney to complete the advanced directive. 3. Once the DNR is written, it can be used for every hospital admission. 4. The health-care provider must write the DNR order in the client's chart

4. The HCP writes the DNR order in the client's chart, and the client completes the AD. TEST-TAKING HINT: Options "1" and "2" involve other legal entities outside the health-care arena, which would make the test taker eliminate them. Content - Medical: Category of Health Alteration - Patient Advocacy: Integrated Nursing Process - Planning: Client Needs - Safe Effective Care Environment, Management of Care: Cognitive Level - Synthesis.

The client is being evaluated for valvular heart disease. Which information would be most significant? 1. The client has a history of coronary artery disease. 2. There is a family history of valvular heart disease. 3. The client has a history of smoking for 10 years. 4. The client has a history of rheumatic heart disease.

4. The client has a history of rheumatic heart disease. 1. An acute myocardial infarction can damage heart valves, causing tearing, ischemia, or damage to heart muscles that affects valve leaflet function, but coronary heart disease does not cause valvular heart disease. 2. Valvular heart disease does not show a genetic etiology. 3. Smoking can cause coronary artery disease, but it does not cause valvular heart disease. 4. Rheumatic heart disease is the most common cause of valvular heart disease. TEST-TAKING HINT: The test taker could rule out option "1" because of knowledge of anatomy: Coronary artery disease has to do with blood supply to heart muscle, whereas the valves are a part of the anatomy of the heart.

25. The nurse is assessing a client diagnosed with chronic pain. Which characteristics would the nurse observe? 1. The client's blood pressure is elevated. 2. The client has rapid shallow respirations. 3. The client has facial grimacing. 4. The client is lying quietly in bed.

4. The client in chronic pain will have adapted to living with the pain, and lying quietly may be the best way for the client to limit the feeling of pain.

69. The nurse is caring for a client diagnosed with diabetes insipidus (DI). Which intervention should be implemented? 1. Administer sliding-scale insulin as ordered. 2. Restrict caffeinated beverages. 3. Check urine ketones if blood glucose is >250. 4. Assess tissue turgor every four (4) hours.

4. The client is excreting large amounts of dilute urine. If the client is unable to drink enough fluids, the client will quickly become dehydrated, so tissue turgor should be assessed frequently. TEST-TAKING HINT: Two (2) of the answer options are appropriate for diabetes mellitus, not diabetes insipidus, and can be eliminated on this basis alone.

The nurse is assessing a client diagnosed with chronic pain. Which characteristics would the nurse observe? 1. The client's blood pressure is elevated 2. The client has rapid shallow respirations 3. The client has facial grimacing 4. The client is laying quietly in bed

4. The client is laying quietly in bed. The client with chronic pain will have adapted to living with pain, and lying quietly may be the best way for the client to limit the feeling of pain

15. The client diagnosed with end-stage congestive heart failure and type 2 diabetes is receiving hospice care. Which action by the nurse demonstrates an understanding of the client's condition? 1. The nurse monitors the blood glucose four (4) times a day. 2. The nurse keeps the client on a strict fluid restriction. 3. The nurse limits the visitors the client can receive. 4. The nurse brings the client a small piece of cake.

4. The client may have diabetes, but the client is also terminal, and allowing some food for pleasure is understanding of the client's life expectancy

The client asks the nurse, "When will the durable power of attorney for health care take effect?" On which scientific rationale would the nurse base the response? 1. It goes into effect when the client needs someone to make financial decisions. 2. It will be effective when the client is under general anesthesia during surgery. 3. The client must say it is all right for it to become effective and enforced. 4. It becomes valid only when the clients cannot make their own decisions.

4. The client must have lost decisionmaking capacity as a result of a condition which is not reversible or must be in a condition which is specified under state law, such as a terminal, persistent vegetative state; an irreversible coma; or as specified in the AD.

9. The client asks the nurse, "When will the durable power of attorney for health care take effect?" On which scientific rationale would the nurse base the response? 1. It goes into effect when the client needs someone to make financial decisions. 2. It will be effective when the client is under general anesthesia during surgery. 3. The client must say it is all right for it to become effective and enforced. 4. It becomes valid only when the clients cannot make their own decisions.

4. The client must have lost decisionmaking capacity as a result of a condition which is not reversible or must be in a condition which is specified under state law, such as a terminal, persistent vegetative state; an irreversible coma; or as specified in the AD.

12. The client is 12 hours postoperative renal surgery. Which data warrant immediate intervention by the nurse? 1. The abdomen is soft, nontender, and rounded. 2. Pain is not felt with dorsal flexion of the foot. 3. The urine output is 60 mL for the past two (2) hours. 4. The client's trough vancomycin level is 24 mcg/mL.

4. The client who has restricted kidney function from surgery should be monitored for damage as a result of the use of aminoglycoside antibiotics, such as vancomycin, which are nephrotoxic. This level is high and warrants notifying the HCP. Content - Surgical: Category

1

4. The client who is of the Jewish faith died during the night. The nurse notified the family, who do not want to come to the hospital. Which intervention should the nurse implement to address the family's behavior? 1. Take no further action because this is an accepted cultural practice. 2. Notify the hospital supervisor and report the situation immediately. 3. Call the local synagogue and request the rabbi go to the family's home. 4. Assume the family does not care about the client and follow hospital protocol.

The client is diagnosed with acute pancreatitis. Which health-care provider's admitting order should the nurse question? 1. Bedrest with bathroom privileges. 2. Initiate IV therapy of D5W at 125 mL/hr. 3. Weigh client daily. 4. Low-fat, low-carbohydrate diet.

4. The client will be NPO, which will decrease stimulation of the pancreatic enzymes, resulting in decreased autodigestion of the pancreas, therefore decreasing pain. TEST-TAKING HINT: The test taker must determine which HCP's order is not

76. Which nursing intervention should be included in the plan of care for the client diagnosed with hyperthyroidism? 1. Increase the amount of fiber in the diet. 2. Encourage a low-calorie, low-protein diet. 3. Decrease the client's fluid intake to 1,000 mL/day. 4. Provide six (6) small, well-balanced meals a day.

4. The client with hyperthyroidism has an increased appetite; therefore, wellbalanced meals served several times throughout the day will help with the client's constant hunger.

14. The nurse is assessing a client diagnosed with urethral strictures. Which data support the diagnosis? 1. Complaints of frequency and urgency. 2. Clear yellow drainage from the urethra. 3. Complaints of burning during urination. 4. A diminished force and stream during voiding.

4. The client with urethral strictures will report a decrease in force and stream during voiding. The stricture is treated by dilation using small filiform bougies.

The nurse is administering a calcium channel blocker to the client diagnosed with a myocardial infarction. Which assessment data would cause the nurse to question administering this medication? 1. The client's apical pulse is 64. 2. The client's calcium level is elevated. 3. The client's telemetry shows occasional PVCs. 4. The client's blood pressure is 90/62.

4. The client's blood pressure is 90/62. 1. The apical pulse is within normal limits— 60 to 100 beats per minute. 2. The serum calcium level is not monitored when calcium channel blockers are given. 3. Occasional PVCs would not warrant immediate intervention prior to administering this medication. 4. The client's blood pressure is low, and a calcium channel blocker could cause the blood pressure to bottom out. TEST-TAKING HINT: The test taker must know when to question administering medications. The test taker is trying to select an option that, if the medication is administered, would cause serious harm to the client.

11. Which intervention should the nurse implement when caring for the client with a nephrostomy tube? 1. Change the dressing only if soiled by urine. 2. Clean the end of the connecting tubing with Betadine. 3. Clean the drainage system every day with bleach and water. 4. Assess the tube for kinks to prevent obstruction.

4. The nephrostomy tube should never be clamped or have kinks because an obstruction can cause pyelonephritis.

The client diagnosed with end-stage congestive heart failure and type 2 diabetes is receiving hospice care. Which action by the nurse demonstrates an understanding of the clients condition? 1. The nurse monitors the blood glucose 4 times a day 2. The nurse keeps the client on a strict restrictive diet 3. The nurse limits visitors the client can receive 4. The nurse brings the client a small amount of ice cream

4. The nurse brings the client a small amount of ice cream. The client may have diabetes, but the client is also terminal, and allowing some food for pleasure is understanding of the client's life expectancy.

45. The nurse is caring for a client who is confused and fell trying to get out of bed. There is no family at the client's bedside. Which action should the nurse implement first? 1. Contact a family member to come and stay with the client. 2. Administer a sedative medication to the client. 3. Place the client in a chair with a sheet tied around him or her. 4. Notify the health-care provider to obtain a restraint order.

4. The nurse must notify the health-care provider before putting the client in restraints. Restraints are used in an emergency situation and for a limited time, and must be for the protection of the client.

56. The client with a TURP who has a continuous irrigation catheter complains of the need to urinate. Which intervention should the nurse implement first? 1. Call the surgeon to inform the HCP of the client's complaint. 2. Give the client a narcotic medication for pain. 3. Tell him that the sensation happens frequently. 4. Assess the continuous irrigation catheter for patency.

4. The nurse should always assess any complaint before dismissing it as a commonly occurring problem.

67. The nurse working in the emergency department is admitting a 34-year-old female client for one of multiple admissions for spousal abuse. The client has refused to leave her husband or to press charges against him. Which action should the nurse implement? 1. Insist the woman press charges this time. 2. Treat the wounds and do nothing else. 3. Tell the woman her husband could kill her. 4. Give the woman the number of a woman's shelter.

4. The nurse should help the client to devise a plan for safety by giving the client the number of a safe house or a woman's shelter. 1. The nurse can encourage the client to press charges but has no right to insist. 2. The nurse should treat the wound and may find it frustrating the client will not press charges, but the nurse is obligated to provide the client information to help the client to get to a safe place. 3. The woman is more aware of this fact than the nurse. TEST-TAKING HINT: The test taker could eliminate option "3" based on common sense; the client lives in an abusive situation and realizes the abuser's potential more than the nurse. Option "2" could be eliminated by the phrase "do nothing else." Option "1" could be eliminated because of the principle of nurses empowering their clients, not overpowering them, which is what has been happening to the client already.

33. The nurse and an unlicensed assistive personnel (UAP) are caring for a group of clients in a pain clinic. Which intervention would be inappropriate to delegate to the UAP? 1. Assist the client diagnosed with intractable pain to the bathroom. 2. Elevate the head of the bed for a client diagnosed with back pain. 3. Perform passive range of motion for a client who is bedfast. 4. Monitor the potassium levels on a client about to receive medication.

4. The nurse should monitor any laboratory work needed to administer a medication safely.

The nurse is moving to another state which is part of the multistate licensure compact. Which information regarding ADs should the nurse be aware of when practicing nursing in other states? 1. The laws regarding ADs are the same in all the states. 2. Advance directives can be transferred from state to state. 3. A significant other can sign a loved one's advance directive. 4. Advance directives are state regulated, not federally regulated.

4. The state determines the definition of terms and requirements for an AD; individual states are responsible for specific legal requirements for ADs

4. The nurse is moving to another state which is part of the multistate licensure compact. Which information regarding ADs should the nurse be aware of when practicing nursing in other states? 1. The laws regarding ADs are the same in all the states. 2. Advance directives can be transferred from state to state. 3. A significant other can sign a loved one's advance directive. 4. Advance directives are state regulated, not federally regulated.

4. The state determines the definition of terms and requirements for an AD; individual states are responsible for specific legal requirements for ADs.

79. Which statement made by the client makes the nurse suspect the client is experiencing hyperthyroidism? 1. "I just don't seem to have any appetite anymore." 2. "I have a bowel movement about every 3 to 4 days." 3. "My skin is really becoming dry and coarse." 4. "I have noticed all my collars are getting tighter."

4. The thyroid gland (in the neck) enlarges as a result of the increased need for thyroid hormone production; an enlarged gland is called a goiter. TEST-TAKING HINT: If the test taker does not know the answer, sometimes thinking about the location of the gland or organ causing the problem may help the test taker select or rule out specific options.

The nurse is teaching a class on valve replacements. Which statement identifies a disadvantage of having a biological tissue valve replacement? 1. The client must take lifetime anticoagulant therapy. 2. The client's infections are easier to treat. 3. There is a low incidence of thromboembolism. 4. The valve has to be replaced frequently.

4. The valve has to be replaced frequently. 1. An advantage of having a biological valve replacement is that no anticoagulant therapy is needed. Anticoagulant therapy is needed with a mechanical valve replacement. 2. This is an advantage of having a biological valve replacement; infections are harder to treat in clients with mechanical valve replacement. 3. This is an advantage of having a biological valve replacement; there is a high incidence of thromboembolism in clients with mechanical valve replacement. 4. Biological valves deteriorate and need to be replaced frequently; this is a disadvantage of them. Mechanical valves do not deteriorate and do not have to be replaced often. TEST-TAKING HINT: This is an "except" question. The test taker might reverse the question and ask, "Which is an advantage of a biological valve?"—which might make answering the question easier.

22. Which intervention should the nurse implement to provide culturally sensitive health care to the European-American Caucasian elderly client who is terminal? 1. Discuss health-care issues with the oldest male child. 2. Determine if the client will be cremated or have an earth burial. 3. Do not talk about death and dying in front of the client. 4. Encourage the client's autonomy and answer questions truthfully.

4. The western Caucasian society values autonomy and truth telling in individual decision making

49. The mother of a 20-year-old African American male client receiving dialysis asks the nurse, "My son has been on the transplant list longer than that white woman. Why did she get the kidney?" Which statement is the nurse's best response? 1. "The woman was famous, and so more people will donate organs now." 2. "I understand you are upset your son is ill. Would you like to talk?" 3. "No one knows who gets an organ. You just have to wait and pray." 4. "The tissues must match or the body will reject the kidney and it will be wasted."

4. There are 27 known human leukocyte antigens (HLAs). HLAs have become the principal histocompatibility system used to match donors and recipients. The greater the number of matches, the less likely the client will reject the organ. Different races have different HLAs.

23. Which arterial blood gas results should the nurse expect in the client diagnosed with diabetic ketoacidosis? 1. pH 7.34, PaO2 99, PaCO2 48, HCO3 24. 2. pH 7.38, PaO2 95, PaCO2 40, HCO3 22. 3. pH 7.46, PaO2 85, PaCO2 30, HCO3 26. 4. pH 7.30, PaO2 90, PaCO2 30, HCO3 18.

4. This ABG indicates metabolic acidosis, which is expected in a client diagnosed with diabetic ketoacidosis. TEST-TAKING HINT: The client must know normal ABGs to be able to correctly answer this question. Normal ABGs are pH 7.35 to 7.45; PaO2 80 to 100; PaCO2 35 to 45; HCO3 22 to 26.

41. The client is in the psychiatric unit in a medical center. Which action by the psychiatric nurse is a violation of the client's legal and civil rights? 1. The nurse tells the client civilian clothes can be worn on the unit. 2. The nurse allows the client to have family visits during visiting hours. 3. The nurse delivers unopened mail and packages to the client. 4. The nurse listens to the client talking on the telephone to a friend.

4. This is a violation of the client's rights. The client has a right to have reasonable access to a telephone and the opportunity to have private conversations by telephone.

40. According to the North Atlantic Treaty Organization (NATO) triage system, which situation is considered a level red (Priority 1)? 1. Injuries are extensive and chances of survival are unlikely. 2. Injuries are minor and treatment can be delayed hours to days. 3. Injuries are significant but can wait hours without threat to life or limb. 4. Injuries are life threatening but survivable with minimal interventions.

4. This is called the Immediate Category. Individuals in this group can progress rapidly to Expectant if treatment is delayed. 1. This describes injures color-coded black or Priority 4 and is called the Expectant Category. 2. This is a description of injuries color-coded green or Priority 3 and is called the Minimal Category. 3. These are injures color-coded yellow or Priority 2 and is called the Delayed Category. TEST-TAKING HINT: This is basically a knowledge-based question, but often the color "red" indicates a high priority.

21. The client with multiple sclerosis who is becoming very debilitated tells the home health nurse the Hemlock Society sent information on euthanasia. Which question should the nurse ask the client? 1. "Why did you get in touch with the Hemlock Society?" 2. "Did you know this is an illegal organization?" 3. "Who do you know who has committed suicide?" 4. "What religious beliefs do you practice?"

4. This question must be asked because Judeo-Christian belief supports the view that suicide is a violation of natural law and the laws of God. The tenets of the Hemlock Society are in direct opposition to Judeo-Christian beliefs. If the client is agnostic, then this organization may be helpful to the client.

69. Which question is an appropriate interview question for the nurse to use with clients involved in abuse? 1. "I know you are being abused. Can you tell me about it?" 2. "How much does your spouse drink before he hits you?" 3. "What did you do to cause your spouse to get mad?" 4. "Do you have a plan if your partner becomes abusive?"

4. This statement assesses the abused client's safety (or a plan for safety). 1. Unless the nurse is being personally abused in the same manner the client is being abused and has seen the abuse taking place, the nurse cannot "know" the client is being abused. 2. Alcohol and drugs are implicated in the abuse of many clients, but not all abusers use alcohol or drugs. 3. This is agreeing with the abuser about the client causing the abuse. TEST-TAKING HINT: Option "3" could be eliminated because it blames the victim. Option "1" can be eliminated because the nurse should not tell the client "I know" unless the nurse has proof or has been in the situation.

4. The client diagnosed with type 1 diabetes is receiving Humalog, a rapid-acting insulin, by sliding scale. The order reads blood glucose level: <150, zero (0) units; 151 to 200, three (3) units; 201 to 250, six (6) units; >251, contact health-care provider. The unlicensed assistive personnel (UAP) reports to the nurse the client's glucometer reading is 189. How much insulin should the nurse administer to the client?

4. Three (3) units. The client's result is 189, which is between 151 and 200, so the nurse should administer 3 units of Humalog insulin subcutaneously. TEST-TAKING HINT: The test taker must be aware of the way the HCPs write medication orders. HCPs order insulin on a sliding scale according to a range of blood glucose levels. Content - Medical:

45. The nurse is assessing the client diagnosed with a lung abscess. Which information supports this diagnosis of lung abscess? 1. Tympanic sounds elicited by percussion over the site. 2. Inspiratory and expiratory wheezes heard over the upper lobes. 3. Decreased breath sounds with a pleural friction rub. 4. Asymmetric movement of the chest wall with inspiration.

4. To ensure the compliance with all medications regimens, the health department has adapted a directly observed therapy (DOT) where the nurse actually observes the client taking the medication every day.

46. The public health department nurse is caring for the client diagnosed with active tuberculosis who has been placed on directly observed therapy (DOT). Which statement best describes this therapy? 1. The nurse accounts for all medications administered to the client. 2. The nurse must complete federal, state, and local forms for this client. 3. The nurse must report the client to the Centers for Disease Control. 4. The nurse must watch the client take the medication daily.

4. To ensure the compliance with all medications regimens, the health department has adapted a directly observed therapy (DOT) where the nurse actually observes the client taking the medication every day.

16. The client diagnosed with HHNS was admitted yesterday with a blood glucose level of 780 mg/dL. The client's blood glucose level is now 300 mg/dL. Which intervention should the nurse implement? 1. Increase the regular insulin IV drip. 2. Check the client's urine for ketones. 3. Provide the client with a therapeutic diabetic meal. 4. Notify the HCP to obtain an order to decrease insulin.

4. When the glucose level is decreased to around 300 mg/dL, the regular insulin infusion therapy is decreased. Subcutaneous insulin will be administered per sliding scale. TEST-TAKING HINT: When two (2) options are the opposite of each other, they can either be eliminated or can help eliminate the other two options as incorrect answers. Options "2" and "3" do not have insulin in the answer; therefore, they should be eliminated as possible answers.

43. The client is diagnosed with cancer of the head of the pancreas. Which signs and symptoms should the nurse expect to assess? 1. Clay-colored stools and dark urine. 2. Night sweats and fever. 3. Left lower abdominal cramps and tenesmus. 4. Nausea and coffee-ground emesis.

43. 1. The client will have jaundice, claycolored stools, and tea-colored urine resulting from blockage of the bile drainage.

48. The nurse is preparing to hang the next bag of aminophylline, a bronchodilator, for the client diagnosed with asthma. The current theophylline level is 18 mcg/mL. Which intervention should the nurse implement? 1. Hang the next bag and continue the infusion. 2. Do not hang the next bag and decrease the rate. 3. Notify the health-care provider of the level. 4. Confirm the current serum theophylline level.

48. 1. The therapeutic level is 10 to 20 mcg/mL; therefore, the nurse should hang the bag and continue the infusion to maintain the aminophylline level.

The nurse is teaching basic cardiopulmonary resuscitation (CPR) to individuals in the community. Which is the order of basic CPR? List in order of performance. 1. Perform head-tilt chin-lift maneuver. 2. Give two (2) rescue breaths. 3. Look, listen, and feel for breathing. 4. Begin cardiac compressions. 5. Shake and shout.

5,1,3,2,4

5. Which client would be most likely to complete an advance directive? 1. A 55-year-old Caucasian person who is a bank president. 2. A 34-year-old Asian licensed practical nurse. 3. A 22-year-old Hispanic lawn care worker. 4. A 65-year-old African American retired cook.

5. 1. ADs are more frequently completed by white, middle- to upper-class individuals.

1

5. The nurse is assessing the client who suffered a near-drowning event. Which data require immediate intervention? 1. The onset of pink, frothy sputum. 2. An oral temperature of 97˚F. 3. An alcohol level of 100 mg/dL. 4. A heart rate of 100 beats/min.

50. Which nursing interventions should the nurse implement for the client who has a respiratory disorder? Select all that apply. 1. Administer oxygen via a nasal cannula. 2. Assess the client's lung sounds. 3. Encourage the client to cough and deep breathe. 4. Monitor the client's pulse oximeter reading. 5. Increase the client's fluid intake. 51. The client in the intensive care unit

50. 1. A client with a respiratory disorder may have decreased oxygen saturation; therefore, administering oxygen via a nasal cannula is appropriate. 2. The client's lung sounds should be assessed to determine how much air is being exchanged in the lungs. 3. Coughing and deep breathing will help the client expectorate sputum, thus clearing the bronchial tree. 4. The pulse oximeter evaluates how much oxygen is reaching the periphery. 5. Increasing fluids will help thin secretions, making them easier to expectorate.

51. The client in the intensive care unit (ICU) on a mechanical ventilator is bucking the ventilator, causing the alarms to sound, and is in respiratory distress. Which assessment data should the nurse obtain? List in the order of priority. 1. Assess the ventilator alarms. 2. Assess the client's pulse oximetry reading. 3. Assess the client's lung sounds. 4. Assess for symmetry of the client's chest expansion. 5. Assess the client's endotracheal tube for secretions.

51. In order of priority: 5, 2, 3, 4, 1. 5. The most common cause of bucking the ventilator is obstructed airway, which could be secondary to secretions in the airway, so assessing the client would be most appropriate. 2. Clients in the ICU are constantly monitored by pulse oximetry; therefore, the nurse should determine if the client has decreased oxygen saturation and if so, the nurse should start to "bag" the client. The client is in respiratory distress. 3. The nurse should assess the client's lung fields to determine if air movement is occurring since the client is in respiratory distress. 4. A complication of mechanical ventilation is a pneumothorax, and the nurse should assess for this since the client is in respiratory distress. 1. The machine is alerting the nurse there is a problem with the client; since the client is in respiratory distress, the client should be assessed first. If the client were not in distress, then the nurse should assess the machine first to determine which alarm is sounding.

54. The nurse is performing discharge teaching for a client diagnosed with Cushing's disease. Which statement by the client demonstrates an understanding of the instructions? 1. "I will be sure to notify my health-care provider if I start to run a fever." 2. "Before I stop taking the prednisone, I will be taught how to taper it off." 3. "If I get weak and shaky, I need to eat some hard candy or drink some juice." 4. "It is fine if I continue to participate in weekend games of tackle football."

54. 1. Cushing's syndrome/disease predisposes the client to develop infections as a result of the immunosuppressive nature of the disease.

4

6. The nurse is giving an in-service on end-of life-issues. Which activity should the nurse encourage the participants to perform? 1. Discuss with another participant the death of a client. 2. Review the hospital postmortem care policy. 3. Justify not putting the client in a shroud after dying. 4. Write down their own beliefs about death and dying.

63. Which laboratory value should be monitored by the nurse for the client diagnosed with diabetes insipidus? 1. Serum sodium. 2. Serum calcium 3. Urine glucose. 4. Urine white blood cells.

63. 1. The client will have an elevated sodium level as a result of low circulating blood volume. The fluid is being lost through the urine. Diabetes means "to pass through" in Greek, indicating polyuria, a symptom shared with diabetes mellitus. Diabetes insipidus is a totally separate disease process.

7. The home health nurse is completing the admission assessment for a 76-year-old client diagnosed with type 2 diabetes controlled with 70/30 insulin. Which intervention should be included in the plan of care? 1. Assess the client's ability to read small print. 2. Monitor the client's serum PT level. 3. Teach the client how to perform a hemoglobin A1c test daily. 4. Instruct the client to check the feet weekly.

7. 1. Age-related visual changes and diabetic retinopathy could cause the client to have difficulty in reading and drawing up insulin dosage accurately.

4

7. The ED nurse is caring for a male client admitted with carbon monoxide poisoning. Which intervention requires the nurse to notify the Rapid Response Team? 1. The client has expectorated black sputum. 2. The client reports trying to kill himself. 3. The client's pulse oximeter reading is 94%. 4. The client has stridor and reports dizziness.

1

8. The ED nurse is working triage. Which client should be triaged first? 1. A client who has multiple injuries from a motor-vehicle accident. 2. A client complaining of epigastric pain and nausea after eating. 3. An elderly client who fell and fractured the left femoral neck. 4. The client suffering from a migraine headache and nausea.

2

8. The Hispanic client who has terminal cancer is requesting a curandero to come to the bedside. Which intervention should the nurse implement? 1. Tell the client it is against policy to allow faith healers. 2. Assist with planning the visit from the curandero. 3. Refer the client to the pastoral care department. 4. Determine the reason the client needs the curandero.

80. The 68-year-old client diagnosed with hyperthyroidism is being treated with radioactive iodine therapy. Which interventions should the nurse discuss with the client? 1. Explain it will take up to a month for symptoms of hyperthyroidism to subside. 2. Teach the iodine therapy will have to be tapered slowly over one (1) week. 3. Discuss the client will have to be hospitalized during the radioactive therapy. 4. Inform the client after therapy the client will not have to take any medication.

80. 1. Radioactive iodine therapy is used to destroy the overactive thyroid cells. After treatment, the client is followed closely for three (3) to four (4) weeks until the euthyroid state is reached.

49. The nurse empted 2000 mL from the drainage bag of a continuous irrigation of a client who had a transurethral resection of the prostate (TURP). The amount of irrigation in the bag hanging was 3000 mL at the beginning of the shift. There was 1800 mL left in the bag eight (8) hours later. What is the correct urine output at the end of the eight (8) hours? _____

800 mL.

81. The nurse is teaching the client diagnosed with hyperthyroidism. Which information should be taught to the client? Select all that apply. 1. Notify the HCP if a three (3)-pound weight loss occurs in two (2) days. 2. Discuss ways to cope with the emotional lability. 3. Notify the HCP if taking over-the-counter medication. 4. Carry a medical identification card or bracelet. 5. Teach how to take thyroid medications correctly.

81. 1. Weight loss indicates the medication may not be effective and will probably need to be increased. 2. The client needs to know emotional highs and lows are secondary to hyperthyroidism. With treatment, this emotional lability will subside. 3. Any over-the-counter medications (for example, alcohol-based medications) may negatively affect the client's hyperthyroidism or medications being used for treatment. 4. This will help any HCP immediately know of the client's condition, especially if the client is unable to tell the HCP.

9. Which data requires immediate intervention by the nurse for the client diagnosed with asbestosis? 1. The client develops an S3 heart sound. 2. The client has clubbing of the fingers. 3. The client is fatigued in the afternoon. 4. The client has basilar crackles in all lobes.

9. 1. The appearance of S3 heart sounds indicates the client is developing heart failure, which is a medical emergency.

9. The diabetic educator is teaching a class on diabetes type 1 and is discussing sick-day rules. Which interventions should the diabetes educator include in the discussion? Select all that apply. 1. Take diabetic medication even if unable to eat the client's normal diabetic diet. 2. If unable to eat, drink liquids equal to the client's normal caloric intake. 3. It is not necessary to notify the health-care provider if ketones are in the urine. 4. Test blood glucose levels and test urine ketones once a day and keep a record. 5. Call the health-care provider if glucose levels are higher than 180 mg/dL.

9. 1. The most important issue to teach clients is to take insulin even if they are unable to eat. Glucose levels are increased with illness and stress. 2. The client should drink liquids such as regular cola or orange juice, or eat regular gelatin, which provide enough glucose to prevent hypoglycemia when receiving insulin. 5. The HCP should be notified if the blood glucose level is this high. Regular insulin may need to be prescribed to keep the blood glucose level within acceptable range. TEST-TAKING HINT: This is an alternatetype question having more than one correct answer. The test taker should read all options and determine if each is an appropriate intervention.

4

9. The nurse is providing discharge teaching for the client with intermaxillary wiring to repair a fractured mandible. Which statement by the client indicates teaching has been effective? 1. Iced alcoholic drinks may be consumed by using a straw. 2. Only one (1) food item should be consumed at one (1) time. 3. Carbonated sodas should be limited to two (2) daily. 4. Teeth can be brushed after tenderness and edema subside.

1,2,5

9. Which interventions should the nurse implement at the time of a client's death? Select all that apply. 1. Allow gaps in the conversation at the client's bedside. 2. Avoid giving the family advice about how to grieve. 3. Tell the family the nurse understands their feelings. 4. Explain this is God's will to prevent further suffering. 5. Allow the family time with the body in private.

The nurse is caring for an elderly client who is 8 hours postop hip replacement and is reporting incisional pain. Which intervention is priority for this client? A. Assist the client to sit in the bedside chair B. Initiate pain meds at the lowest dose C. Assess PERRLA D. Monitor the clients UO every hour

B. Initiate pain meds at the lowest dose

39. The nurse performs bladder irrigation through an indwelling catheter. The nurse instilled 90 mL of sterile normal saline. The catheter drained 710 mL. What is the client's output? ________

620 mL of urine.

Which client would be most likely to complete an advance directive? 1. A 55-year-old Caucasian person who is a bank president. 2. A 34-year-old Asian licensed practical nurse. 3. A 22-year-old Hispanic lawn care worker. 4. A 65-year-old African American retired cook.

. 1. ADs are more frequently completed by white, middle- to upper-class individuals.

4. Which isolation procedure should be instituted for the client admitted to rule out severe acute respiratory syndrome (SARS)? 1. Airborne isolation. 2. Droplet isolation 3. Reverse isolation. 4. Strict isolation.

...

77. The nurse is planning the care of a postoperative client with an ileal conduit. Which intervention should be included in the plan of care? 1. Provide meticulous skin care and pouching. 2. Apply sterile drainage bags daily. 3. Monitor the pH of the urine weekly. 4. Assess the stoma site every day.

1. Urine is acidic and the abdominal wall tissue is not designed to tolerate acidic environments. The stoma is pouched so that urine will not touch the skin.

1

14. The health-care facility has been notified an alleged inhalation anthrax exposure has occurred at the local post office. Which category of personal protective equipment (PPE) should the response team wear? 1. Level A. 2. Level B. 3. Level C. 4. Level D.

3

16. The school nurse is caring for a child with a deep laceration. Which intervention should the nurse implement first? 1. Clean with saline solution. 2. Apply direct pressure. 3. Don nonsterile gloves. 4. Notify the child's parents.

19. The client in the intensive care unit diagnosed with end-stage chronic obstructive pulmonary disease has a Swan-Ganz mean pulmonary artery pressure of 35 mm Hg. Which health-care provider order would the nurse question? 1. Administer intravenous fluids of normal saline at 125 mL/hr. 2. Provide supplemental oxygen per nasal cannula at 2 L/min. 3. Continuous telemetry monitoring with strips every four (4) hours. 4. Administer a loop diuretic intravenously every six (6) hours.

19. 1. Normal mean pulmonary artery pressure is about 15 mm Hg and an elevation indicates right ventricular heart failure or cor pulmonale, which is a comorbid condition of chronic obstructive pulmonary disease. The nurse should question this order because the rate is too high.

2

19. The client diagnosed with chronic pain is laughing and joking with visitors. When the nurse asks the client to rate the pain on a 1-to-10 scale, the client rates the pain as 10. According to the pain scale, how would the nurse chart the client's pain? 1. The client's pain is between a zero (0) and two (2) on the faces scale. 2. The client's pain is a "10" on a 1-to-10 pain scale. 3. The client is unable to accurately rate the pain on a scale. 4. The client's pain is moderate on the pain scale.

3. The nurse administered 28 units of Humulin N, an intermediate-acting insulin, to a client diagnosed with type 1 diabetes at 1600. Which intervention should the nurse implement? 1. Ensure the client eats the bedtime snack. 2. Determine how much food the client ate at lunch. 3. Perform a glucometer reading at 0700. 4. Offer the client protein after administering insulin.

3. 1. Humulin N peaks in 6 to 8 hours, making the client at risk for hypoglycemia around midnight, which is why the client should receive a bedtime snack. This snack will prevent nighttime hypoglycemia.

The family is dealing with the imminent death of the client. Which information is most important for the nurse to discuss when planning interventions for the grieving process? 1. How angry are the family members about the death 2. Which family member will be making decisions 3. What previous coping skills have been used 4. Wah type of funeral service has been planned

3. What previous coping skills have been used. The nurse should access previous coping skills use by the family and build on those to assist the family in dealing with their loss. Coping mechanisms are learned behaviors and should be supported if they are health behaviors. If the client use unhealthy behaviors, the the nurse should attempt to guide the family to counselor or support group.

40. The nurse and an unlicensed assistive personnel (UAP) are caring for clients on an oncology floor. Which intervention should the nurse delegate to the UAP? 1. Assist the client with abdominal pain to turn to the side and flex the knees. 2. Monitor the Jackson Pratt drainage tube to ensure it is draining properly. 3. Check to see if the client is sleeping after pain medication is administered. 4. Empty the bedside commode of the client who has been having melena.

40. 1. The UAP can help a client to turn to the side and assume the fetal position, which decreases some abdominal pain.

57. The client diagnosed with Addison's disease is admitted to the emergency department after a day at the lake. The client is lethargic, forgetful, and weak. Which intervention should the nurse implement? 1. Start an IV with an 18-gauge needle and infuse NS rapidly. 2. Have the client wait in the waiting room until a bed is available. 3. Obtain a permit for the client to receive a blood transfusion. 4. Collect urinalysis and blood samples for a CBC and calcium level.

57. 1. The client was exposed to wind and sun at the lake during the hours prior to being admitted to the emergency department. This predisposes the client to dehydration and an addisonian crisis. Rapid IV fluid replacement is necessary.

To which client would the nurse question administering the osmotic diuretic mannitol (Osmitrol)? A. The client with 4+ pitting edema B. The client with decorticate posturing C. The client with widening pulse pressure D. The client with a positive doll's eye test

A

The client diagnosed with septicemia is receiving a broad-spectrum antibiotic. Which laboratory data require the nurse to notify the health-care provider? 1. The client's potassium level is 3.8 mEq/L. 2. The urine culture indicates high sensitivity to the antibiotic. 3. The client's pulse oximeter reading is 94%. 4. The culture and sensitivity is resistant to the client's antibiotic.

A sensitivity report indicating a resistance to the antibiotic being administered indicates the medication the client is receiving is not appropriate for the treatment of the infectious organism, and the HCP needs to be notified so the antibiotic can be changed.

A 58-year-old patient with Type 2 diabetes was admitted to the acute care unit with a diagnosis of COPD exacerbation. When you prepare a care plan for this patient, what would you be sure to include? SELECT ALL THAT APPLY A. Fingerstick BGL before meals and at bedtime B. Sliding-scale insulin as ordered C. Bed rest until COPD exacerbation resolved D. Teach about Atkins diet for weight loss E. Discussion of the components of foot care

A, B, E.

The nurse is administering digoxin to a client with CHF. Which interventions should the nurse implement? A. Check the apical heart rate for 1 minute B. Monitor the client's serum sodium level C. Teach the client how to take his or her radial pulse D. Evaluate the client's serum digoxin level E. Assess the client for buffalo hump and moon face.

A, C, D

The nurse is planning the patient assignment for the day. Which patient should the nurse assign to the unlicensed assistive personnel (UAP)? A. A patient on strict bed rest B. A patient scheduled for discharge home C. A patient scheduled for cardiac catheterization D. A postoperative patient after appendectomy

A. A patient on strict bed rest

The nurse and UAP are caring for clients in a bone marrow transplantation unit. Which nursing task should the nurse delegate? A. Take hourly V/S on a client receiving blood transfusions B. Monitor the infusion of antineoplastic medications C. Transcribe the HCP's orders onto the Medication Administration Record D. Determine the client's response to the therapy

A. After the first 15 minutes during which the client tolerates the blood transfusion, it is appropriate to ask the UAP to take the VS as long as the UAP has been given specific parameters for the V/S. Any V/S outside the normal parameters must have an intervention by the nurse.

The clinic nurse is caring for a 10-year-old boy with a diagnosis of type 2 diabetes being managed with therapeutic lifestyle change. Which problem is priority? A. Altered nutrition-excessive intake B. Risk for low self esteem C. Hypoglycemia D. Risk for loss of body part

A. Altered nutrition-excessive intake

The registered nurse (RN) has completed making client assignments for the shift and is preparing to delegate appropriate nursing care activities to the staff. Which tasks may the RN delegate to the certified nursing assistant (CNA)? Select all that apply. A. Ambulating a client in the hallway B. Recording I/O from meal tray C. Recording a clients oxygen saturation measurements D. Measuring and recording V/S E. Assess pressure ulcer risks using Braden Scale

A. Ambulating pt. B. Recording I/O from meal trays C. Recording a client's oxygen sat D. Measuring and recording V/S

The nurse is assessing the elderly client first thing in the morning. The client is confused and sleepy. Which intervention should the nurse implement first? A. Determine if the client received a sedative last night B. Allow the client to continue to sleep and do not disturb. C. Encourage the client to ambulate in the room with assistance D. Notify the health-care provider about the client's status.

A. Determine if the client received a sedative last night

After receiving intershift report on a medical nursing unit, the registered nurse (RN) prepares for the work of the day. Which of the following activities can the nurse delegate to a licensed practical/vocational nurse (LPN/LVN)? Select all that apply. A. Irrigating a nasogastric tube on a client receiving tube feedings B. Performing an ECG on a newly admitted patient C. Reassessing VS on a post-op patient with BP of 86/60 D. Changing a dressing on a patient with an infected food ulcer.

A. Irrigating a NGT on a client receiving tube feedings B. Performing an ECG on a newly admitted patient

The nurse and the LPN are caring for clients on an oncology floor. What client should not be assigned to the LPN? A. The client newly diagnosed with chronic lymphocytic leukemia B. The client who is 4 hours postprocedure bone marrow biopsy. C. The client who received two units of PRBCs on the previous shift D. The client who is receiving multiple IV piggyback medications

A. The client newly diagnosed with chronic lymphocytic leukemia. They will need teaching.

8. The nurse and an UAP are caring for a group of clients on a medical floor. Which action by the UAP warrants intervention by the nurse? A. The UAP places a urine specimen in a biohazard bag in the hallway B. The UAP uses the alcohol foam hand cleanser after removing gloves C. The UAP puts soiled linen in a plastic bag in the client's room D. The UAP obtains a disposable stethoscope for a client in an isolation room

A. Specimens should be placed in a biohazard bag in the patient's room.

2. The client is admitted into the ER with diaphoresis, pale clammy skin, and BP of 90/70. Which intervention should the nurse implement first? A. Start an IV with an 18G catheter B. Administer dopamine intravenous infusion C. Obtain ABGs D. Insert an indwelling urinary catheter

A. There are many types of shock, but the one common intervention which should be done is to establish an IV line with a large-bore catheter.

The client diagnosed with septicemia has the following health-care provider orders. Which HCP order has the highest priority? 1. Provide clear liquid diet. 2. Initiate IV antibiotic therapy. 3. Obtain a STAT chest x-ray. 4. Perform hourly glucometer checks.

An IV antibiotic is the priority medication for the client with an infection, which is the definition of sepsis—a systemic bacterial infection of the blood. A new order for an IV antibiotic should be implemented within one (1) hour of receiving the order.

17. The client with ESRD is placed on a fluid restriction of 1500 milliliters per day. On the 7 A.M. to 7 P.M. shift the client drank an eight (8)-ounce cup of coffee, 4 ounces of juice, 12 ounces of tea, and 2 ounces of water with medications. What amount of fluid can the 7 P.M. to 7 A.M. nurse give to the client? _____________

Answer: 720 mL.

The nurse in the emergency department administered an intramuscular antibiotic in the left gluteal muscle to the client with pneumonia who is being discharged home. Which intervention should the nurse implement? 1. Ask the client about drug allergies. 2. Obtain a sterile sputum specimen. 3. Have the client wait for 30 minutes. 4. Place a warm washcloth on the client's left hip.

Anytime a nurse administers a medication for the first time, the client should be observed for a possible anaphylactic reaction, especially with antibiotics.

Which task is most appropriate for the nurse to delegate to a UAP? A. Feed a client who is postoperative tonsillectomy the first meal of clear liquids B. Encourage the client diagnosed with a cold to drink a glass of OJ C. Obtain a throat culture on a client diagnosed with bacterial pharyngitis D. Escort the client diagnosed with laryngitis outside to smoke a cigarette

B.

What changed the structure of Medicare payments from a retrospectively adjusted cost reimbursement system to a prospective, risk-based one? A. Zero-based budgets B. Diagnostic-related groups C. Prospective payment system D. Incremental budgets

B. DRGs

The client tells the nurse that his cholesterol level is 240 mg/dL. Which action should the nurse implement first? A. Praise the client for having a normal cholesterol level B. Explain that the client needs to lower the cholesterol level C. Discuss dietary changes that could help increase the level. D. Allow the client to ventilate feelings about the blood test result

B. Explain that the client needs to lower the cholesterol level.

The pt's spouse is frustrated and tells the nurse," Everyone is telling me something different about when my wife can go home. I don't know who to believe!" Which statement is the nurse's best response? A. I can see you are frustrated. Would you like to talk about how you feel? B. I will contact the case manager and have her talk to you as soon as possible. C. Don't worry. Your wife won't go home until both you and she are ready. D. Your wife's healthcare provider should be able to give you all that information.

B. I will contact the case manager and have her talk to you as soon as possible.

The client is getting out of bed and becomes very anxious and has a feeling of impending doom. The nurse thinks the client may be experiencing a pulmonary embolism. Which action should the nurse implement first? A. Administer oxygen 10 L via nasal canula B. Place the client in high Fowler's position C. Obtain a STAT pulse oximeter reading D. Auscultate the client's lung sounds

B. Placing the client in this position facilitates maximal lung expansion and reduces venous return to the right side of the heart thus lowering pressures in the pulmonary vascular system.

The nurse, a LPN, and a UAP are caring for clients on a medical floor. Which nursing task would you assign to the LPN? A. Assist the UAP to learn to perform BGL checks B. Monitor the K+ levels of a client with diarrhea C. Administer bulk laxative to a client diagnosed with constipation D. Assess the abdomen of a client who has complaints of pain.

C.

The nurse is caring for a client diagnosed with pneumonia who is having SOB and difficulty breathing. Which intervention should the nurse implement first? A. Take V/S B. Check pulse oximeter C. Administer oxygen via NC D. Notify respiratory therapist STAT

C. Administer oxygen via NC. After elevating the HOB, the nurse should administer oxygen to the client who is in respiratory distress.

The overhead page has just announced a Code Red, actual fire, on a unit two floors below the unit where the nurse is working. Which action should the nurse implement first? A. Turn off oxygen supply to the rooms. B. Evacuate the patients to a lower floor. C. Close all of the doors to the patient's rooms. D. Make a list of patients ready for discharge.

C. Close all of the doors to the patient's rooms

The client has just returned from a cardiac catheterization. Which assessment data would warrant immediate intervention from the nurse? A. The client's BP is 110/70 and pulse is 90 B. The client's groin dressing is dry and intact C. The client refuses to keep the leg straight D. The client denies any numbness and tingling

C. If the client bends the leg, it could cause the insertion site to bleed. "warrants immediate intervention" means the nurse should probably notify the HCP or do something independently because a complication may occur.

The client is complaining of incisional pain. Which intervention should the nurse implement first? A. Administer the pain medication STAT B. Determine when the last pain medication was given C. Assess the client's pulse and blood pressure D. Teach the client distraction techniques to address pain.

C. The first step is to assess, and the nurse must determine if this is routine postoperative pain the client would have or if this is a complication which requires immediate attention. Decreased BP and increased pulse = blood loss = hypovolemic shock

The LPN is administering 0800 medications to clients on a medical floor. Which action by the LPN would warrant immediate intervention by the nurse? A. The LPN scores the medication to give the correct dose. B. The LPN checks the client's armband and birthdate C. The LPN administers sliding-scale insulin IM D. The LPN is 30 minutes late hanging the IV antibiotic

C. Insulin is delivered SQ

The client diagnosed with a myocardial infarction is on bedrest. The UAP is encouraging the client to move their legs. Which action should the nurse implement? A. Instruct the UAP to stop encouraging the leg movements B. Report this behavior to the charge nurse C. Praise the UAP for encouraging the client to move legs D. Take no action

C. Leg movement helps prevent DVTs

3. The nurse is caring for a client diagnosed with septic shock. Which assessment data would warrant immediate intervention by the nurse? A. V/S: T: 100.4, P: 104, R: 26, BP: 102/60 B. WBC 18,000 mm^3 C. UO of 90 mL last 4 hours D. The client complains of thirst

C. The client must have a urinary output of at least 30 ml/hr so 90 ml for 4 hours indicates renal impairment, a sign of worsening shock. "Warrant immediate intervention" mean the nurse must do something, which frequently is to notify the HCP.

26. The UAP is performing cardiac compressions on an adult client during a code. Which behavior warrants immediate intervention by the nurse? A. The UAP has hand placement on the lower half of the sternum B. The UAP performs cardiac compressions and allows for rescue breathing C. The UAP depresses the sternum 0.5 to one inch during compressions D. The UAP asks to be relieved from performing compressions because of exhaustion

C. The sternum should be depressed 1 - 1.5" to 2" during compressions to ensure adequate circulation of blood to the body.

The UAP reported an intake of 1000 mL and a urinary output of 1500 mL for a client who received thiazide diuretics this morning. Which nursing task could the nurse delegate to the UAP? A. Instruct the UAP to restrict the client's fluid intake B. Request the UAP to insert a Foley catheter with an urimeter C. Tell the UAP urinary outputs are no longer needed D. Ask the UAP to document fluids on the bedside I & O records

D

The 29 year old client is admitted to the medical floor diagnosed with meningitis. Which assessment by the nurse has priority? A. Assess lung sounds B. Assess the six cardinal fields of gaze. C. Assess apical pulse D. Assess level of consciousness

D. Assess LOC

Which intervention should the nurse implement first when caring for a client with a respiratory disorder? A. Administer a respiratory treatment B. Check the client's radial pulses daily. C. Check the client's VS daily D. Assess the client's capillary refill time

D. Assessing the client's cap refill time has the highest priority for the nurse because it indicates the oxygenation of the client

A parent calls the emergency department stating, "I think my toddler might have swallowed a little toy. He is breathing okay, but I don't know what to do." What is the most essential question to ask the caller? A. Has he vomited? B. Have you been checking his stools? C. What do you think he swallowed? D. Has he been coughing?

D. Has he been coughing?

The nurse and the UAP are caring for a 64 y/o client who is 4 hours post-op bilateral femoral-popliteal bypass surgery. Which nursing task should be delegated to the UAP? A. Monitor the continuous passive motion machine B. Assist the client to the bedside commode C. Feed the client the evening meal. D. Elevate the foot of the client's bed.

D. Elevate the food of the client's bed.

The client is admitted to the medical floor with a diagnosis of closed head injury. Which nursing interventions has priority? A. Assess neuro status B. Monitor pulse, respiration, and BP C. Initiate an IV access D. Maintain an adequate airway

D. Maintain an adequate airway

The nurse is caring for a patient with an acute head injury. The nurse should monitor which parameter as the priority? A. Urine output B. Electrolyte results C. Neurological changes D. Respiratory rate and rhythm

D. Respiratory rate and rhythm

The nurse and an unlicensed assistive personnel (UAP) are caring for a group of clients on a medical floor. Which action by the UAP warrants intervention by the nurse? 1. The UAP places a urine specimen in a biohazard bag in the hallway. 2. The UAP uses the alcohol foam hand cleanser after removing gloves. 3. The UAP puts soiled linen in a plastic bag in the client's room. 4. The UAP obtains a disposable stethoscope for a client in an isolation room.

Specimens should be put into biohazard bags prior to leaving the client's room.

The nurse is caring for a client diagnosed with septic shock who has hypotension, decreased urine output, and cool, pale skin. Which phase of septic shock is the client experiencing? 1. The hypodynamic phase. 2. The compensatory phase. 3. The hyperdynamic phase. 4. The progressive phase.

The hypodynamic phase is the last and irreversible phase of septic shock, characterized by low cardiac output with vasoconstriction. It reflects the body's effort to compensate for hypovolemia caused by the loss of intravascular volume through the capillaries.

The elderly female client with vertebral fractures who has been self-medicating with ibuprofen, a nonsteroidal anti-inflammatory drug (NSAID), presents to the ED complaining of abdominal pain, is pale and clammy, and has a P of 110 and a BP of 92/60. Which type of shock should the nurse suspect? 1. Cardiogenic shock. 2. Hypovolemic shock. 3. Neurogenic shock. 4. Septic shock.

These client's signs/symptoms make the nurse suspect the client is losing blood, which leads to hypovolemic shock, which is the most common type of shock and is characterized by decreased intravascular volume. The client's taking of NSAID medications puts her at risk for hemorrhage because NSAIDs inhibit prostaglandin production in the stomach, which increases the risk of developing ulcers, which can erode the stomach lining and lead to hemorrhaging.


Conjuntos de estudio relacionados

World Culture: set 19B; Aswan Dam

View Set

Freud and the Psychodynamic Perspective

View Set

OBGYN final comprehensive review

View Set

Rules of orbital diagrams and electrons

View Set